S.No. Content

You might also like

Download as pdf or txt
Download as pdf or txt
You are on page 1of 196

S.No.

Content Page

Syllabus I - VI

01. Chapter-1 : Relation and Function 01 – 26

02. Chapter-2 : Inverse Trigonometric Function 27 – 44

03. Chapter-3 : Matrices 45 – 72

04. Chapter-4 : Determinant 73 – 104

05. Chapter-5 : Continuity and Differentiability 105 – 148

06. Chapter-6 : Application of Derivatives 149 - 188


All rights including trademark and copyrights and rights of translation etc. reserved and vested exclusively with
Allen Career Institute Private Limited. (Allen)
No part of this work may be copied, reproduced, adapted, abridged or translated, transcribed, transmitted,
stored or distributed in any form retrieval system, computer system, photographic or other system or
transmitted in any form or by any means whether electronic, magnetic, chemical or manual, mechanical, digital,
optical, photocopying, recording or otherwise, or stood in any retrieval system of any nature without the written
permission of the Allen Career Institute Private Limited. Any breach will entail legal action and prosecution
without further notice.
This work is sold/distributed by Allen Career Institute Private Limited subject to the condition and undertaking
given by the student that all proprietary rights (under the Trademark Act, 1999 and Copyright Act, 1957) of the
work shall be exclusively belong to Allen Career Institute Private Limited. Neither the Study Materials and/or
Test Series and/or the contents nor any part thereof i.e. work shall be reproduced, modify, re-publish,
sub-license, upload on website, broadcast, post, transmit, disseminate, distribute, sell in market, stored in a
retrieval system or transmitted in any form or by any means for reproducing or making multiple copies of it.
Any person who does any unauthorised act in relation to this work may be liable to criminal prosecution and
civil claims for damages. Any violation or infringement of the propriety rights of Allen shall be punishable
under Section- 29 & 52 of the Trademark Act, 1999 and under Section- 51, 58 & 63 of the Copyright Act, 1957 and
any other Act applicable in India. All disputes are subjected to the exclusive jurisdiction of courts, tribunals and
forums at Kota, Rajasthan only.
Note:- This publication is meant for educational and learning purposes. All reasonable care and diligence have
been taken while editing and printing this publication. Allen Career Institute Private Limited shall not hold any
responsibility for any error that may have inadvertently crept in. Allen Career Institute Private Limited is not
responsible for the consequences of any action taken on the basis of this publication.


SYLLABUS
CLASS-XII

MATHEMATICS (2023-24)

One-paper Max Marks : 80

No. Units No. of Periods Marks

I. Relations and Functions 30 08

II. Algebra 50 10

III. Calculus 80 35

IV. Vectors and Three - Dimensional Geometry 30 14

V. Linear Programming 20 05

VI. Probability 30 08

Total 240 80

Internal Assessment 20

Unit-I: Relations and Functions


1. Relations and Functions 15 Periods
Types of relations: reflexive, symmetric, transitive and equivalence relations. One to one and
onto functions.
2. Inverse Trigonometric Functions 15 Periods
Definition, range, domain, principal value branch. Graphs of inverse trigonometric functions.
Unit-II: Algebra
1. Matrices 25 Periods
Concept, notation, order, equality, types of matrices, zero and identity matrix, transpose of a
matrix, symmetric and skew-symmetric matrices. Operation on matrices: Addition and
multiplication and multiplication with a scalar. Simple properties of addition, multiplication and
scalar multiplication. On- commutativity of multiplication of matrices and existence of non-zero
matrices whose product is the zero matrix (restrict to square matrices of order 2). Invertible
matrices and proof of the uniqueness of inverse, if it exists; (Here all matrices will have real
entries).

E I

2. Determinants 25 Periods
Determinant of a square matrix (up to 3 x 3 matrices), minors, co-factors and applications of
determinants in finding the area of a triangle. Adjoint and inverse of a square matrix.
Consistency, inconsistency and number of solutions of system of linear equations by examples,
solving system of linear equations in two or three variables (having unique solution) using
inverse of a matrix.
Unit-III: Calculus
1. Continuity and Differentiability 20 Periods
Continuity and differentiability, chain rule, derivative of inverse trigonometric functions, like
sin–1x, cos–1x and tan–1x, derivative of implicit functions. Concept of exponential and logarithmic
functions.
Derivatives of logarithmic and exponential functions. Logarithmic differentiation, derivative of
functions expressed in parametric forms. Second order derivatives.
2. Applications of Derivatives 10 Periods
Applications of derivatives: rate of change of bodies, increasing/decreasing functions, maxima
and minima (first derivative test motivated geometrically and second derivative test given as a
provable tool). Simple problems (that illustrate basic principles and understanding of the subject
as well as real-life situations).
3. Integrals 20 Periods
Integration as inverse process of differentiation. Integration of a variety of functions by
substitution, by partial fractions and by parts, Evaluation of simple integrals of the following
types and problems based on them.
dx dx dx dx dx
x 2
a 2 
,
x2  a2
, , 2
a 2  x 2 ax  bx  c
,
ax 2  bx  c
px  q px  q
 ax 2
 bx  c
dx,
ax  bx  c
2
dx, a 2  x 2 dx,  x 2  a 2 dx

 ax 2  bx  c dx

Fundamental Theorem of Calculus (without proof), Basic properties of definite integrals and
evaluation of definite integrals.
4. Applications of Integrals 15 Periods
Applications in finding the area under simple curves, especially lines, circles/ parabolas/ellipses
(in standard form only)

II E

5. Differential Equations 15 Periods

Definition, order and degree, general and particular solutions of a differential equation. Solution
of differential equations by method of separation of variables, solutions of homogeneous
differential equations of first order and first degree. Solutions of linear differential equation of
the type:
dy
 py  q, where p and q are functions of x or constants.
dx
dx
 px  q , where p and q are functions of y or constants.
dy
Unit-IV: Vector algebra and Three-Dimensional Geometry
1. Vector algebra 15 Periods
Vectors and scalars, magnitude and direction of a vector. Direction cosines and direction ratios of
a vector. Types of vectors (equal, unit, zero, parallel and collinear vectors), position vector of a
point, negative of a vector, components of a vector, addition of vectors, multiplication of a vector
by a scalar, position vector of a point dividing a line segment in a given ratio. Definition,
Geometrical Interpretation, properties and application of scalar (dot) product of vectors, vector
(cross) product of vectors.
2. Three - dimensional Geometry 15 Periods
Direction cosines and direction ratios of a line joining two points. Cartesian equation and vector
equation of a line, skew lines, shortest distance between two lines. Angle between two lines.
Unit-V: Linear Programming
1. Linear Programming 20 Periods
Introduction, related terminology such as constraints, objective function, optimization, graphical
method of solution for problems in two variables, feasible and infeasible regions (bounded or
unbounded), feasible and infeasible solutions, optimal feasible solutions (up to three non-trivial
constraints).
Unit-VI: Probability
1. Probability 30 Periods
Conditional probability, multiplication theorem on probability, independent events, total
probability, Bayes’ theorem, Random variable and its probability distribution, mean of random
variable.

E III

MATHEMATICS (Code No. - 041)
QUESTION PAPER DESIGN CLASS - XII
(2023-24)
Time: 3 hours Max. Marks: 80
S. No. Total %
Typology of Questions
Marks Weightage
1. Remembering: Exhibit memory of previously learned
material by recalling facts, terms, basic concepts, and
answers.
44 55
Understanding: Demonstrate understanding of facts and
ideas by organizing, comparing, translating, interpreting,
giving descriptions, and stating main ideas
2. Applying: Solve problems to new situations by applying
acquired knowledge, facts, techniques and rules in a different 20 25
way.
3. Analysing :
Examine and break information into parts by identifying
motives or causes. Make inferences and find evidence to
support generalizations
Evaluating:
Present and defend opinions by making judgments about
16 20
information, validity of ideas, or quality of work based on a
set of criteria.
Creating:
Compile information together in a different way by
combining elements in a new pattern or proposing
alternative solutions
Total 80 100

1. No chapter wise weightage. Care to be taken to cover all the chapters


2. Suitable internal variations may be made for generating various templates keeping the overall
weightage to different form of questions and typology of questions same.
Choice(s):
There will be no overall choice in the question paper.
However, 33% internal choices will be given in all the sections
INTERNAL ASSESSMENT 20 MARKS

Periodic Tests ( Best 2 out of 3 tests conducted) 10 Marks


Mathematics Activities 10 Marks
Note: For activities NCERT Lab Manual may be referred.
IV E

Conduct of Periodic Tests:
Periodic Test is a Pen and Paper assessment which is to be conducted by the respective subject
teacher. The format of periodic test must have questions items with a balance mix, such as, very
short answer (VSA), short answer (SA) and long answer (LA) to effectively assess the
knowledge, understanding, application, skills, analysis, evaluation and synthesis. Depending on
the nature of subject, the subject teacher will have the liberty of incorporating any other types of
questions too. The modalities of the PT are as follows:
(a) Mode: The periodic test is to be taken in the form of pen-paper test.
(b) Schedule: In the entire Academic Year, three Periodic Tests in each subject may be
conducted as follows:
Test Pre Mid-term (PT-I) Mid-Term (PT-II) Post Mid-Term (PT-III)
Tentative Month July-August November December-January
This is only a suggestive schedule and schools may conduct periodic tests as per their
convenience. The winter bound schools would develop their own schedule with similar time
gaps between two consecutive tests.
(c) Average of Marks: Once schools complete the conduct of all the three periodic tests, they
will convert the weightage of each of the three tests into ten marks each for identifying best
two tests. The best two will be taken into consideration and the average of the two shall be
taken as the final marks for PT.
(d) The school will ensure simple documentation to keep a record of performance as suggested
in detail circular no.Acad-05/2017.
(e) Sharing of Feedback/Performance: The students’ achievement in each test must be
shared with the students and their parents to give them an overview of the level of learning
that has taken place during different periods. Feedback will help parents formulate
interventions (conducive ambience, support materials, motivation and morale-boosting) to
further enhance learning. A teacher, while sharing the feedback with student or parent,
should be empathetic, non- judgmental and motivating. It is recommended that the teacher
share best examples/performances of IA with the class to motivate all learners.
Assessment of Activity Work:
Throughout the year any 10 activities shall be performed by the student from the activities given
in the NCERT Laboratory Manual for the respective class (XI or XII) which is available on the
link: http://www.ncert.nic.in/exemplar/labmanuals.htmla record of the same may be kept by
the student. An year end test on the activity may be conducted
The weightage are as under:
 The activities performed by the student throughout the year and record keeping : 5 marks
 Assessment of the activity performed during the year end test: 3 marks
 Viva-voce: 2 marks
Prescribed Books:
(1) Mathematics Part I - Textbook for Class XII, NCERT Publication
(2) Mathematics Part II - Textbook for Class XII, NCERT Publication
(3) Mathematics Exemplar Problem for Class XII, Published by NCERT
(4) Mathematics Lab Manual Class XII, published by NCERT

E V

IMPORTANT NOTES
__________________________________________________________________________________
__________________________________________________________________________________
__________________________________________________________________________________
__________________________________________________________________________________
__________________________________________________________________________________
__________________________________________________________________________________
__________________________________________________________________________________
__________________________________________________________________________________
__________________________________________________________________________________
__________________________________________________________________________________
__________________________________________________________________________________
__________________________________________________________________________________
__________________________________________________________________________________
__________________________________________________________________________________
__________________________________________________________________________________
__________________________________________________________________________________
__________________________________________________________________________________
__________________________________________________________________________________
__________________________________________________________________________________
__________________________________________________________________________________
__________________________________________________________________________________
__________________________________________________________________________________
__________________________________________________________________________________
__________________________________________________________________________________
__________________________________________________________________________________
__________________________________________________________________________________
__________________________________________________________________________________
__________________________________________________________________________________
__________________________________________________________________________________

VI E


CHAPTER–1 : RELATIONS AND FUNCTIONS


1. CARTESIAN PRODUCT OF TWO SETS :
Given two non-empty sets A and B. The cartesian product A × B is the set of all ordered pairs of
the form (a, b) where the first entry comes from set A & second comes from set B.
A × B = {(a, b) | a  A, b  B}
Example : A = {1, 2, 3}and B = {p, q}
A × B = {(1, p),(1, q), (2, p), (2, q), (3, p), (3, q)}
Note :
(i) If either A or B is a null set, then A × B will also be empty set, i.e. A × B = 
(ii) If n(A) = m, n(B) = n, then n(A × B) = mn.
2. RELATIONS :
Let A and B be two sets, then a relation R from A to B is a subset of A × B.
Thus, R is a relation from A to B  R  A × B.
Note :
(i) If (a, b)  R then b is the image of a under R and a is the pre-image of b under R.
(ii) If n(A) = m, n(B) = n, then the total number of relations defined from set A to B are 2mn.
3. DOMAIN AND RANGE OF A RELATION :
Let R be a relation from a set A to a set B. Then the set of all first components or coordinates of
the ordered pairs belonging to R is called the domain of R, while the set of all second
components or coordinates of the ordered pairs in R is called the range of R.
Thus, Domain (R) = {a : (a, b)  R}
And, Range (R) = {b : (a, b)  R}
It is evident from the definition that the domain of a relation from A to B is a subset of A and its
range is a subset of B.

Illustration 1: Let A = {1, 2, 3, 4} and B = {x, y, z}. Consider the subset R = {(1, x), (1, y), (2, z), (3, x)}
of A × B. Is R, a relation from A to B? If yes, find domain and range of R. Draw arrow diagram of R.
Solution: Since every subset of A × B is a relation from A to B, therefore, R is 1 x
also a relation from A to B. 2 y
Domain of R = {1, 2, 3}; Range of R = {x, y, z} 3 z
4
4. TYPES OF RELATIONS :
(i) Void or Empty Relation : A relation R on a set A is called void or empty relation, if no
element of set A is related to any element of set A.
Example : The relation R on the set A = {1, 2, 3, 4, 5} defined by R = {(a, b) : a – b = 12}.
We observed that a – b  12 for any two elements of A.
 (a, b)  R for any a, b A.
 R does not contain any element of A × A.
 R is empty set.
   R is the void or empty relation on A.
E 1

(ii) Universal Relation : A relation R on a set A is called universal relation, if each element
of set A is related to every element of set A.
Example : The relation R on the set = {1, 2, 3, 4, 5, 6} defined by T = {(a, b)  R : |a – b|  0}.
We observe that
|a – b|  0 for all a, b  A
 (a, b)  R for all (a, b)  A × A
 Each element of set A is related to every element of set A
 R = A × A
 R is universal relation on set A.
(iii) Trivial Relation : Trivial Relation means either each element of set A is related to every
element of set A or no element of set A is related to any element of set A.
Example : Both empty relation or universal relation are trivial relation.
(iv) Identity Relation : A relation IA on A is called the identity relation if every element of A is
related to itself only.
Example : If A = {1, 2, 3}, then the relation IA = {(1, 1), (2, 2), (3, 3)} is the identity
relation on set A. But, relation R1 = {(1, 1), (2, 2)} and R2 = {(1, 1), (2, 2), (3, 3), (1, 3)} are
not identity relations on set A, because (3, 3) R1 and in R2 element 1 is related to
elements 1 and 3.
(v) Reflexive Relation : A relation R on a set A is said to be reflexive if every element of A is
atleast related to itself.
Thus, R is reflective  (a, a)  R a A.
Example : In a set A = {1, 2, 3}, relation R1 = {(1, 1), (2, 2), (1, 2), (3, 3)} is
reflexive because every elements of set A is related to itself under R 1, while relation
R2 = {(1, 1), (2, 2), (1, 3)} is not reflexive because (3, 3)  R2.
Note : Total number of reflexive relation on set A is 2n(n –1), where n is number of elements in
set A.
(vi) Symmetric Relation : A relation R on a set A is said to be a symmetric relation
iff (a, b)  R  (b, a)  R for all a, b  A
i.e. a R b  b R a for all a, b  A.
Example : In a set A = {1, 2, 3}, relation R1 = {(1, 1), (2, 2), (1, 2), (2, 1)} is symmetric
because (a, b)  R1 (b, a) R1,  a, b  A, while relation R2 = {(1, 1), (3, 3), (1, 3)} is
not symmetric because (3, 1)  R2.
n(n 1)
Note : Total number of symmetric relation on set A is 2 2 , where n is number of
elements in set A.
(vii) Transitive Relation : Let A be any set. A relation R on set A is said to be a transitive
relation
iff (a, b)  R and (b, c)  R  (a, c)  R for all a, b, c  A
i.e. a R b and b R c  a R c for all a, b, c  A
Example : In a set A = {1, 2, 3}, relation R1 = {(1, 1), (2, 2), (1, 2), (2, 1)} is
transitive because (a, b)  R1 and (b, c)  R1 (a, c) R1,  a, b, c  A, while relation
R2 = {(1, 3), (3, 2)} is not transitive because (1, 2)  R2.
2 E

5. EQUIVALENCE RELATION :
A relation R on a set A is said to be an equivalence relation on A iff
(i) it is reflexive i.e. (a, a)  R for all a  A
(ii) it is symmetric i.e. (a, b)  R  (b, a)  R for all a, b  A
(iii) it is transitive i.e. (a, b)  R and (b, c)  R  (a, c)  R for all a, b, c  A.
Note : Empty relation defined on set A is not reflexive but symmetric and transitive while
universal relation define on set A is reflexive, symmetric and transitive.
Illustration 2: Let A be the set of all students of a boys school. Show that the relation R on A given by
R = {(a, b) : a is sister of b} is empty relation and R' = {(a, b) : the difference between the heights of a
and b is less than 5 meters} is the universal relation.
Solution : Since the school is boys school. Therefore, no student of the school can be sister of any
student of the school. Thus,
(a, b) R for any a, b  A.
Hence, R = i.e. R is the empty or void relation on A.
It is obvious that the difference between the heights of any two students of the school has to
be less than 5 meters.
  (a, b)  R for all a, b  A
  R' = A × A
  R' is the universal relation on set A.
Illustration 3: Relation R in the set N of natural numbers defined as R = {(x, y): y = x + 5 and x < 4}.
Determine whether relation are reflexive, symmetric and transitive :
Solution: R = {(x, y): y = x + 5 and x < 4}  R = {(1, 6), (2, 7), (3, 8)}
Reflexive: (1, 1)  R.
  R is not reflexive.
Symmetric : (1, 6) R. but (6, 1)  R. R is not symmetric.
Transitive : Since there is no three elements x, y, z  N such that (x, y)  R, (y, z)  R
but (x, z)  R
  R is transitive. Hence, R is neither reflexive, nor symmetric but it is transitive.
Illustration 4 : Let L be the set of all lines in XY-plane and R be the relation in L defined as
R = {(L1, L2) : L1 is parallel to L2}. Show that R is an equivalence relation. Find the set of all lines
related to the line y = 2x + 4.
Solution: Reflexive : Since every line  L is parallel to itself, therefore ( , ) R   L
 R is reflexive.
Symmetric : Let L1 , L2  L such that (L1, L2)  R  L1 || L2
 L2 || L1  (L2, L1)  R.
 R is symmetric.
Transitive : Let L1 , L2, L3  L such that (L1, L2)  R and (L2, L3)  R then
(L1, L2)  R  L1 || L2 …..(1)
and (L2, L3)  R  L2 || L3 …..(2)
From equation (1) and (2)
 L1 || L3  (L1, L3)  R
 R is transitive.
Hence, R is an equivalence relation.
Required set = { : is a line whose equation is y = 2x + k, k being any real number}.
E 3

Illustration 5: Show that the relation R in R defined as R = {(a, b) : a  b}, is reflexive and transitive
but not symmetric.
Solution: Reflexive: Given R = {(a, b): a  b}
Let a R such that if (a, a)  R  a  a, which is true, a R
 R is reflexive
Symmetric: Let a, b  R such that if (a, b)  R  a  b  b  a  (b, a) R
Example: 1  2  (1, 2)  R
But 2  1  (2, 1) R
 R is not symmetric
Transitive: Let a, b, c  R such that if (a, b)  R and (b, c)  R
 a  b and b  c  a  c (a, c)  R
 R is transitive
Hence R is reflexive and transitive but not symmetric.
Illustration 6: Let N denote the set of all natural numbers and R be the relation on N × N by
(a, b) R (c, d)  ad(b + c) = bc(a + d). Check whether R is an equivalence relation on N × N.
Solution: Reflexive : Let (a, b)  N × N then (a, b) R (a, b)
 ab(b + a) = ba(a + b) [by commutativity of addition and multiplication on N]
 (a , b) R (a, b)
Thus, (a, b) R (a, b) for all (a, b)  N × N. So R is reflexive on N × N.
Symmetric : Let (a, b), (c, d)  N × N. Then, (a, b) R (c, d).
 ad(b + c) = bc(a + d)
 cb(d + a) = da(c + b) [by commutativity of addition and multiplication on N]
 (c, d) R (a, b)
Thus, (a, b) R (c, d)  (c, d) R (a, b) for all (a, b), (c, d)  N × N
So, R is symmetric on N × N.
Transitive : Let (a, b), (c, d), (e, ƒ)  N × N Then,
bc ad 1 1 1 1
(a, b) R (c, d)  ad(b + c) = bc(a + d)       …….(1)
bc ad c b d a
de cf 1 1 1 1
and, (c, d) R (e, ƒ)  cf(d + e) = de(c + f)       …….(2)
de cf e d f c
Adding (1) and (2), we get
1 1  1 1   1 1  1 1
 c  b  e  d    d  a  f  c 
       
1 1 1 1 be af
       af(b + e) = be (a + f)  (a, b) R (e, f)
b e a f be af
So, R is transitive on N × N.
Hence; R being reflexive, symmetric and transitive; is an equivalence relation on N × N.

4 E

6. EQUIVALENCE CLASS :
Let X be a nonempty set and R be an equivalence relation on X. Let a be an element of X. The
set of all elements of X which are in relation R and related to a is denoted by [a] and is called an
equivalence class.
Thus, [a] = {x : X  A, x R a i.e., (x, a)  R}
Given an arbitrary equivalence relation R in an arbitrary set X, R divides X into mutually disjoint
subsets Ai called partitions or subdivisions of X satisfying :
(i) all elements of Ai are related to each other, for all i.
(ii) no element of Ai is related to any element of Aj, i  j.
(iii) UA j = X and Ai  Aj = , i  j.

The subsets Ai are called equivalence classes.

Illustration 7: If the relation R in the set A = {x  Z : 0 < x < 15} given by R = {(a,b) : a, b  Z}, |a – b| is
multiple of 5} is an equivalence relation, then find the equivalence class [2].
Solution: Let x  A which is related to 2 by given relation.
 x R 2  |x – 2| is a multiple of 5
 |x – 2| = 0, 5, 10, 15
 x = 2, 7, 12
Hence, equivalence class of [2] = {2, 7, 12}.
Illustration 8: If A = {1, 2, 3, ......, 9} and R be the relation in A × A defined by (a, b) R (c, d)
if a + d = b + c for a, b, c, d  A is an equivalence relation, then find the equivalence class [(2,5)].
[Exemplar]
Solution: Given set A = {1, 2, 3, .......9}
Let (x, y)  A × A which is related to (2, 5) by given relation.
(x, y) R (2, 5)  x+5=y+2
 1 + 5 = 4 + 2, 2 + 5 = 5 + 2, 3 + 5 = 6 + 2, ..........
Hence equivalence class of [(2, 5)] = {(1, 4), (2, 5), (3,6), (4,7), (5, 8), (6, 9)}.

7. FUNCTION :
A relation R from set A to set B is called a function if each element of A is uniquely associated
with some element of B. It is denoted by the symbol :
f
f : A  B or A   B

which reads 'f' is a function from A to B ‘or’ f maps A to B,


If an element a  A is associated with an element b  B, then b is called ‘the f image of a’ or
‘image of a under f’ or ‘the value of the function f at a’. Also a is called the ‘pre-image of b’ or
‘argument of b under the function ƒ’. We write it as
b = f (a) or f : a  b or f : (a, b)
Thus a function ‘f’ from set A to set B is subset of A × B in which each a belonging to A appears
in one and only one ordered pair belonging to f.
Note : Total number of functions defined from set A to set B is nm, where n(A) = m and n(B) = n)

E 5

Representation of Function :
Ordered pair : Every function from A  B satisfies the following conditions :
(i) f  A × B
(ii)  a  A there exists b  B
(iii) (a, b)  f & (a, c)  f  b = c

8. DOMAIN, CO-DOMAIN & RANGE OF A FUNCTION :


Let f : A  B, then the set A is known as the domain of f and the set B is known as
co-domain of f. The set of f images of all the elements of A is known as the range of f.
Domain of f = {a a  A, (a, f (a))  f}
Range of f = {f (a) a  A, f (a)  B}
(a) If only the rule of function is given then the domain of the function is the set of those real
numbers, where function is defined.
(b) For a continuous function, the interval from minimum to maximum value of a function
gives the range
(c) It should be noted that range is a subset of co-domain.

Illustration 9: The domain of the function f : R  R defined by f(x) = x2  3x  2 is ? [Exemplar]


Solution: Here, x2 – 3x + 2  0  (x – 1)(x – 2)  0  x  1 or x  2
Hence, the domain of f = (–, 1]  [2, )
1
Illustration 10 : Let f : R  R be the function defined by f(x) = ; x  R, then find the range of f.
2  cos x
[Exemplar]
Solution: We know that range of cos x is :
–1  cos x  1  1  – cos x  –1  3  2 – cos x  1
1 1 1
   1   f(x)  1
3 2  cos x 3
Hence, range of f(x) = [1/3, 1]

9. CLASSIFICATION OF FUNCTIONS :
One-One Function (Injective mapping) :
A function f : A  B is said to be a one-one function or injective mapping if different elements
of A have different f images in B.
Thus there exists x1, x2  A & f(x1), f(x2)B , f(x1) = f(x2)  x1 = x2 or x1  x2  f(x1)  f(x2).
Diagramatically, an injective mapping can be shown as:
A B A B

or

n!
Note : Total number of one-one functions defined from set A to set B is  n Pm , where
(n  m)!
n(A) = m and n(B) = n (m  n))

6 E

Many-one function :
A function f : A  B is said to be a many one function if two or more elements of A have
the same f image in B.
Thus f : A  B is many one if there exists x1, x2  A such that f (x1) = f (x2) but x1  x2.
Diagramatically, many-one function can be shown as:

or

Note :
(i) If a line parallel to x-axis cuts the graph of the function at atmost one point, then the
function is one-one.
(ii) If any line parallel to x-axis cuts the graph of the function at atleast two points, then f is
many-one.
(iii) If continuous function f (x) is always increasing or decreasing in whole domain, then f(x) is
one-one.
(iv) All linear functions are one-one.

Onto function (Surjective mapping) :


If the function f : A  B is such that each element in B (co-domain) is the f image of atleast
one element in A, then we say that f is a function of A 'onto' B. Thus f : A  B is surjective
iff  b  B,  some a  A such that f (a) = b.
Diagramatically, surjective mapping can be shown as:

A B

or

Note that : if range is same as co-domain, then f (x) is onto.

Into function :
If f : A  B is such that there exists atleast one element in co-domain which is not the image of
any element in domain, then ƒ (x) is into.
Diagramatically, into function can be shown as:
A B A B

or

Note :
(i) A polynomial of degree even defined from R  R will always be into
(ii) A polynomial of degree odd defined from R  R will always be onto.
(iii) A one-one function (injective) and onto function (surjective) is known as bijective function.
(Also known as Bijective mapping, Bi-uniform or non-singular)

E 7

Illustration 11: Let A  R – 3
2
and B  R –
3
2 
. Consider the function f : A B defined by

 3x  2 
f(x)    . Is f one-one and onto? Justify your answer.
 2x  3 
Solution: Let x1, x2  A such that f(x1) = f(x2)
3x1 – 2 3x 2  2
   (3x1 – 2) (2x2 – 3) = (3x2 – 2) (2x1 – 3)
2x1 – 3 2x 2 – 3
  x1x2 – 9x1 – 4x2 + 6 = 6x1x2 – 9x2 – 4x1 + 6  – 9x1 – 4x2 = – 9x2 – 4x1
  9x1 – 4x1 = 9x2 – 4x2  x1 = x2
  f is one – one.
Let y = f(x) y  B
3x  2 3y  2
  y xy – 3y = 3x – 2 xy – 3x = 3y – 2 x(2y – 3) = 3y – 2  x 
2x  3 2y  3

Now, 2y – 3  0  y 
3
2
 y  R 
3
2
,xA 
i.e. every element of B has pre-image in A
Range = codomain f is onto.
Hence, function f is one-one and onto.

Illustration 12: Show that the function f : R R given by f(x) = x3 + x is a bijection.
Solution: Injectivity : Let x1, x2  R such that f(x1) = f(x2)
 x13  x1  x 32  x 2  x13  x 32  (x1  x 2 )  0
 (x1  x 2 )(x12  x1x 2  x 22  1)  0

 x1 – x2 = 0  x12  x1x2  x22  0 for all x1 , x2  R  x12  x1x2  x22  1  for all x1, x2  R 
 x1 – x2 = 0
Thus, f(x1) = f(x2)
 x1 = x2 for all x1, x2  R
So, f is an injective map
Surjectivity : Let y be an arbitrary element of R. Then,
f(x) = y x3 + x = y x3 + x – y = 0
We know that an odd degree equation has at least one real root. Therefore, for every real
value of y, the equation x3 + x – y = 0 has a real root  such that
Thus, for every y  R there exists   R.
So, every element codomain has preimage in domain.
 Range = Codomain
Therefore f is surjective function. Hence, f : R  R is a bijection.

8 E

SOME IMPORTANT POINTS FOR OBJECTIVE QUESTIONS
1. Total number of relations defined from set A to set B, if n(A) = m and n(B) = n, is 2mn

2. Total number of non-empty relations defined from set A to set B, if n(A) = m and n(B) = n, is 2mn – 1

3. Total number of reflexive relations on set A is 2n(n-1)

(Where n is number of elements in set A)

4. Total number of symmetric relations on set A is 2((n(n1))/2)

(Where n is number of elements in set A)

5. Total number of functions defined from set A to set B is nm.

(Where n(A) = m and n(B) = n)

n!
6. Total number of one-one functions defined from set A to set B is  n Pm
(n  m)!

(Where n(A) = m and n(B) = n (m  n))

7. If n(A) = n(B) i.e. m = n; then total number of one-one functions from set A to set B is n!

8. If n(A) = m, n(B) = n and m n; then total number of onto functions from set A to set
n
  1
nr n
Cr r m , if m  n
B =  r 1
 0 , if m  n

n! , if m  n
9. Total number of bijective functions defined from set A to set B = 
 0 , if m  n

10. For deciding the nature of function in case of finite set A and set B, use the following conditions
directly:

(i) If f is an injection, then n(A) ≤ n (B)

(ii) If f is a surjection, then n(A) ≥ n (B)

(iii) If f is a bijection, then n(A) = n (B)

11. A polynomial of degree odd defined from R  R will always be onto.

12. A polynomial of degree even defined from R  R will always be into.

E 9

EXERCISE-I
1. Consider the relation R = {(a, b) (a, c), (a, a), (c, c)} on the set A = {a, b, c, d}. Minimum
number of elements of A × A which must be adjoined to R in order to make R an equivalence
relation is:

(A) 4 (B) 5 (C) 6 (D) 7

2. If A and B are two sets having 10 and 15 elements respectively and 8 elements are common in A
and B; then number of relations which can be defined from A to B are:

(A) 217 (B) 225 (C) 2150 (D) 280

3. Let R be a relation on natural number N defined as aRb  a is a factor of b; then relation R is:

(A) Reflexive and symmetric (B) Reflexive and transitive

(C) Symmetric and transitive (D) Only reflexive

4. The number of reflexive relations defined on the set S = {1, 2, 3, 4, 5} must be:

(A) 25 (B) 225 (C) 52 (D) 220

5. The number of symmetric relation defined on the set A = {1, 2, 3, 4}.

(A) 25 (B) 26 (C) 28 (D) 210

6. The maximum number of equivalence relation on the set A = {1, 2, 3} are : [Exemplar]

(A) 1 (B) 2 (C) 3 (D) 5


7. Let us define a relation R in R as aRb if a  b. Then, R is
(A) an equivalence relation (B) reflexive, transitive but not symmetric
(C) symmetric, transitive but not reflexive (D) neither transitive nor reflexive but symmetric
2
8. Let f : R+  [5, ), f(x) = 4x + 12x + 5, where R+ is non-negative real number. Then, f is ?
(A) injective (B) surjective (C) bijective (D) None of these
9. Let A = {2, 3, 4, 5 ........ 17}. Let R be the equivalence relation on A × A, cartesian product of A
with itself, defined by (a, b) R (c, d) iff ad = bc. Then, the number of ordered pairs of the
equivalence class of (3, 2) is :
(A) 6 (B) 5 (C) 7 (D) 4
10. Which of the following functions from Z into Z are bijections?
(A) f(x) = x2 (B) f(x) = x + 2 (C) f(x) = 2x + 1 (D) f(x) = x2 + 1
11. Let A = {1, 2, 3} and R = {(1, 2), (2, 3), (1, 3)} be a relation on A. Then, R is
(A) neither reflexive nor transitive (B) neither symmetric nor transitive
(C) transitive (D) None of these

10 E

x2
12. If the function f : R A given by f(x)  is a surjection, then A =
x2  1
(A) R (B) [0, 1] (C) (0, 1] (D) [0, 1)
x
 , if x is even
13. Let f : Z  Z be given by f(x)   2 . Then f is
 0 , if x is odd

(A) onto but not one-one (B) one-one but not onto
(C) one-one and onto (D) neither one-one nor onto

ASSERTION-REASON BASED QUESTIONS


In the following questions, a statement of assertion (A) is followed by a statement of Reason (R).
Choose the correct answer out of the following choices.
(A) Both A and R are true and R is the correct explanation of A.
(B) Both A and R are true but R is not the correct explanation of A.
(C) A is true but R is false.
(D) A is false but R is true.
14. Let R be the relation in the set of integer Z given by R = {(a, b) : 2 divides a – b}
Assertion (A) : R is reflexive relation.
Reason (R) : A relation is said to be reflexive if x R x,  x  Z.
15. Assertion (A) : The function f : R R, f(x) = x2 + bx + c, where b and c are real constants,
describes many-one onto mapping.
Reason (R) : Let A = {1, 2, 3,……., n} and B = {a, b}. Then the number of surjections from
A to B is 2n – 2.

ANSWER KEY
Q. No. 1 2 3 4 5 6 7 8 9 10
Ans. C C B D C D B C B B
Q. No. 11 12 13 14 15
Ans. C D A A D
E 11

SOLUTIONS
1. (C) (b, b), (b, a), (c, a), (b, c), (c, b) and (d, d) must be included to make R an equivalence
relation.
 Minimum number of elements of A × A which must be adjoined to R to make it an
equivalence relation is 6.
2. (C) Total number of relation defined from A to B will be 2n(A) × n(B) = 210×15 = 2150
3. (B) Given, aRb  a is a factor of b
a is a factor of a  a R a  a N  R is reflexive.
Now; 2 is a factor of 4 but 4 is not a factor of 2
 R is not symmetric
Let aRb and bRc  b = ma, c = nb
 c = n (ma)  c = (mn) a  aRc
 R is transitive
4. (D) Total number of reflexive relations are = 2n(n–1) = 25(5–1)= 220 ( n = 5)
n  n 1 4  4 1

5. (D) Number of symmetric relation  2 2


2 2
 2 20 /2 = 210 ( n = 4)
6. (D) R1  (1,1),(2, 2),(3,3)
R 2  (1,1),(2, 2),(3,3),(1, 2),(2,1)
R 3  (1,1),(2, 2),(3,3),(2,3),(3, 2)
R 4  (1,1),(2, 2),(3,3),(1,3),(3,1)
R 5  (1,1),(2, 2),(3,3),(1, 2),(2,1),(2,3),(3, 2),(1,3),(3,1)
Hence, total no. of equivalence relation = 5
7. (B) Give that , aRb if a  b
 aRa a  a which is true
Let aRb, a  b, then b  a which is not true R is not symmetric.
But aRb and bRc
 a  b and b  c
 ac
Hence, R is transitive
8. (C) One-one : Let x1, x2  R+ such that
if f(x1) = f(x2)  4x12  12x1  5  4x 22  12x 2  5
 4(x12  x 22 )  12(x1  x 2 )  0
 (x1 – x2) [4(x1 + x2) + 12] = 0
 x1 – x2 = 0 [4(x1 + x2) + 12 0, x1, x2  R+]
 x1 = x2
 f is one-one

12 E

Onto : Let y = f(x) = 4x2 + 12x + 5
 4x2 + 12x + (5 – y) = 0
12  144  4  4(5  y)
 x=
8
3  4  y
 x ...(i) ( x  R+)
2
3  4  y
  0  4  y  3 4 + y  9 y 5
2
 y [5, )
Range of f(x) = co-domain of f(x).
 f is onto.
9. (B) Given; (a, b) R (c, d) iff ad = bc where 'R' is an equivalence relation on A × A
Now, for equivalence class of (3, 2);
Let (3, 2) R (x, y)
2x
3y = 2x or y =
3
Hence; [(3, 2)] = {(3, 2), (6, 4), (9, 6), (12, 8), (15, 10)}
The number of ordered pairs are 5.
10. (B) Here, f(x) = x + 2 f(x1) = f(x2)
x1 + 2 = x2 + 2 x1 = x2
Let y = x + 2
x = y – 2  Z, y  Z
Hence, f(x) is one-one and onto.
11. (C) R = {(1, 2) (2, 3), (1, 3)}
(i) (1, 1) R not reflexive.
(ii) (1, 2) R but (2, 1) R not symmetric.
(iii) (1, 2)  R and (2, 3)  R (1, 3) R but in case of (2, 3) and (1, 3) there is no three
elements x, y, z N such that (x, y)  R, (y, z)  R but (x, z) R. So, R is transitive.
Hence, R is neither reflexive nor symmetric, but it is transitive.
12. (D) f is surjective. Range = Co-domain = A
x2
Let y = f(x)  y  x2y + y = x2x2(y – 1) = – y
x 1
2

y y
  x2   x   R 
1 y 1 y
y y
  if  0   0  + – +

1 y y 1
0 1
  y  [0, 1)
E 13

0
13. (A) f(0) =  0 and f(1) = 0
2
Here, two elements have same image so f is not one-one.
Now, Let x = 2n (given) n  Z
2n
f(2n)   n
2
Range = Co-domain f is onto.
14. (A) Assertion : R = {(a, b) : 2 divides a – b}  a, b  Z
Let a  Z such that (a, a) R
 a – a = 0 which is divided by 2.
Hence, R is reflexive.
Assertion is true.
Reason : Reason is true as it is a definition of reflexive relation.
Hence, both A and R are true and R is correct explanation of A.
15. (D) Assertion : Given function is f : R  R; f(x) = x2 + bx + c
It is a quadratic equation in x.
f(x) is an even polynomial.
Hence, it is a many-one mapping and not onto mapping.
 Assertion is false.

Reason : Total number of functions =  n(B) 


n(A)
 2n
Clearly, a function will not be onto if all elements of A map to either a or b.
In the case range co-domain
Function is not onto.
 Total no. of surjective functions  2n  2 .
 Reason is true.
Hence, A is false, R is true.

14 E

EXERCISE-II
1. Show that the number of equivalence relations on the set {1, 2, 3} containing (1, 2) and (2, 1) is
two.
2. Let R = {(1,3), (4,2), (2,4), (2,3), (3,1)} be a relation on the set A = {1, 2, 3, 4}. Check whether R
is reflexive, symmetric or transitive.
3. Let the function f : R  R be defined by f(x) = cos x, x  R. Show that f is neither one-one nor
onto. [Exemplar]
4. Let R be the equivalence relation in the set A = {0, 1, 2, 3, 4, 5} given by R = {(a, b) : 2 divides
(a – b)}. Write the equivalence class [0]. [Exemplar]
5. Let A = {x  R : –1  x  1} = B. Show that f : A  B given by f(x) = x |x| is a bijection.
6. Let R be relation on the set of natural number N as follows, R = {(x, y) : x  N, 2x + y = 41}.
Find the domain and range of the relation R. Also verify whether R is reflexive, symmetric and
transitive. [Exemplar]
7. Let N be set of all natural number and let R be a relation on N × N, defined by
(a, b) R (c, d) a + d = b + c for all (a, b), (c, d)  N × N. Show that R in equivalence relation
on N × N.
3x  2
8. If f : R – {1}  R – {3} is defined by f(x)  , show f is bijective.
x 1
9. Let f : X Y be a function. Define a relation R on X given by R = {(a, b) : f(a) = f(b)}. Show
that R is an equivalence relation on X.

10. Consider f: R+  [–5, ) given by f (x) = 9x2 + 6x – 5. Show that f is bijective.

CASE STUDY
11. Herry and Messi are playing Ludo at have during covid-19. While rolling the dice, Herry's
sister Raji observed and noted the possible outcomes of the thrown every time belongs to set
{1, 2, 3, 4, 5, 6}. Let A be the set of players while B be set of all possible outcomes.
A = {H, M}, B= {1, 2, 3, 4, 5, 6}
Based on the above information, answer the following questions:
(i) Let R be the relation defined on set B such that R = {(x, y) : y is divisible by x}. Write R is
roster form and hence check whether R is reflexive, symmetric or transitive.
(ii) Raji wants to know that how many functions and relation are possible from set A to set B.

E 15

SOLUTIONS
1. Required relation are
R1 = {(1, 1),(2, 2),(3, 3),(1, 2),(2, 1)}
R2 = {(1, 1),(2, 2),(3, 3),(1, 2),(2, 1),(1, 3),(3, 1),(2, 3),(3, 2)}.

2. Since (1, 1)  R, R is not reflexive


Since (2, 3)  R but (3, 2)  R
 R is not symmetric
Since (1, 3)  R and (3, 1)  R but (1, 1)  R
 R is not transitive
3. Given function, f(x) = cos x,  x  R

           
Now, f    cos  0  f    cos  0  f    f  
2 2  2  2 2  2 
 
But 
2 2
So, f(x) is not one-one.
Now, f(x) = cos x, x  R is not onto as there is no pre-image for any real number. Which does
not belonging to the intervals [–1, 1], the range of cos x.
4. Let x  A which is related to '0' by the given relation.
 xR0  2 divides (x – 0)  [0] = {0, 2, 4}
5. We observe the following properties of f.
Infectivity : Let x1, x2 be any two elements in A. Then,
x1  x2  x1 |x1|  x2| x2| f(x1)  f(x2)
so, f : A  B is an injective map.

 x2 , if x  0
Surjectivity : We have f(x)  x | x |  2
x , if x  0
If 0  x  1, then f(x) = x2 takes all values between 0 and 1 including these two points.
Also, if –1  x < 0, then f(x) = – x2 takes all values between – 1 and 0 including –1. Therefore,
f(x) takes every value between –1 and 1 including –1 and 1. So, range of f is same as its
co-domain.
Hence, f : A  B is an onto function.
Thus, f : A  B is both one-one and onto.
Hence, it is a bijection.
16 E

6. We have R = {(x, y) : x  N, y  N, 2x + y = 41}
Domain = {1, 2, 3, ………. 20} [ y  N]
 R = {(1, 39), (2, 37), (3, 35), ………….(19, 3), (20, 1)}
 Range = {1, 3, 5, …..39}
R is not reflexive as (2, 2) R as 2 × 2 + 2  41
Also R is not symmetric as (1, 39)  R but (39, 1) R
Further R is not transitive as (11, 19)R and (19, 3)  R but (11, 3) R
Hence, R is neither reflexive, nor symmetric and not transitive.
7. We observe the following properties of reaction R.
Reflexivity : Let (a, b)  N × N. Then
(a, b) R (a, b) a + b = b + a [by commutativity of addition and multiplication on N]
So, R is reflexive on N × N .
Symmetry : Let (a, b), (c, d)  N × N be such that (a, b) R (c, d). Then
 a + d = b + c  c + b = b + a [by commutativity of addition and multiplication on N]
 (c, d) R (a, b)
Thus, (a, b) R (c, d) (c, d) R (a, b) for all (a, b), (c, d)  N × N
So, R is symmetric on N × N.
8. Let f(x1) = f(x2),  x1, x2  R – {1}
3x1  2 3x 2  2
 
x1  1 x2  1
 (3x1 – 2) (x2 – 1) = (3x2 – 2) (x1 – 1)
 3x1x2 – 3x1 – 2x2 + 2 = 3x1x2 – 3x2 – 2x1 + 2
 –3x1 + 2x2 = –3x2 + 2x1
 x1 = x2
 f is injective
To show that f is surjective
Let y  R – {3} i.e. y  3 and f(x) = y
3x  2
 y 3x – 2 = xy – y 3x – xy = 2 – y
x 1
2y
 x(3 – y) = 2 – y  x  y  R – {3}, x R – {1}
3y
Every element of co-domain has pre-image in domain.
Range = Co-domain
f is surjective
Hence, f is a bijective function.
E 17

9. We observe the following properties of relation R.
Reflexivity : For any a  X, we have
f(a) = f(a) (a, a)  R R is reflexive.
Symmetry : Let a, b  X be such that (a, b)  R. Then,
(a, b)  R f(a) = f(b) f(b) = f(a) (b, a)  R.
So, R is symmetric.
Transitivity : Let a, b, c X be such that (a, b)  R and (b, c)  R. Then,
(a, b) R and (b, c)  R
 f(a) = f(b) and f(b) = f(c)
 f(a) = f(c)
 (a, c)  R
So, R is transitive.
Hence, R is an equivalence relation.
10. One-one : Let x1, x2  Rt such that
if f(x1) = f(x2)  9x12  6x1  5  9x 22  6x 2  5
 9(x12  x 22 )  6(x1  x 2 )  0 (x1 – x2) [9(x1 + x2) + 6] = 0
 x1 – x2 = 0 [9(x1 + x2) + 6 0]  x1 = x2
 f is one-one
Onto : Let y = f(x) = 9x2 + 6x – 5
 9x2 + 6x – (5 + y) = 0
6  36  36(5  y) 1  y  6
 x=  x ...(i) ( x  R+)
18 3
1  y  6
  0  y  6  1 y  –5 y  [–5, )
3
Range = Co-domain
 f is onto
Hence, f is bijective.
11. (i) Given that B = {1, 2, 3, 4, 5, 6} and R = {(x, 4) : y is divisible by x}then, roster form of R is
R = {(1, 1), (2, 2), (3, 3), (4, 4), (5, 5), (6, 6), (1, 2), (1, 3), (1, 4), (1, 5), (1, 6), (2, 4), (2, 6), (3, 6)}
Now, R is reflexive as (a, a)  R,  a  B
R is not symmetric as (1, 2)  R but (2, 1) R
pair for which (a, b)  R and (b, c)R  (a, c)  R  , a, b, c  B.
(ii) n(A) = 2 and n(B) = 6
Total no. of functions defined from set A to set B is 62 and
Total no. of relations defined from set A to set B is 22 × 6 = 212.

18 E

NCERT IMPORTANT QUESTIONS
1. Show that the relation R in the set R of real numbers, defined as R = {(a, b ) : a  b2} is neither
reflexive nor symmetric nor transitive. [Ex.1.1, Q.2]
2. Show that the relation R in R defined as R = {(a, b): a  b}, is reflexive and transitive but not
symmetric. [Ex.1.1, Q.4]
3
3. Check whether the relation R in R defined as R = {(a, b): a  b } is reflexive, symmetric or
transitive. [Ex.1.1, Q.5]

4. Show that each of the relation R in the set A = {x  Z : 0  x  12}, given by


(i) R = {(a, b) : |a – b| is a multiple of 4}
(ii) R = {(a, b) : a = b}
is an equivalence relation. Find the set of all elements related to 1 in each case. [Ex.1.1, Q.9]
5. Let L be the set of all lines in XY plane and R be the relation in L defined as R = {(L1, L2) : L1 is
parallel to L2}. Show that R is an equivalence relation. Find the set of all lines related to the line
y = 2x + 4. [Ex.1.1, Q.14]
 n 1
 , if n is odd
6. Let f : N  N be defined by f (n) =  2 ; for all n  N.
 n , if n is even
2
State whether the function f is bijective. Justify your answer. [Ex.1.2, Q.9]
x–2
7. Let A = R - {3} and B = R – {1}. Consider the function f: A  B defined by f(x) =  .
 x–3
Is f one-one and onto? Justify your answer. [Ex.1.2, Q.10]
x
8. Show that the function f: R  {x  R : –1 < x < 1} defined by f (x)  , x  R is one-one
1 | x |
and onto function. [Misc. Ex., Q.1]

ANSWER KEY
3. R is neither reflexive nor symmetric nor transitive.
7. Yes

E 19

PREVIOUS YEAR QUESTIONS

1 MARK QUESTIONS

1. Let set X = {1, 2, 3} and a relation R is defined in X as : R = {(1, 3), (2, 2), (3,2)}, then minimum
ordered pairs which should be added in relation R to make it reflexive and symmetric are :
(A) {(1, 1), (2, 3), (1, 2)} (B) {(3, 3), (3, 1), (1, 2)}
(C) {(1, 1), (3, 3), (3, 1), (2, 3)} (D) {(1, 1), (3, 3), (3, 1), (1, 2)}
Sol. (C) Given ; R = {(1,3), (2, 2), (3, 2)} on set X = {1, 2, 3}
Then ; if R is reflexive and symmetric, then R = {(1, 1), (2, 2), (3, 3), (1, 3), (3, 2), (3, 1), (2, 3)}
Hence ; ordered pairs to be added are : {(1, 1), (3, 3), (3, 1), (2, 3)} [1]
2. If R = {(x, y) ; x, y Z, x2 + y2  4} is a relation in set Z, then domain of R is : [CBSE 2022]
(A) {0, 1, 2} (B) {–2, –1, 0, 1, 2} (C) {0, –1, –2} (D) {–1, 0, 1}
Sol. (B) Given ; R = {(x, y) : x, y  Z, x2 + y2  4} is a relation in set Z.
Hence ; according to the given relation,
Domain of R = {–2, –1, 0, 1, 2} [1]
3. Let X = {x2 : x N} and the function f : N  X is defined by f(x) = x2, x N. Then, this
function is : [CBSE 2022]
(A) injective only (B) not bijective (C) surjective only (D) bijective
Sol. (D) f(x) = x2 ; x N ; f : N  X
where X = {x2 : x  N}
Now ; f(x1) = f(x2)

 x12  x 22

 x1 + x2) x1 – x2) = 0


Now : x1 + x2  0 ( x1, x2  N)
 x1 – x2  0 x1 = x2
Hence ; f is one-one.

Let y = f(x) y = x2  x  y

y  X, there exists x  N


Hence, every element of co-domain has pre-images in domain.
 Range = Co-domain f is onto.
 The given function is bijective. [1]
20 E

4. A function f : R  R defined by f(x) = 2 + x2 is : [CBSE 2022]
(A) not one-one (B) one-one
(C) not onto (D) neither one-one nor onto
Sol. (D) f(x) = 2 + x2 ; f : R  R
Now ; f(x1) = f(x2)
 x12  2  x 22  2 x1 = ±x2 f is not one-one

Also ; f is not onto as there is no pre-image for negative real numbers.


Hence ; f is neither one-one nor onto. [1]
5. Write the smallest reflexive relation on set A = {a, b, c}. [CBSE 2021 C]
Sol. For reflexive relation, each and every element of the given set must be related to itself, atleast i.e.
the smallest reflexive relation on set A will be {(a, a), (b, b), (c, c)}. [1]
6. If f = {(1, 2), (2, 4), (3, 1), (4, k)} is a one-one function from set A to A, where A = {1, 2, 3, 4};
then find the value of k. [CBSE 2021 C]
Sol. In a one-one function, different elements must have different images. So, according to the given
function, f = {(1, 2), (2, 4), (3, 1), (4, k)}, the value of k must be 3. [1]
7. (a) Check whether the relation R defined on the set {1, 2, 3, 4} as R = {(a, b) ; b = a + 1} is
transitive. Justify your answer. [CBSE 2021 C]
OR
(b) If the relation R on the set A = {x : 0  x  12} given by R = {(a, b) : a = b} is an
equivalence relation, then find the set of all elements related to 1.
Sol. (a) Given, R = {(a, b) : b = a + 1} defined on the set A = {1, 2, 3, 4}
 R = {(1, 2), (2, 3), (3, 4)}
By definition, for transitive relation, if a R b and b R c  a R c  a, b, c  A
For 1, 2, 3 A ; (1, 2) R and (2, 3) R but (1, 3) R
Therefore, R is not transitive. [1]
Aliter:
Given; R = {(a, b) : b = a + 1} defined on the set A = {1, 2, 3, 4}
For transitive relation;
Let (a, b)  R and (b, c)  R  a, b, c  A
 b = a + 1 and c = b + 1  c = a + 2
or (a, c)  R
 R is not transitive [1]

E 21

OR
(b) The elements in R that are related to 1 will be those elements from set A which are equal to 1.
Hence, the set of elements related to 1 is {1}. [1]
Aliter:
Let x  A which is related to 1 by the given relation R = {(a, b) : a = b}
 x R 1  (x, 1)  R  x = 1
 [1] = {1} [1]
8. The relation R in the set {1, 2, 3} given by R = {(1, 2), (2, 1), (1, 1)} is : [CBSE 2020]
(A) symmetric and transitive, but not reflexive (B) reflexive and symmetric, but not transitive
(C) symmetric, but neither reflexive nor transitive (D) an equivalence relation
Sol. (C) Let A = {1, 2, 3} and R = {(1, 2), (2, 1), (1, 1)}
R is not reflexive as (2, 2) & (3, 3) R.
R is symmetric relation as (a, b) R & (b, a) R a, b A.
R is not transitive relation because (2, 1) R & (1, 2) R but (2, 2) R.
Hence; R is symmetric, but neither reflexive nor transitive. [1]
9. Let A = {1, 3, 5}. Then the number of equivalence relations in A containing (1, 3) is : [CBSE 2020]
(A) 1 (B) 2 (C) 3 (D) 4
Sol. (B) R1 = {(1, 1), (3, 3), (5, 5), (1, 3), (3, 1)}
and R2 = {(1, 1), (3, 3), (5, 5), (1, 3), (3, 1), (1, 5), (5, 1), (3, 5), (5, 3)}
Hence, the number of equivalence relations in A containing (1, 3) is 2. [1]

4 MARKS QUESTIONS
10. Check whether the relation R in the set N of natural numbers given by R = {(a, b) : a is divisor of b}
is reflexive, symmetric or transitive. Also, determine whether R is an equivalence relation.
[CBSE 2020]
Sol. The relation R in the set N is given by:
R = {(a, b) : a is divisor of b }
For reflexive relation ; let (a, a)  R  a  N
 a is divisor of a,  a  N, which is true
 (a, a) R
So, R is reflexive relation [1]
For symmetric relation; let (a, b)  R  a, b  N
 a is divisor of b
But it is not necessary that b is also divisor of a.
So; (b, a)  R
 R is not symmetric relation. [1]
For transitive relation; let (a, b)  R and (b, c)  R  a, b, c  N
 a is divisor of b and b is divisor of c
22 E

Then, we can say that a is also divisor of c
i.e. (a, c) R
So, R is transitive relation [1]
 R is reflexive and transitive but not symmetric
So, R is not an equivalence relation on N. [1]
11. Check whether the relation R defined on the set A = {1, 2, 3, 4, 5, 6} as R = {(a, b) : b = a + 1} is
reflexive, symmetric or transitive. [CBSE 2019]
Sol. Given, R = {(a, b) : b = a + 1} is defined on the set A = {1, 2, 3, 4, 5, 6}
 R = {(1, 2), (2, 3), (3, 4), (4, 5), (5, 6)}
Reflexive : By definition, a R a  a  A
Now, 1  A but (1, 1)  R.
Therefore, R is not reflexive. [1]
Symmetric : By definition, if a R b  b R a  a, b  A
For 1, 2 A, It can be observed that (1, 2) R but (2, 1)  R.
Therefore, R is not symmetric. [1]
Transitive : By definition, if a R b and b R c  a R c  a, b, c  A
For 1, 2, 3  A, (1, 2) R and (2, 3) R but (1, 3)  R
Therefore, R is not transitive. [1]
Hence, R is neither reflexive nor symmetric nor transitive. [1]
Aliter:
Given; R = {(a, b) : b = a + 1}defined on the set A = {1, 2, 3, 4, 5, 6}
For reflexive relation;
(a, a)  R  a A
 a = a + 1; which is not true
 R is not reflexive [1]
For symmetric relation;
Let (a, b)R  a, b A
 b=a+1
 (b, a)  R [ a  b + 1]

 R is not symmetric [1]


For transitive relation;
Let (a, b)R and (b, c) R  a, b, c A
 b = a + 1 and c = b + 1  c = a + 2
or (a, c)  R
 R is not transitive [1]
Hence; R is neither reflexive nor symmetric nor transitive. [1]

E 23

6 MARKS QUESTIONS

12. Let A = {x  Z : 0  x  12}. Show that R = {(a, b) : a, b  A, |a – b| is divisible by 4} is an


equivalence relation. Find the set of all elements related to 1. Also; write the equivalence
class [2]. [CBSE 2018]
OR
x
Show that the function f : R  R defined by f(x)  2
,  x  R is neither one-one nor onto.
x 1
Sol. Given, R = {(a, b) : a, b  A, |a – b| is divisible by 4}
(i) For Reflexive relation:
Let a  A
Now, |a – a| = 0, which is divisible by 4
So, (a, a)  R  a  A
Hence, R is reflexive. [1]
(ii) For Symmetric relation:
Let a, b  A such that (a, b)  R
i.e. |a – b| is divisible by 4.
 |–(b – a)| = |b – a| is also divisible by 4.
Hence; (b, a) R.
So, R is symmetric. [1]
(iii) For Transitive relation:
Let a, b, c  A such that (a, b), (b, c)  R
i.e. |a – b| & |b – c| is divisible by 4.
Let |a – b| = 4k1
& |b – c| = 4k2
 (a – b) = ±4k1 .......(1)
& (b – c) = ±4k2 .......(2)
Adding equations (1) & (2);
 (a – b) + (b – c) = ± 4k1 ± 4k2 = ± 4(k1 + k2)

 a – c is divisible by 4.
 |a – c| is divisible by 4.
Hence; (a, c)  R
So, R is transitive. [1]
Hence; R is an equivalence relation. [1]
24 E

Further, let (x, 1)  R  x  A
 |x – 1| is divisible by 4
 x – 1 = 0, 4, 8, 12
 x = 1, 5, 9 [x = 13 A]

 Equivalence class of [1] = {1, 5, 9} [1]


[ The set of all elements related to 1 represents its equivalence class]

Now, we will find equivalence class of [2]


Let (x, 2)  R  x  A
 |x – 2| = 0, 4, 8, 12
 x = 2, 6, 10 [x = 14 A]

 Equivalence class of [2] = {2, 6, 10}. [1]


OR
x
f : R  R ; f(x) 
x2  1
(1) For one-one function: Let x1 = 2  R ; x2 = 1/2  R
Then, f(x1) = f(2) = 2/5
and f(x2) = f(1/2) = 2/5
 x1  x2 but f(x1) = f(x2)
 f is not one-one function [2½]
(2) For onto function: Let y = f(x)
x
 y  2
x 1
 x2y – x + y = 0
1  1  4y2
 x  R if 1 – 4y2  0 and y  0 [1½]
2y

 4y2 – 1  0 or (2y + 1)(2y – 1)  0


+ – +
–1/2 1/2

 1   1
 x    , 0    0,  [½]
 2   2
Thus, every element in the Co-domain does not have its pre-image in domain.
Hence, f : R  R is not onto. [½]
 f is neither one-one nor onto. [1]

E 25

IMPORTANT NOTES
__________________________________________________________________________________
__________________________________________________________________________________
__________________________________________________________________________________
__________________________________________________________________________________
__________________________________________________________________________________
__________________________________________________________________________________
__________________________________________________________________________________
__________________________________________________________________________________
__________________________________________________________________________________
__________________________________________________________________________________
__________________________________________________________________________________
__________________________________________________________________________________
__________________________________________________________________________________
__________________________________________________________________________________
__________________________________________________________________________________
__________________________________________________________________________________
__________________________________________________________________________________
__________________________________________________________________________________
__________________________________________________________________________________
__________________________________________________________________________________
__________________________________________________________________________________
__________________________________________________________________________________
__________________________________________________________________________________
__________________________________________________________________________________
__________________________________________________________________________________
__________________________________________________________________________________
__________________________________________________________________________________
__________________________________________________________________________________
__________________________________________________________________________________

26 E


CHAPTER-2 : INVERSE TRIGONOMETRIC FUNCTIONS


1. DEFINITION :
(a) Inverse of a function : Let f : A B be a one-one & onto function, then there exists a
unique function g : B  A such that f(x) = y  g(y) = x, x  A & y B. Then g is said
to be inverse of f. Thus g = f–1 : B  = {(f(x), x))|(x, f (x)) f}
(b) Inverse trigonometric function : Inverse function which is related to trigonometric ratios
  
is called inverse trigonometric function. If sin = x  = sin–1x;    , 
 2 2
Note 1: (sin–1x)  (sin x)–1
Note 2: From the definition of the inverse function, it follows that sin (sin–1 x) = x if –1  x  1
 
and sin–1 (sin x) = x if   x  . In other words, if y = sin–1 x, then sin y = x
2 2
(I) (i) y = sin (sin–1x) = x ; x  [–1,1]
(ii) y = cos (cos–1 x) = x ; x  [–1,1]
(iii) y = tan (tan–1 x)=x ; xR
(iv) y = cot (cot–1 x) = x ; x R
(v) y = sec (sec–1 x) = x ; |x| > 1
(vi) y = cosec (cosec–1 x) = x ; |x| > 1
 15 
Illustration 1: Evaluate : sin  tan 1  .
 8
15 15 15
Solution: Let tan 1    tan    sin  
8 8 17
15 h=17
   sin 1 p=15
17

b=8
15 15
So, tan 1  sin 1
8 17
 15   15  15
Now, sin  tan 1   sin  sin 1  
 8  17  17
 1 4
Illustration 2: Evaluate : cos sin 1  sec1  .
 4 3
1 1 4 4
Solution: Let sin 1    sin   and sec 1    sec 1  
4 4 3 3
Now, cos ( + ) cos  cos– sin  sin

3 15  7
2 2
3 1 1 3
 1    1   
4 4 4 4 16

E 27

  
(II) (i) y = sin–1(sin x) = x ; x   , 
 2 2
(ii) y = cos–1(cos x) = x ; x  [0,]
  
(iii) y = tan–1(tan x) = x ; x   , 
 2 2
(iv) y = cot–1(cot x) = x ; x  (0, )
   
(v) y = sec–1(sec x) = x ; x  0,    , 
 2 2 
    
(vi) y = cosec–1(cosec x) = x ; x    , 0    0, 
 2   2
 7 
Illustration 3: Evaluate : sin 1  sin  .
6
 7  7
Solution: sin 1  sin   ,
 6  6
7  
because does not lie between  and .
6 2 2
 7       7 
Now, sin 1  sin   sin 1 sin        
 6    6  6 6
       
 sin 1   sin    sin 1  sin        ,  [sin() = –sin; sin–1(–) = –sin–1]
 6  6 6  2 2

2. DOMAIN, RANGE (PRINCIPAL VALUE BRANCH) :


Function Domain Range (Principal value branch)
1. sin–1 x [–1, 1] [– /2, /2]
2. cos–1 x [–1, 1] [0, ]
3. tan–1 x R (–/2, /2)
4. cot–1 x R (0, )

5. sec–1 x R – (–1, 1) [0, ] – {/2}

6. cosec–1 x R – (–1, 1) [–/2, /2] – {0}

Illustration 4: Find the domain of cos–1 (2x – 1).

Solution : We know that domain of cos–1(x) is [–1, 1]

So, the domain of cos–1 (2x –1) is the set of all values of x satisfying

–1  2x –1  1  0  2x  2

 0  x  1; Hence, domain is [0, 1]

28 E

Illustration 5 : Find the domain of f(x) = sin–1 (–x2).

Solution : f(x) = sin–1(–x2) is defined for all x satisfying


–1  – x2  1
 1  x2  –1  0  x2  1
 x2  1  x2 –1  0  (x –1) (x + 1)  0  –1  x  1
Hence, the domain of f(x) = sin–1(–x2) is [–1, 1]
 3 
Illustration 6 : Find the principal value of sec 1  2sin  .
 4 

 3   1 
Solution : sec 1  2sin  = sec 1  2
 4   2
 = sec 1  2  =

4
[0, ] –

2 
  1 
Illustration 7 : Write the value of sin   sin 1     .
3  2 

  1   
Solution : sin   sin 1     = sin  = sin
3  2  3 6   

2
=1

  15  
Illustration 8 : Write value of tan 1 tan   .
  4 

  15   1     1        
Solution : tan 1 tan    = tan tan  4     = tan tan     =     , 
  4    4    4  4  2 2
 
Illustration 9 : Write the value of sin 1  cos  .
 9

         7    
Solution : sin 1  cos  = sin 1 sin     =  =   ,
 9   2 9  2 9 18  2 2 

3. GRAPHS OF INVERSE TRIGONOMETRIC FUNCTIONS :


(a) f 1 :[ 1, 1]  [  /2, /2]  (b) f 1 :[1, 1]  [0, ]
f 1 (x)  sin 1 x f 1 (x)  cos1 x
Y
y=arc sinx
/2 Y
y

–1
X' 0 1 X
x
/2

/2
y=arc sinx X' X
–1 O 1 x
–1
(y = Y'
sin x) Y' –1 x)
(y = cos –1
(y = sin–1 x) (y = cos x)

E 29

(c) f 1 : R  (   /2,  /2) (d) f 1 : R  (0, )
f 1 (x)  tan 1 x f 1 (x)  cot 1 x

Y Y
y y

/2
y=arc tanx y=arc cotx
X' X
x
/2
0
y=arc tanx y=arccotx
X' X
x
–/2 0
Y' Y'

(e) f–1 : R – (–1, 1)  [0, ]–{ / 2} (f) f–1 : R – (–1, 1)  [  /2, /2]–{0}
f 1 (x)  sec 1 x f 1 (x)  cosec 1 x
Y
y
Y
y /2

/2 X' –1 X
x
0

/2
X' xX
–1 0

Y' Y'

30 E

EXERCISE-I
1. Shown below is the graph of a function ƒ(x) whose domain is R –(–1, 1). Some portion of the
graph is hidden behind the star.

y
ƒ(x)

x
0
–10 –5 5 10

then f(x) =

(A) sec–1 x (B) cosec–1 x (C) tan–1 x (D) cos–1 x


2. The principal value of sec–1(–2) is ?
 2  
(A) (B) (C) (D)
3 3 4 6
 13 
3. The value of cos1  cos  is
 6 
13 7 5 
(A) (B) (C) (D)
6 6 6 6
  1 
4. The principal value of cos1 sin  cos1   is ?
  2 
   
(A) (B) (C) (D)
6 3 2 4
5. The principal value of cot sin1 cos  tan 1 1  is :
1 1
(A) (B)  (C) (D) 1
2 2
 1 
6. tan 2 tan 1  ?
 5 4 
7 7 7 7
(A) (B) (C) (D)
17 17 12 12
 5
7. sin 2 tan 1  = ?
 8
25 80 75
(A) (B) (C) (D) None of these
64 89 128
E 31

 2   2 
8. cos1  cos   sin 1  sin   ?
 3   3 
4  3
(A) (B) (C) (D) 
3 2 4
1
9. Find the domain of the function ƒ(x)  sec 1 (5x  3) .
2

2 4 2 4 2 4
(A) x  or x  (B) x  or x  (C) x  (D) x 
5 5 5 5 5 5
  3
10. The principal value of tan 1 2sin  2 cos1   is ?
  2 

   
(A) (B) (C) (D)
2 6 4 3

ASSERTION-REASON BASED QUESTIONS


In the following questions, a statement of Assertion (A) is followed by a statement of
Reason (R). Choose the correct answer out of the following choices.
(A) Both A and R are true and R is the correct explanation of A.
(B) Both A and R are true but R is not the correct explanation of A.
(C) A is true but R is false.
(D) A is false but R is true.
  3 
11.  
Assertion (A): The value of sin  tan 1  3  cos 1     is 1.
  2  

Reason (R): tan–1(x) = – tan–1x and cos–1(x) = cos–1(x).

ANSWER KEY
Q. No. 1 2 3 4 5 6 7 8 9 10
Ans. A B D A D B B D B D
Q. No. 11
Ans. C

32 E

SOLUTIONS
1. (A) f(x) has domain R– (–1,1)

In given options sec–1 x and cosec–1 x has domain R – (–1, 1) but range of sec–1 x is [0, ]  

2
.


In given graph, we say that f(x) does not include and graph of sec–1x is given below :
2
y


x' x
0
–10 –5 5 10

y'
Hence, f(x) is sec–1x.
2. (B) Let sec–1(–2) =   sec = –2
1 2
 cos     cos 
2 3

   
2
3
 [0, ] 

2

 Aliter



3

sec–1(–2) =  – sec–1 2 =  – sec–1  sec  =   =
3
2
3
 [0, ] 

2
 
 13   13 
3. (D) cos1  cos   [0, ]
 6  6 
       
 cos1 cos  2     = cos 1 cos   
  6    6 

=  [0, ]
6
 1 
4. (A) We know that cos1   
 2 3

  1     3 
 cos1 sin  cos1   cos1  sin   cos1    [0, ]
  2   3  2  6

5.

  
 
4 

(D) cot sin 1 cos   cot  sin 1

1 

2

 cot  1 R
4

E 33

1  1
6. (B) Let 2 tan 1    tan 
5 2 5
1  1
2 tan  2 5 
tan   2      2  25   5
1  
1  tan 2   1    5 24  12
1
2  25 
  5   7 
  1
tan   tan  

 1   
tan 2 tan 1    tan      4   12    12   7
 5 4  4  1  tan .tan   5   17  17
1  1
4  12   12 
  5 
  5       2x  
7. (B) sin 2 tan 1    = sin tan 1  4    2 tan 1 x  sin 1  
  8    1  25     1  x 2  
  64  
 80   80  80
= sin  tan 1  = sin  tan 1  =  [ 1,1]
 39   39  89
 2   2         
8. (D) cos1  cos   sin 1  sin   cos1 cos       sin 1 sin     
 3   3    3    3 
  
   1    2  
 cos1   cos   sin 1 sin  cos1        = 
 3 3  2  3  3 3
9. (B) Domain of sec–1x is (–, –1]  [1, ).
 5x  3  1 or 5x  3  1
2 4
 x  or x 
5 5
  3
10. (D) tan 1 2sin  2 cos1 
  2 
        3


 tan 1 2sin  2     tan 1 2sin     tan 1 2 
 6    3  
  tan
2 
1
 3   3
  3 
11. 
(C) Assertion : sin  tan 1  3  cos 1    
  2  
  3     
 sin   tan 1  3     cos 1
    sin        sin  1
  2    3 6 2
Reason : tan–1 (–x) = – tan–1 x is true. But cos–1 (–x) = cos–1 x false. So reason is false.
Hence, Assertion is true but reason is false.

34 E

EXERCISE-II
 9 
1. Find the value of tan 1  tan  .
 8 

1 3 4 7
2. Show that : tan  sin 1   .
2 4 3

  1 
3. For the principal values, evaluate tan 1 2 cos  2 sin 1   .
  2 

 1 
4. Find the range (principal value branch) of the function ƒ(x)  3cos1    2 . Show your
 2x  1 

work.

 sin 1 x  cos1 x  3
5. Write the value of the expression tan   , when x  .
 2  2

 x 
6. Write the simplest form of tan 1  .
1 1 x
2

 ax 
7. Write the function in the simplest form tan 1   , –a < x < a.
 ax 

 2x 1 ·3x 
1
8. Write the simplest form of sin  x
.
1  (36) 

5 3 63
9. Show that : sin 1  cos1  tan 1
13 5 16

CASE STUDY
10. The Government of India is planning to fix a hoarding board at the face of a building on the road
of a busy market for awareness on COVID-19 protocol. Ram, Robert and Rahim are the three
engineers who are working on this project. "A" is considered to be a person viewing the hoarding
board 20 metres away from the building, standing at the edge of a pathway nearby. Ram, Robert
and Rahim suggested to the firm to place the hoarding board at three different locations namely
C, D and E. "C" is at the height of 10 metres from the ground level. For the viewer A, the angle
of elevation of "D" is double the angle of elevation of "C". The angle of elevation of "E" is triple
the angle of elevation of "C" for the same viewer.
E 35

Look at the figure given and based on the above information, answer the following questions:
E

10m

A' 5m A 20 m B

(i) Evaluate CAB = ?

(ii) Evaluate DAB = ?

(iii) Evaluate EAB = ?

OR

(iii) A' is another viewer standing on the same line of observation across the road. If the width of
the road is 5 meters, then the difference between CAB and CAB is ?

36 E

SOLUTIONS
 9             
1. tan 1  tan  = tan 1 tan      = tan 1 tan    =    , 
 8    8    8  8  2 2

1 3
2. LHS = tan  sin 1 
2 4

3 3 9 7
Let sin 1   then sin   and cos   1  
4 4 16 4

 1  cos  1 7 / 4 4 7 4 7 4 7
Now tan     
2 1  cos  1 7 / 4 4 7 4 7 3

  1     
3.  tan–1 2 cos  2 sin 1   = tan–1 2 cos  2   
  2    6 

   1    
= tan–1  2cos  = tan 1  2    tan –1 (1)     , 
 3   2  4  2 2

  
4. 0  cos1  
 2x  1 
  
3(0)  2  3cos1    2  3()  2
 2x  1 
3 (0) – 2  y  3 () – 2
2  y  3  2
 sin 1 x  cos1 x 
5. tan  
 2 
3
Put x =
2
 1  3  1
 3  
 sin    cos   
= tan   2   2    tan  3 6   tan    = 1
    
 2   2  4
 x 
6. tan 1 
2 
1 1 x 
Put x = sin = sin–1 x
  
 2 sin cos 
 sin   1  sin   2 2
 tan 1  1
 = tan  1  cos   = tan  
 1  1  sin    2 cos2  
2

 2 
    1
= tan 1  tan   = sin 1 x    sin1 x
 2 2 2

E 37

 ax 
7. tan 1  
 ax 
x
Put x = a cos    cos 1
a

a  a cos  1  cos  2sin 2  2 


= tan 1  tan 1 = tan 1  tan 1 tan
a  a cos  1  cos  2 cos  2
2
2

      
= tan 1  tan   a  x  a  0    0    tan  tan 
 2  2 2 2 2
 1 x
=  cos 1
2 2 a

 2x 1 ·3x  1  2 · 6
x

8. sin 1  x
= sin  x 2
1  (36)  1  (6 ) 
Put 6x = tan = tan–16x

= sin 1  2 · tan 
1  tan 
2 
= sin–1 (sin2) = 2= 2 tan–16x [  = tan–1 6x]

5 3 63
9. We have sin 1  cos 1  tan 1
13 5 16
5 5
Let sin 1  x  sin x 
13 13
25 144
and cos2 x  1  sin 2 x  1   13
169 169
5
144 12 x
 cos x  
169 13 12
5
sin x 13 5
 tan x   
cos x 12 12
13
3 3
Again, let cos 1  y  cos y 
5 5
5
2
3 9 16 4 4
 sin y  1  cos2 y  1     1    y
5 25 25 5
3
4
sin y 5 4
 tan y   
cos y 3 3
5

38 E

tan x  tan y
We know that, tan(x  y) 
1  tan x · tan y
5 4 15  48 63

63 63
 tan(x  y)  12 3  36  36   x  y  tan 1
5 4 36  20 16 16 16
1 ·
12 3 36 36
5 3 63
  sin 1  cos 1  tan 1 Hence proved.
13 5 16
10. (i) Given : DAB = 2 CAB = 2 E
and EAB = 3 CAB = 3 
D
In CAB, we have :
BC
tan = C
AB
10 1 2
3 10m
 tan = 
20 2 
B
A' 5m A 20 m
 1
or  = tan–1   ....(1)
 2

(ii) In DAB, DAB = 2

 1  1  1  
  DAB = 2 tan–1      tan  2  
 2   

 1 
 2 
1  2   1 1  2x 
= tan
  1 2   2 tan x  tan  1  x 2 
  
1    
 2 
4
  DAB = 2= tan 1   ....(2)
3
(iii) In EAB,
4 1
EAB = 3 = 2 +  EAB = tan–1   + tan 1   [from 1 & 2]
3 2

 4 1 
   
3 2  1  x  y  
= tan 
1 1 1
 tan x  tan y  tan  
1   4  1     1  xy  
  3 2  

 11 
 EAB = tan 1  
2
E 39

OR
1
(iii) CAB = tan 1   [from (1)]
2
 10  2
and CA'B = tan 1   = tan–1  
 25  5
 CAB – CA'B
 1 2 
     x  y 
1  1  1  2 
= tan    tan   = tan 1
 2 5   tan 1 x  tan 1 y  tan 1  
2 5  1  1  2     1  xy  
 2 5
  

1
= tan 1  
 12 

40 E

NCERT IMPORTANT QUESTIONS
1 1
1. Find the values of cos–1 + 2 sin–1 . [Ex.2.1, Q.12]
2 2
2. tan–1 ( 3) – sec –1 (–2) is equal to [Ex.2.1, Q.14]
  2
(A)  (B) – (C) (D)
3 3 3
 cos x  sin x 
Write the function in the simplest form tan–1  ,0<x<. [Ex.2.2, Q.5]
 cos x  sin x 
3.

1 2x 1 1  y 
2

Find the value of tan sin 1  cos , | x |  1, y  0 and xy  1 [Ex.2.2, Q.9]


1  y 2 
4.
2 1  x2

  1
5. sin  – sin –1  –   is equal to [Ex.2.2, Q.15]
3  2 
1 1 1
(A) (B) (C) (D) 1
2 3 4
 7 
6. Find the value of tan–1  tan  . [Misc. Ex., Q. 2]
 6
 1  sin x  1  sin x  x  
7. Prove that : cot 1    , x   0,  [Misc. Ex., Q.9]
 1  sin x  1  sin x  2  4
 1 x  1 x  x 1 1
Prove that : tan 1     cos x,   x 1
1
8. [Misc. Ex., Q.10]
 1 x  1 x  4 2 2
9. Solve : 2tan–1 (cos x) = tan–1 (2 cosec x) [Misc. Ex., Q.11]

10. sin 1 (1  x)  2 sin 1 x  then x is equal to [Misc. Ex., Q.14]
2
1 1 1
(A) 0, (B) 1, (C) 0 (D)
2 2 2
 cos x  3 
11. Express tan 1  ,  x  in the simplest form. [Example 4]
 1  sin x  2 2
 3 
12. Find the value of sin 1  sin  . [Example 6]
 5 

ANSWER KEY
2  xy 
1. 2. (B) 3. x 4. 5. (D) 6.
3 4 1  xy 6
  x 2
9. x  n  , n  Z 10. (C) 11.  12.
4 4 2 5

E 41

PREVIOUS YEAR QUESTIONS
1 MARK QUESTIONS
1  1 
1. The principal value of cos1    sin 1    is : [CBSE 2022]
2  2
  
(A) (B)  (C) (D)
12 3 6

1  1      4   3 
Sol. (A) cos1    sin 1    =     = 12  12 [1]
2  2  3  4

 9 
2. The principal value of tan 1  tan  is : [CBSE 2022]
 8 
 3  3
(A) (B) (C)  (D) 
8 8 8 8
  9               
Sol. (A) tan 1  tan    = tan 1 tan      = tan 1 tan    =    ,  [1]
  8    8    8  8  2 2 
3. What is the domain of the function cos–1 (2x – 3) ? [CBSE 2022]
(A) [–1, 1] (B) (1, 2) (C) (–1, 1) (D) [1, 2]
Sol. (D) For the given function ; –1  2x – 3  1
2  2x  4 1  x  2 [1]

4. The principal value of  tan 1 3  cot 1 ( 3) is : [CBSE 2018, 2022]


(A)  (B)  (C) 0 (D) 2 3
2
      5 3 
Sol. (B) tan 1 ( 3)  cot 1 (  3) =  (  cot 1 ( 3)) =      =    [1]
3 3  6 3 6 6 2
5. The range or the principal value branch of the function y = sec–1 x is ............ . [CBSE 2020]
OR
 1
The principal value of cos1    is ............... .
 2
Sol. y = sec–1x (given)
Range of sec–1 x is [0, ] – {/2} [1]
OR

1  1 1  
Let cos     y  cos y   cos y = cos    
 2 2  3
2 2
 cos y = cos  y   [0, ] [1]
3 3

42 E

2 MARKS QUESTIONS

Simplify sec–1  2  , 0 < x <


1 1
6. . [CBSE 2021 C]
 2x  1  2

Sol. Let y = sec–1  2  , 0 < x <


1 1
 2x  1  2
Put x = cos ; then   cos1 x [1]

 y = sec–1  
1

 2 cos   1 
2

= sec–1 
1  –1 –1
 = sec (sec2) = 2 = 2 cos x [1]
 cos 2 

 1 1
7. Prove that : 3sin–1x = sin–1(3x – 4x3), x    ,  [CBSE 2018]
 2 2
 1 1
Sol. To prove : 3sin–1x = sin–1(3x – 4x3), x    , 
 2 2
R.H.S. = sin–1(3x – 4x3)
Put x = sin;  = sin–1x [1]
 R.H.S. = sin–1(3sin – 4sin3)
= sin–1(sin3) = 3 = 3sin–1x = L.H.S. [1]
Hence, proved

E 43

IMPORTANT NOTES
__________________________________________________________________________________
__________________________________________________________________________________
__________________________________________________________________________________
__________________________________________________________________________________
__________________________________________________________________________________
__________________________________________________________________________________
__________________________________________________________________________________
__________________________________________________________________________________
__________________________________________________________________________________
__________________________________________________________________________________
__________________________________________________________________________________
__________________________________________________________________________________
__________________________________________________________________________________
__________________________________________________________________________________
__________________________________________________________________________________
__________________________________________________________________________________
__________________________________________________________________________________
__________________________________________________________________________________
__________________________________________________________________________________
__________________________________________________________________________________
__________________________________________________________________________________
__________________________________________________________________________________
__________________________________________________________________________________
__________________________________________________________________________________
__________________________________________________________________________________
__________________________________________________________________________________
__________________________________________________________________________________
__________________________________________________________________________________
__________________________________________________________________________________

44 E


CHAPTER-3 : MATRICES
1. DEFINITION :
A rectangular arrangement of numbers in rows and columns, is called a matrix. Such a
rectangular arrangement of numbers is enclosed by small ( ) or big [ ] brackets. Generally a
matrix is represented by a capital letter A, B, C......... etc. and its element are represented by small
letters a, b, c, x, y etc.
p q  2 4 6   5
Examples : A =  , B= 1 0 3  , C = 1  , D = [2 3 4], E = [7]
r s    

2. ORDER OF MATRIX :
A matrix which has m rows and n columns is called a matrix of order m × n, and its represented
by Am× n or A = [aij]m× n
It is obvious that a matrix of order m × n contains mn elements. Every row of such a matrix
contains n elements and every column contains m elements.
 1 2  2 5 6  5
Examples : A =   , B=   ,C= 1 , D = [2 3 4]1×3, E = [7]1×1
 1 3  22  3 9 3  23   21

3. TYPES OF MATRICES :
(i) Row matrix : If in a matrix, there is only one row, then it is called a Row Matrix.
Thus A=[aij]m×n is a row matrix if m = 1
Example : [2 4 6] is a row matrix of order 1 × 3
(ii) Column Matrix : A matrix which contains only one column, is called a column matrix.
Thus, A = [aij]m×n is a column matrix if n = 1.

5
 
Example :  6  is column matrix of order 3 × 1.
 7 

(iii) Square matrix : If number of rows and number of columns in a matrix are equal, then it is
called a square matrix.
Thus, A = [aij]m×n is a square matrix if m = n.

 2 3 6 
 1 2  
Example : A =   , B =  1 2 0 
 1 3  22  5 1 7  33

E 45

(iv) Diagonal matrix : If all elements except the principal diagonal in a square matrix are zero,
it is called a diagonal matrix. Thus a square matrix A = [aij] is a diagonal matrix if aij = 0,
when i  j.

2 0 0 
Example : 0 3 0  is a diagonal matrix of order 3 × 3, which also can be denoted by
0 0 4 

diag. [2 3 4]

(v) Scalar Matrix : If all the elements of the diagonal of a diagonal matrix are equal, it is
called a scalar matrix.
0 i  j
Thus a square matrix A=[aij] is a scalar matrix is aij =  where k is a constant.
k i  j
 2 0 0 
3 0 
Example : A =   , B =  0 2 0 
0 3   0 0 2 
(vi) Unit Matrix (Identity Matrix) : If all elements of principal diagonal in a diagonal matrix
are 1, then it is called unit matrix. A unit matrix of order n is denoted by In.
1 i  j
Thus a square matrix A = [aij] is a unit matrix if aij = 
0 i  j
1 0 0 
1 0  0 1 0 
Example : 2 =   , 3 =  
0 1  0 0 1 
Note : Every unit matrix is a scalar matrix.
(vii) Null or zero matrix : If in a matrix all the elements are zero then it is called a zero matrix
and it is generally denoted by O.
Thus, A = [aij]m×n is a zero matrix if aij = 0 for all i and j.
0 0  0 0 0   0 
Example :  ,   ,   , [0 0 0]
0 0  0 0 0   0 

4. EQUALITY OF MATRICES :
Two matrices A and B are said to be equal matrix if they are of same order and their
corresponding elements are equal
5 6 7 a a2 a3 
eg. if A =   and B =  1 are equal matrices then
2 4 8   b1 b2 b3 

a1 = 5, a2 = 6, a3 = 7, b1 = 2, b2 = 4, b3 = 8

46 E

 x  y 2x  z   1 5 
Illustration 1: Find x, y, z and w if   .
2x  y 3z  w   0 13
xy 2x  z   1 5 
Solution: Given   .
2x  y 3z  w   0 13
By definition of equality, we have
x–y=–1 ...(i) 2x – y = 0 ...(ii)
2x + z = 5 ...(iii) 3z + w = 13 ...(iv)
To find x and y ; subtract (ii) from (i) to get x – 2x = –1
x = 1 and then from (i) y = x + 1 = 1 + 1 = 2.
To find z and w ; substitute x = 1 in (iii) to obtain z = 5 – 2x = 5 – 2 = 3 and then from (iv)
w = 13 – 3z = 13 – 3 × 3 = 4
Hence, x = 1, y = 2, z = 3 and w = 4.

5. ADDITION AND SUBTRACTION OF MATRICES :


If A = [aij]m×n and B = [bij]m×n are two matrices of the same order then their sum A + B is a
matrix whose each element is the sum of corresponding elements.
i.e., A + B = [aij + bij ] m×n

5 2 1 5   5  1 2  5  6 7 
Example : If A =  1 3  and B =  2 2  , then A + B =
 
1  2 3  2  =  3 5 
   
 4 1  32  3 3  32  4  3 1  3   7 4  32

Similarly their subtraction A – B is defined as A – B = [aij – bij]m×n

 5  1 2  5  4 3 
  
i.e., in above example A – B = 1  2 3  2  =  1 1 

 4  3 1  3   1 2  32

Note : Matrix addition and subtraction can be possible only when matrices are of same order.
Properties of matrices addition :
If A, B and C are matrices of same order, then -
(i) Commutative Law : A + B = B + A
(ii) Associative law : (A + B) + C = A + (B + C)
(iii) Additive Identity : A + O = O + A = A (where O is zero matrix is additive identity of the
matrix A).
(iv) Additive Inverse : A + (–A) = O = (–A) + A (where (–A) is additive inverse of the matrix A).

E 47

6. SCALAR MULTIPLICATION OF MATRICES :
Let A=[aij]m×n be a matrix and k be a number then the matrix which is obtained by multiplying
every element of A by k is called scalar multiplication of A by k and it denoted by kA.
Thus A = [aij]m×n  kA = [kaij]m×n

4 2 20 10 
e.g. if A =  3 5  then 5A = 15 25
 
 6 7  30 35

Properties of scalar multiplication :

If A, B are matrices of the same order and m, n are any numbers, then the following results can be
easily established.
(i) m(A + B) = mA + mB
(ii) (m + n)A = mA + nA
(iii) m(nA) = (mn)A = n(mA)
 sin   cos    cos  sin  
Illustration 2: If A    and B   then compute (sin) A + (cos) B.
cos  sin     sin  cos  

 sin   cos    cos  sin  


Solution: (sin) A + (cos) B = sin   + cos  
 cos  sin     sin  cos  

 sin 2   sin  cos    cos2  cos  sin  


      
sin  cos  sin 2     cos  sin  cos2  

 sin 2   cos2   sin  cos   sin  cos   1 0 


  
sin  cos   cos  sin  sin 2   cos2   0 1 
Illustration 3:
4 2  2 1   x 4 
(i) Find x, y if 3   2  O
1 3   3 2 3 y 
 1 2   3 2 
(ii) Find the matrix X such that 2A + B + X = O, where A    and B  1 5  .
 3 4  
4 2   2 1   x 4 
Solution: (i) Given 3    2  O
1 3   3 2 3 y 
12 6   4 2   x 4  12  (4)  x 6  2  4  0 0 
  3 9   6 4  3 y   O   
9  4  y  0 0 
       363
 16 + x = 0 and 5 + y = 0
x = –16 and y = –5. (By definition of equality)

48 E

(ii) Given 2A + B + X = O
 1 2  3 2   2 4  3 2 
 2     X  O    XO
 3 4  1 5   6 8  1 5 
 2  3 4  2  1 2  0 0 
  6  1 8  5   X  O   7 13   X  0 0 
     
1 2   x y   0 0  x y 
  7 13   z t    0 0  , where X   
      z t 
1  x 2  y  0 0 
  7  z 13  t   0 0 
   
 1 + x = 0, 2 + y = 0, 7 + z = 0 and 13 + t = 0
 x = –1, y = –2, z = –7 and t = –13.
 x y   1 2 
Hence, X    .
 z t   7 13

1 2 3   3 0 1
Illustration 4: Find A and B, if 2A  3B    and A – 2B =  .
 2 0 1  1 6 2 

1 2 3 
Solution: Given 2A  3B    .....(1)
 2 0 1

 3 0 1
and A  2B    …..(2)
 1 6 2 

Multiplying (2) by 2 and subtracting it from (1), we obtain

1 2 3   3 0 1
2A + 3B – 2(A – 2B) =    2 
2 0 1  1 6 2 

1  6 2  0 3  2  1   5 2 1 
 7B     B   
2  2 0  12 1  4  7  4  12 5

Multiplying (1) by 2, (2) by 3 and adding, we get

1 2 3   3 0 1
2(2A + 3B) + 3(A – 2B) = 2   3 
2 0 1  1 6 2 

2  9 4  0 6  3  1 11 4 9 
  7A     A  
 4  3 0  18 2  6  7  1 18 4 

E 49

7. MULTIPLICATION OF MATRICES :
If A and B be any two matrices, then their product AB will be defined only when number of columns
in A is equal to the number of rows in B. If A = [aij]m×n and B = [bij]n×p then their product

AB = C = [cij], will be matrix of order m × p, where


n
(AB)ij = Cij   a ir b rj
r 1

1 2 
1 4 2  1.1  4.2  2.1 1.2  4.2  2.3 11 16 
Example : If A =   and B =  2 2  then AB =   AB =  
2 3 1  1 3   2.1  3.2  1.1 2.2  3.2  1.3   9 13 

Properties of matrix multiplication :

If A, B and C are three matrices such that their product is defined, then

(i) AB  BA (Generally not commutative)

But multiplication of diagonal matrices of same order will be commutative.

1 0  3 0  3 0 
If A    , B   , then AB  BA  
 0 2   0 4  0 8 

(ii) (AB) C = A (BC) (Associative Law)

(iii) IA = A = AI   is identity matrix for matrix multiplication

(iv) A (B + C) = AB + AC (Distributive law)

(v) If AB = AC 
 B=C (cancellation Law is not applicable)

(vi) If AB = O It does not mean that A=O or B=O, again product of

two non-zero matrix may be zero matrix.

0 1  2 0  0 0 
Example : A =   and B =  0 0  , then AB = 0 0 
0 0     

1 1 a 1  2 2 2
Illustration 5: If A    , B  and (A + B) = A + B , find a and b.
 2 1  b 1
Solution: We have, (A + B)2 = A2 + B2
 (A + B) (A + B) = A2 + B2
 (A + B)A + (A + B)B = A2 + B2 [By distributive law]
 A2 + BA + AB + B2 = A2 + B2 [By distributive law]
 BA + AB = O

50 E

 a 1  1 1 1 1  a 1  0 0 
  b 1 2 1  2 1  b 1  0 0 
       
 a  2 a  1   a  b 2   0 0 
  b  2  b  1  2a  b 3   0 0 
     
2a  b  2 a  1  0 0 
  2a  2 
  b  4  0 0 
 2a – b + 2 = 0, –a + 1 = 0, 2a – 2 = 0 and –b + 4 = 0
 a = 1, b = 4
 2 3 2
Illustration 6: Let A    and ƒ(x) = x – 4x + 7. Show that ƒ(A) = O.
 1 2 
2
Solution: We have, ƒ(x) = x – 4x + 7
2
 f(A) = A – 4A + 7I2

 2 3   2 3   4  3 6  6   1 12 
Now, A 2     =  
 1 2   1 2   2  2 3  4   4 1 
 8 12  7 0 
4A    and 7I 2   
 4 8  0 7 
2
 f(A) = A – 4A + 7I2

 1 12   8 12  7 0 
 f (A)     
 4 1   4 8   0 7 
 1  8  7 12  12  0  0 0 
 f (A)    O
 4  4  0 1  8  7  0 0 
 2 1   1 8 
Illustration 7 : Find matrix A such that  1 0  A   1 2  .
 
 3 4   9 22 
   
a c 
Solution : Let matrix A   
b d 
 2 1  1 8   2a  b 2c  d   1 8 
 1 0   a c    1 2      1 2 
   b d      a c   
 3 4  32   22  9 22 
  32 
 3a  4b 3c  4d 
   9 22 

2a – b = –1 and 2c – d = –8
a = 1, c = –2
b=3,d=4
1 2 
So A 
3 4 
E 51

8. TRANSPOSE OF MATRIX :
If we interchange the rows and columns of a matrix A, then the matrix so obtained is called the
T t
transpose of A and it is denoted by A or A or A'
From this definition it is obvious to note that
T
(i) Order of A is m × n  order of A is n × m
(ii) (Aij)' = (Aji),  i, j

Properties of Transpose :
If A, B are matrices of suitable order then
(i) (A')' = A (ii) (A ± B)' = A' ± B' (iii) (kA)' = kA' (iv) (AB)' = B'A'
1 2 
0 3 4 
Illustration 8: If A  3 0  , B   , prove that (AB)' = B'A'.
 1 2 0 
5 6 

1 2 
0 3 4 
Solution: Here, AB  3 0  
 1 2 0 
5 6 

1 0  (2) 1 1 (3)  (2)  2 1 4  (2)  0  2 7 4 


  3  0  0 1 3  (3)  0  2 3  4  0  0    0 9 12 
 5  0  6 1 5  (3)  6  2 5  4  6  0   6 3 20 

 2 0 6 
  (AB)'   7 9 3 ...(1)
 4 12 20 

 0 1  0 1  1 (2) 0  3  1 0 0  5  1 6 
   1 3 5 
Now, B' A '   3 2     (3) 1  2  (2) 3  3  2  0 (3)  5  2  6 
 2 0 6   4 1  0  (2)
 4 0   43  0 0 4  5  0  6 

 2 0 6 
  7 9 3 ...(2)
 4 12 20 
From (1) and (2), we have (AB)' = B'A'.

9. SYMMETRIC AND SKEW-SYMMETRIC MATRIX :


(a) Symmetric matrix : A square matrix A = [aij] is called symmetric matrix if aij = aji  i, j
or A' = A.
1 2 3 

Example : 2 3 1 

 3 1 2 

52 E

Note :
(i) Every unit matrix and square zero matrix are symmetric matrices.
n(n  1)
(ii) Maximum number of different elements in a symmetric matrix is .
2
(b) Skew-symmetric matrix : A square matrix A = [aij] is called skew-symmetric matrix if
aij = – aji  i, j or A' = –A
 0 2 3
Example :  2 0 1 
 3 1 0 

Note : All principal diagonal elements of a skew-symmetric matrix are always zero because
for any diagonal element aii = – aii  aii = 0
Properties of symmetric and skew-symmetric matrices :
T T T T
(i) If A is a square matrix, then A + A , AA , A A are symmetric matrices while A–A is
skew-symmetric matrices.
(ii) Every square matrix A uniquely can be expressed as sum of a symmetric and skew
1  1 
symmetric matrices i.e. A =  (A + A)  +  2 (A – A) 
2   
Symmetric Skewsymmetric
matrix matrix

2 4 1
Illustration 9: Express  3 5 8  as the sum of a symmetric and a skew-symmetric matrix.
1 2 1 

2 4 1 2 3 1
Solution: Let A   3 5 8   A   4 5 2  
 
1 2 1   1 8 1 
2 4 1  2 3 1   4 7 0 
 A  A  3 5 8    4 5 2    7 10 6  
1 2 1   1 8 1  0 6 2 

4 7 0   2 7 / 2 0
1 1  
Now, (A  A)   7 10 6    7 / 2 5 3  = P (say) …..(1)
2 2
0 6 2   0 3 1 

 2 7 / 2 0

Here, P   7 / 2 5 3   P
 0 3 1 

So, P is symmetric matrix


E 53

2 4 1 2 3 1  0 1 2 
T    
and, A  A  3 5 8    4 5 2    1 0 10 
1 2 1   1 8 1   2 10 0 
 0 1 2   0 1 / 2 1
1 1  
 (A  A)   1 0 10    1 / 2 0 5   Q (say) ….(2)
2 2
 2 10 0   1 5 0 

 0 1 / 2 1 
Q  1 / 2 0 5  Q
 1 5 0 
So, Q is skew-symmetric matrix.
Adding equation (1) and (2)
 2 7 / 2 0  0 1 / 2 1 2 4 1
 7 / 2 5 3    1 / 2 0 5   3 5 8  .
  
 0 3 1   1 5 0  1 2 1 

10. INVERTIBLE MATRICES :


If A is a square matrix of order m and if there exists another square matrix B of the same order m,
such that AB = BA = I, then B is called the inverse matrix of A and it is denoted by A–1. In that
case A is said to be invertible.
2 3   2 3
For example, Let A =   and B =   be two matrices.
1 2   1 2 
2 3   2 3  4  3 6  6  1 0 
Now, AB =   =   I
1 2  1 2  2  2 3  4  0 1 
1 0 
Also BA     I.
0 1 
Thus B is the inverse of A, in other words B = A–1 and A is inverse of B, i.e., A = B–1
Uniqueness of inverse : Inverse of square matrix, if it exists, is unique.
Proof ; Let A = [aij] be a square matrix of order m. If possible, let B and C be two inverses of A.
We shall show that B = C.
Since B is the inverse of A
AB = BA = I
Since C is also the inverse of A
AC = CA = I
Thus, B = BI = B (AC) = (BA) C = IC = C

54 E

EXERCISE-I
1 a   2 3  1 3
1 If X =   and 3X –  0 2    0 1  then 'a' is equal to-
0 1     
(A) 1 (B) 2 (C) 0 (D) –2
0 1 
2. If A    and a and b are arbitary constants then (aI + bA) =
2

 0 0 
(A) a2I + abA (B) a2I + 2abA
(C) a2I + b2A (D) None of these
1 0   0 1  cos  sin  
3. If I =  , J =  1 0  and B =   sin  cos   , then B equals
0 1     
(A) I cos  + J sin  (B) I cos  – J sin 
(C)  sin  + J cos  (D) – I cos  + J sin 
2 3 1   x 
4. If [1 x 2] 0 4 2   1  = O, then the value of x is-
0 3 2   1

(A) –1 (B) 0 (C) 1 (D) 2

5. P and Q are matrices such that both (P + Q) and (PQ) are defined. Which of the following is
true about P and Q ?
(A) P and Q can be any matrices but of the same order.
(B) P and Q must be square matrices of the same order.
(C) P and Q must be square matrices not necessarily of the same order.
(D) Order of P and Q must be form m × k and k × n respectively, with no condition on m and n.
 2 1 4 1 
6. If A =   and B =  7 2  then B'A' is equal to-
 7 4   
1 0  1 1 0 1  1 0 
(A)   (B)   (C)   (D)  
0 1  1 1 1 0  0 0 
1, if i  j
7. If matrix A  a ij  , where a ij   , then A2 is equal to :
22
0, if i  j
(A) I (B) A (C) O (D) –I
1 0   a 1  2 1 
8. If     , then values of a, b are-
3 4   1 b  2 2 
(A) 1, –2 (B) –1, 2 (C) –1, –2 (D) 1, 2

E 55

9. If A is a square matrix such that A2 = I then (A – I)3 + (A + I)3 – 7A is equal to
(A) A (B) I – A (C) I + A (D) 3A
1 4 
10. If A    , then skew symmetric part of A is :-
2 3 

 1 
 1 
 0 1 0 1   0 1
(A)   (B)   (C)   (D)  2 
 1 0  1 0   1 0   2 3 
 2 

ASSERTION-REASON BASED QUESTIONS


In the following questions, a statement of Assertion (A) is followed by a statement of Reason (R).
Choose the correct answer out of the following choices.
(A) Both A and R are true and R is the correct explanation of A.
(B) Both A and R are true but R is not the correct explanation of A.
(C) A is true but R is false.
(D) A is false but R is true.

2 3
2 3 1
11. Assertion (A) : If A    and B   4 5 , then AB and BA both are defined.
1 4 2  2 1 
 
Reason (R) : For the two matrices A and B, the product AB is defined, if number of columns in
A is equal to the number of rows in B.

ANSWER KEY
Q. No. 1 2 3 4 5 6 7 8 9 10
Ans. B B A A B A A D A A
Q. No. 11
Ans. A
56 E

SOLUTIONS
1 a   3 3a 
1. (B) Given X =    3X   
0 1  0 3 
 2 3   1 3  3 3a   2 3  1 3 1 3a  3 1 3
Also given 3X               
0 2  0 1 0 3  0 2  0 1  0 1  0 1 
Now by equality of two matrices, we have 3a – 3 = 3  a = 2.
0 1 
2. (B) Given A =  
0 0 
a 0 0 b a b
Here aI + bA =     
0 a  0 0 0 a 
 a b   a b   a 2  0 ab  ba 
(aI +bA)2 =    = 2 
0 a  0 a   0 0 0 a 
 a2 0   0 2ab 
 (aI +bA)2 =   
0 a2   0 0 
1 0 0 1
= a2    2ab    a I  2abA
2

 0 1   0 0 
1 0   0 1  cos  sin  
3. (A) Given I    , J  and B   
0 1   1 0    sin  cos  
 cos  sin    cos  0   0 sin  
Here   =   + 
  sin  cos    0 cos     sin  0 
1 0   0 1
= cos    + sin    = I cos  + J sin 
0 1   1 0 
2 3 1   x  x
4. (A) Given [1 x 2] 0 4 2   1  = O = [2 4x + 9 2x + 5] 1  =O
 
0 3 2   1  1
 [2x + 4x + 9 – 2x – 5] = O
Thus 4x + 4 = 0  x = –1
5. (B) P and Q must square matrices of the same order.
Let P and Q matrices has order m × n and p × q respectively.
If P + Q defined then,
m × n = p × q m = pand n = q …..(1)
If PQ defined then n = p …..(2)
from equation (1) and (2)
n=m=p=q
Thus P and Q must be square matrices of same order.

E 57

 2 1 4 1 
6. (A) Given A =   and B = 7 2
 7 4   
 2 7  4 7
A' =   and B' =  
 1 4  1 2 
 4 7   2 7  8  7 28  28  1 0 
Now B'A' =     =   
1 2   1 4  2  2 7  8  0 1 
1, if i  j
7. (A) Given that A  a ij  , where a ij  
22
0, if i  j
0 1 
Now, A   
1 0 
0 1  0 1  1 0 
A 2  AA     I
1 0  1 0  0 1 
1 0   a 1  2 1 
8. (D) Given    
3 4   1 b  2 2 
1  a 0  1   2 1 
       1 + a = 2 and –4 + b = –2  a = 1, b = 2
 3  1 4  b   2  2 
9. (A) Given that A2 = I
(A – I)3 = A3 – I3 – 3AI (A – I)
= A – I – 3A2 + 3A [ A2 = I]
= A – I – 3I + 3A [ A2 = I]
= 4A – 4I …..(1)
3
Similarly (A + I) = 4A + 4I …..(2)
3 3
Now, (A – I) + (A + I) – 7A = 4A – 4I + 4A + 4I – 7A [From equation (1) & (2)]
=A
1 1  1 4   1 2    0 1
10. (A) We know skew symmetric part of matrix A  A  AT     =  1 0 
2 2 2 3   4 3  
  
2 3
2 3 1
11. (A) Assertion : The given matrices are A   and B   4 5
1 4 2  2 1 
 
Order of A = 2 × 3; Order of B = 3 × 2
Since, number of columns in A is equal to the number of rows in B.AB is defined.
Also, number of columns in B is equal to the number of rows in A. BA is defined.
Reason : By property we know that the product AB is defined, if number of columns in
matrix A is equal to the number of rows in matrix B.
Both Assertion and Reason are true and Reason is the correct explanation of Assertion.

58 E

EXERCISE-II
 cos   sin  
1. If A =   , then find the values of satisfying the equation A' + A = I2.
 sin  cos  

 3 1
2. Find A2 – 5A + 7I if A =  .
 –1 2 

3. 
 sin  cos   
If A  cos  sin  , find a satisfying 0    when A + A' =
2
2I 2 ; where A' is transpose

of A.

3 3
4. If matrix A =   and A2 = A, then write the value of .
 3 3 

a  b 2   6 2 
5. If  7 ab    7 8  , then find the value of a and b.
 –3 4   –3 4 

1 2 2 
6. Show that matrix A  2 1 2  satisfies the equation A2 – 4A – 5I = O.
2 2 1 

1 2 2 
7. If A   2 1 2  is a matrix satisfying AA' = 9I3, then find the values of a and b.
a 2 b 
 

 2 2 4 
8. Express the matrix A=  1 3 4  as the sum of a symmetric and a skew symmetric matrix.
 1 2 3 
 

 0 6 7 0 1 1   2
9. If A  6 0 8  , B  1 0 2  , C   2  , then calculate AC, BC and (A + B)C. Also
 7 8 0  1 2 0   3
     

verify that (A + B)C = AC + BC.

E 59

CASE STUDY-I

10. Read the following passage and answer the questions given below:
Three schools DPS, CVC and KVS decided to organize a fair for collecting money for helping
the flood victims. They sold handmade fans, mats and plates from recycled material at a cost of
Rs. 25, Rs.100 and Rs. 50 each respectively. The numbers of articles sold are given as

School/Article DPS CVC KVS


Handmade fans 40 25 35
Mats 50 40 50
Plates 20 30 40
(i) What is the total money (in Rupees) collected by the school DPS? (Using matrix algebra)
(ii) What is the total amount of money (in Rs.) collected by schools CVC and KVS? (Using matrix
algebra)
(iii) If the number of handmade fans and plates are interchanged for all the schools, then what is the
total money collected by all schools? (Using matrix algebra)
OR
How many articles (in total) are sold by three schools and also find the total amount of money
collected by all three schools DPS, CVC and KVS?

CASE STUDY-II
11. Read the following passage and answer the questions given below:
Assume that X, Y, Z, W and P are matrices of order 2 × n, 3 × k, 2 × p, n × 3 and p × k ;
respectively.
(i) Find the restriction on XZ for which it will be defined.
(ii) If n = p, then find the order of the matrix 7X – 5Z.
(iii) Find the restriction on n, k and p for which PY + WY will be defined.
OR
If p = 3, then find the order of the matrix 3Y + 10P and also find the condition for multiplication
of matrices W and P.

60 E

SOLUTIONS
 cos   sin    cos  sin  
1. We have, A =   A' =  
 sin  cos     sin  cos  
Now, A' + A = I2
 cos  sin   cos   sin  1 0  2 cos  0  1 0 
        
  sin  cos    sin  cos   0 1   0 2 cos  0 1 
1  
 2 cos  = 1  cos  = cos  = cos    = 2n  , n  Z
2 3 3

Given A  
3 1
2.
 1 2 
 3 1   3 1   9  1 3  2   8 5
A2  A  A    
 1 2   1 2   3  2 1  4   5 3

Now, A2 – 5A + 7I  
8 5 15 5  1 0  0 0 
    7 
 5 3   5 10  0 1  0 0 
3. A  A'  2I2
 cos  sin    cos   sin   1 0
   sin  cos     sin  cos    2  0 1 
     
 2 cos  0   2 0 
    
 0
 2 cos    0 2
1   
2 cos   2  cos       0, 
2 4  2 
 3 3
4. Given A = 
 3 3 
A2 = A
 3 3  3 3  3 3
A.A = A       
 3 3   3 3   3 3 
 18 18  3 3  1 1  1 1
 18 18     3 3  or 18  1 1   3  1 1  18 = 3  = 6
   
a  b 2   6 2 
5. Given  7 ab    7 8 
   
 3 4   3 4 
By equality of matrices
a+b=6 …..(1) and ab = 8 …..(2)
Put the value of b in equation (1)
8
a   6  a2 – 6a + 8 = 0
a
Solving we get, a = 2, 4 then put the value of a in equation (2)
a = 4, b = 2 or a = 2, b = 4
E 61

1 2 2 
6. We have, A  2 1 2 
2 2 1 

 1 2 2  1 2 2   9 8 8  4 8 8  5 0 0 
2 
  A  AA  2 1 2  2 1 2   8 9 8  , 4A   8 4 8  and 5I  0 5 0 
      

2 2 1  2 2 1  8 8 9   8 8 4  0 0 5 

9 8 8   4 8 8   5 0 0 
  A  4A  5I  8 9 8   8 4 8   0 5 0 
2

8 8 9 8 8 4  0 0 5 

 9  4  5 8  8  0 8  8  0  0 0 0 
 A  4A  5I  8  8  0 9  4  5 8  8  0   0 0 0  = O
2

8  8  0 8  8  0 9  4  5 0 0 0 


1 2 2  1 2 a 
7. 
Given A  2 1 2  A '  2 1 2 

a 2 b  2 2 b 
   
 AA' = 9I3
1 2 2  1 2 a  1 0 0 
  2 1 2  2 1 2   9 0 1 0 
 a 2 b  2 2 b  0 0 1 
    

 9 0 a  2b  4  9 0 0 
   0 9 2a  2  2b   0 9 0 
 2  
a  2b  4 2a  2  2b a  4  b  0 0 9 
2

  a + 2b + 4 = 0, 2a + 2 – 2b = 0 and a2 + 4 + b2 = 9
  a + 2b + 4 = 0, a – b + 1 = 0 and a2 + b2 = 5
Solving a + 2b + 4 = 0 and a – b + 1 = 0, we get a = – 2 and b = – 1
 2 2 4 
8. Given A   1 3 4 
 
 1 2 3

A square matrix A can be expressed as sum of a symmetric and skew symmetric matrices.
1 1
A  [A  A]  [A  A]
2 2
(P  symmetric (Q  skew symmetric
matrix) matrix)

 2 1 1 
Here A   2 3 2 
 
 4 4 3 

62 E

 3 3
 2   
 4 3 3   2 2

1 1 3
Let P  (A  A)   3 6 2     3 1 
2 2    2 
 3 2 6   3 
 1 3 
 2 
 3 3
 2   
2 2
 
3
Now, P '    3 1   P (say)
 2 
 3 
 1 3 
 2 
 1 5
0   
2 2
 
1 1
Also, Let Q  (A  A )  
 0 3 
2  2 
5 
 3 0 
 2 
 1 5
 0 2 2
 
1
Q    0 3  Q (say)
 2 
 5 
 3 0
 2 
1
Q= (A – A') is a skew symmetric matrix
2
 3 3  1 5
 2    0   
2 2 2 2
     2 2 4 
3 1
Now P  Q    3 1  0 3    1 3 4   A
 2  2   
 3  5   1 2 3
 1 3   3 0 
 2   2 
 0 6 7 0 1 1   2
9. Given A   6 0 8  , B  1 0 2  , C   2 
 7 8 0  1 2 0   3
     
 0 6 7  2
Now, AC  6 0 8  2 
 7 8 0   3
  
 0  12  21  9
AC  12  0  24   12  .........(1)
 14  16  0  30 
   
0 1 1   2  0  2  3   1
BC  1 0 2 2  BC  2  0  6    8
    ..........(2)
 1 2 0   3 2  4  0  2 
      
E 63

  0 6 7  0 1 1    2 
 
(A + B)C =   6 0 8   1 0 2    2 
      3
  7 8 0  1 2 0    
 0 7 8   2   0  14  24  10 
  5 0 10   2   10  0  30   20  .........(3)
 8 6 0   3  16  12  0  28 
      
10 
Now, LHS = (A + B)C =  20  from equation (3)
 28 
 

9 1 10 


RHS = AC + BC = 12    8  =  20  from equation (1) and (2)
30   2   28 
     
Hence, (A +B) . C = AC + BC
10. Let cost of recycled material in matrix form is D = [25 100 50]
Now, let numbers of articles sold by DPS, CVC and KVS schools in matrix form are respectively
 40   25   35 
A  50 , B  40 and C   50 
   
 20  30   40 
     
(i) Total money (in Rupees) collected by the school DPS will given by
 40 
DA   25 100 50   50  = [1000 + 5000 + 1000] = [7000]
 20 
 
Therefore total money (in Rupees ) collected by the school DPS = 7000
(ii) Total amount of money (in Rupees) collected by the school CVC and KVS will given by
DB + DC = D(B + C) (Using distributive law)
  25   35    60 
    
25 100 50   40  50  = 25 100 50  90 
      70 
 30   40    
= [25 × 60 + 100 × 90 + 50 × 70] = [14000]
Therefore total amount (in Rupees) collected by the school CVC and KVC = 14000
(iii) The number of handmade fans and plates are interchanged for all the schools then in matrix
form will be given respectively
 20  30   40 
Let X   50  , Y   40  and Z   50 
 40   25   35 
     
Now, total money collected by all schools will be given by DX + DY + DZ,

64 E

where D = [25 100 50]
 20   30   40 
= [25 100 50] 50 + [25 100 50] 40 + [25 100 50]  50 
   
 40   25   35 
     
= [500 + 5000 + 2000] + [750 + 4000 + 1250] + [1000 + 5000 + 1750]
= [7500] + [6000] + [7750] = [21250]
Therefore, total money collected by all schools = 21250
OR
Total articles = (40 + 25 + 35) + (50 + 40 + 50) + (20 + 30 + 40) = 330
And now, total amount of money collected by all three schools DPS, CVC and KVS
= 7000 + 6125 + 7875 = 21000
11. Given orders of matrices X, Y, Z, W and P are 2 × n, 3 × k, 2 × p, n × 3 and p × k ; respectively.
(i) If XZ is defined, we have
n=2 [ order of X is 2 × n and order of Z is 2 × p]

(ii) Given n = p
then, the order of 7X is 2 × n and the order of 5Z is 2 × P
 Order of 7X – 5Z will be 2 × n
(iii) If PY is defined, then k = 3 [ order of P is p × k and order of Y is 3 × k]

Also, order of PY will be 'p × k'


Also, the order of WY will be 'n × k'
If PY + WY is defined k = 3 and p = n
OR
If p = 3, then the order of matrix (3Y + 10P) is 3 × k
[ order of Y is '3 × k' and order of P is 'p × k']

And now given, order of matrix W is 'n × 3' and order of matrix P is 'p × k'
[If matrix WP is defined ; then p = 3]

E 65

NCERT IMPORTANT QUESTIONS
1
1. Construct a 3 × 4 matrix, whose elements are given by a ij  3i  j . [Ex.3.1, Q.5(i)]
2
2. Which of the given values of x and y make the following pair of matrices equal
3x  7 5  0 y  2  [Ex.3.1, Q.9]
 y  1 2  3x  , 8 4 
1
(A) x  ,y=7 (B) Not possible to find
3
2 1 2
(C) y = 7, x  (D) x ,y
3 3 3
3. The number of all possible matrices of order 3 × 3 with each entry 0 or 1 is: [Ex.3.1, Q.10]
(A) 27 (B) 18 (C) 81 (D) 512
Compute the following: : 
a b a b
4.  [Ex.3.2, Q.2(i)]
 b a   b a 
x z  1 1  3 5
5. Solve the equation for x, y, z and t, if 2   3   3 [Ex.3.2, Q.10]
y t  0 2   4 6 
2 0 1 
6. Find A2 – 5A + 6I, if A =  2 1 3  [Ex.3.2, Q.15]
 1 1 0 
 
 
 0  tan 2 
7. If A =   and I is the identity matrix of order 2,

 tan 0 
 2 
cos   sin  
show that I + A = (I – A)  [Ex.3.2, Q.18]
 sin  cos  
 0 a b
1 1
8. Find (A + A') and (A – A'), when A =  a 0 c  [Ex.3.3, Q.9]
2 2   b c 0 
 
9. If A, B are symmetric matrices of same order, then AB – BA is a [Ex.3.3, Q.11]
(A) Skew symmetric matrix (B) Symmetric matrix
(C) Zero matrix (D) Identity matrix
1 2 0   0 
10. For what values of x : 1 2 1 2 0 1   2  = O ? [Misc.Ex.,Q.4]
1 0 2   x 
  
  
11. If A =   is such that A2 = I, then [Misc.Ex.,Q.9]
  
(A) 1 + 2 +  = 0 (B) 1 – 2 +  = 0 (C) 1 – 2 –  = 0 (D) 1 + 2 –  = 0
1
12. Construct a 3 × 2 matrix whose elements are given by aij = |i – 3j|. [Example 3]
2
 x 5  3 4   7 6 
13. Find the values of x and y from the following equation 2   
 7 y  3 1 2  15 14 
[Example 10]

66 E

 1 2 3
14. If A =  3 2 1  , then show that A3 – 23A – 40I = O [Example 18]
 4 2 1
 
 2 2 4 
15. Express the matrix B =  1 3 4  as the sum of a symmetric and a skew symmetric matrix.
 1 2 3 
 
[Example 22]

ANSWER KEY
 1 1
1 2
0
2
 
5  2a 2b 
1  2.
3
1. (B) 3. (D) 4. 0 2a 
2
2 2  
 7 5 
4 3
 2 2 
 1 –1 –3  0 0 0   0 a b
5. x = 3, y = 6, z = 9, t = 6 6.  –1 –1 –10  8. 0 0 0  ,  –a 0 c 
     
 –5 4 4  0 0 0   –b –c 0 
 5
1 2 
 
 1
9. (A) 10. x = –1 11. (C) 12. 2
2 
 
0 3
 2 
13. x=2,y=9
 3 3  1 5
 2 –
2
– 
2 0 –
2
– 
2
   
3 1
15. Symmetric matrix =  – 3 1  , Skew-symmetric matrix = 0 3 
 2  2 
 3  5 
– 1 –3   –3 0 
 2   2 

E 67

PREVIOUS YEAR QUESTIONS

1 MARK QUESTIONS
1. The number of all possible matrices of order 2 × 3 with each entry 1 or 2 is : [CBSE 2022]
(A) 16 (B) 6 (C) 64 (D 24
Sol. (C) Required number of possible matrices
= (Number of entries)order
= (2)2 × 3 = (2)6 = 64 [1]
2. If a matrix A is both symmetric and skew-symmetric, then A is necessarily a : [CBSE 2022]
(A) Diagonal matrix (B) Zero square matrix (C) Square matrix (D) Identity matrix
T
Sol. (B) Given ; A = A (symmetric matrix)
and –AT = A (skew-symmetric matrix)
 2A = O or A = O [1]
Hence, A is necessarily a zero square matrix.
3c  6 a  d  12 2 
3. If    , then the value of ab – cd is : [CBSE 2022]
 a  d 2  3b   8 4 
(A) 4 (B) 16 (C) –4 (D) –16
3c  6 a  d  12 2 
Sol. (A) Given ;   
 a  d 2  3b   8 4 
On comparing both sides, we get ;
 a – d = 2 and a + d = –8
 2a = –6 or a = –3  d = –5
Also ; 3c + 6 = 12, 2 – 3b = –4  c = 2, b = 2 [1]
Hence ; ab – cd = (–3)2 – 2(–5) = –6 + 10 = 4
 3 4
 1 2 1 
4. For two matrices P =  1 2  and QT =  , P – Q is : [CBSE 2022]
 0 1  1 2 3
 
2 3 4 3 4 3 2 3 
(A)  3 0  (B)  3 0  (C)  0 3 (D) 0 3
 0 3  1 2   1 2  0 3
       
 1 1 
 1 2 1  2 2
Sol. (B) QT =  Q =
 1 2 3  1 3
 

 3 4   1 1  4 3
Hence ; P – Q =  1 2  –  2 2  =  3 0  [1]
 0 1   1 3  1 2 
     

68 E

5. A matrix A = [aij]3 × 3 is defined by :

2i  3j ; i  j

a ij   5 ; ij
3i  2 j ; i  j

The number of elements in A, which are more than 5, is : [CBSE 2022]
(A) 3 (B) 4 (C) 5 (D) 6
Sol. (B) Given ; A = [aij]3× 3

2i  3j ; i  j  5 8 11
  4 5 13
where aij =  5 ; i  j A = [1]
 7 5 5 
3i  2j ; i  j  

Hence ; required number = 4

0 1 1 
6. For the matrix X =  1 0 1  , (X2 – X) is : [CBSE 2022]
1 1 0 
 

(A) 2I (B) 3I (C) I (D) 5I

 0 1 1  0 1 1  2 1 1 
Sol. (A) X2 = 1 0 1  . 1 0 1  = 1 2 1 
 1 1 0  1 1 0  1 1 2 
     

2 1 1  0 1 1   2 0 0 
 X – X = 1 2 1   1 0 1    0 2 0  = 2I
2
[1]
1 1 2   1 1 0   0 0 2 
     

2 1  1 8
7. Find the order of matrix A such that  1 0 A1
 2  [CBSE 2021 C]
3 4  9 22 
 
OR

1 5   0 4
If B =  and A + 2B =  7 5  , find matrix A.
0 3

2 1  1 8
Sol. Let B =  1 0 and C   1
 2 
3 4  9 22 
 

2 1  1 8
We have  1 0 A1 2 
 3 4  32 9 22  32
 

E 69

Let order of A be m × n
BA will be possible if number of columns in matrix B should be equal to number of rows in
matrix A  m = 2.
and order of BA is 3 × n
Since, BA = C,
 Order of BA will be same as that of matrix C
 3×n=3×2  n = 2
Then, order of matrix A is 2 × 2 [1]
OR
 0 4
A + 2B =   (given)
 7 5 
 0 4 1 5  0 4   2 10 
 2B = 
0 4
 A =    2 =  7 5    0 6 
 7 5   7 5  0 3    
 2 14 
 A=   [1]
 7 11

2
8. If A = 1 0 4 and B =  5  ; find AB. [CBSE 2021 C]
 
 6 

2
 5  = 1 2  0  5  4  6 = [2 + 0 + 24] = [26]
Sol. AB = 1 0 413
   11 [1]
 6  31

 0 a 1
9. Given, a skew-symmetric matrix A   1 b 1  , the value of (a + b + c)2 is ....... [CBSE 2020]
 1 c 0 
 
 0 a 1
Sol. Given, A   1 b 1 
 1 c 0 
 

 A is skew symmetric matrix, AT = –A

0 1 1 0 a 1
A = a b c  and  A  1 b 1
T  
1 1 0  1 c 0 

So, a = 1, b = 0 & c = –1
Now (a + b + c)2 = (1 + 0 – 1)2= 0 [1]

70 E

 0 a 3 
10. If the matrix A   2 0 1 is skew symmetric, find the values of 'a' and 'b'. [CBSE 2018]
b 1 0 
 
0 a 3 
Sol. A   2 0 1  (given)
b 1 0 

A is skew-symmetric  AT = –A
 0 2 b   0 a 3 
 a 0 1    2 0 1   a  2, b  3 [1]
 3 1 0    b 1 0 
   

2 MARKS QUESTIONS

0 6  5x 
11. If the matrix A =  2 is symmetric, find the values of x? [CBSE 2021 C]
x x  3 

0 6  5x 
Sol. A =  2 (given)
x x  3 

For symmetric matrix, A = A'


0 6  5x   0 x2 
 x2  …..(1) [1]
 x  3  6  5x x  3

  6 – 5x = x2 [from (1)]
 x2 + 5x – 6 = 0
 (x + 6) (x – 1) = 0
 x = – 6, 1 [1]
3 2 
If A   and I  
1 0
12.  , find scalar k such that A2 + I = KA. [CBSE 2020]
 1 1 0 1 

 3 2  1 0 
Sol. Given, A    and I  
 1 1 0 1 

 3 2   3 2  1 0   3 2 

 1 1  1 1  0 1   K  1 1 [ A2 + I = KA] [½]

 11 8 1 0   3K 2K 
  4 3   0 1    K K  [½]

12 8  3K 2K 


  4 4    K K 

 K = –4 (on comparing both sides) [1]

E 71

 4 2
13. If A    , show that (A – 2I) (A – 3I) = O. [CBSE 2019]
 1 1 

Sol. A  
4 2
(given)
 1 1 

  4 2 1 0   2 2 
A  2I    2  [½]
 1 1  0 1   1 1

and A  3I  
4 2 1 0   1 2
 3  [½]
 1 1  
0 1   1 2 

2  2 4  4  0 0 
(A – 2I) (A – 3I) = 
2 1 2
= 
2
    O [1]

 1 1  1 2   1  1 2  2  0 0 

72 E

CHAPTER-4 : DETERMINANTS
1. DEFINITION:
When an algebraic or numerical expression is expressed in a square form containing some rows
and columns, this square form is named as a determinant of that expression. For example when
a1 b1
expression a1b2 – a2 b1 is expressed in the form then it is called a determinant of order 2
a2 b2

a1 b1 c1
clearly a determinant of order 2 contains 2 rows and 2 columns. Similarly a 2 b2 c 2 is a
a3 b3 c3
determinant of order 3.
Obviously in every determinant, the number of rows and columns are equal and this number is
called the order of that determinant.
Note :(i) For matrix A, |A| is read as determinant of A and not modulus of A.
(ii) Only square matrices have determinant.
(iii) Determinant is a number associated to a square matrix.

2. REPRESENTATION OF A DETERMINANT :
Generally we use  or |A| symbols to express a determinant and a determinant of order 3 is
represented by
a11 a12 a13
a 21 a 22 a 23
a 31 a 32 a 33
It should be noted that the ( i, j)th element (the element of the ith row and jth column) of the
determinant has been expressed by aij , i = 1,2,3 ; j = 1,2,3 . The elements for which i = j are
called diagonal elements and the diagonal containing them is called principal diagonal or simply
diagonal of the determinant. For the above determinant a11 , a22 , a33 are diagonal elements.

3. (i) Determinant of a matrix of order one :


Let A = [a ] be the matrix of order 1, then determinant of A is defined to be equal to a
(ii) Determinant of a matrix of order two :
a a12 
Let A   11 be a matrix of order 2 × 2,
a 21 a 22 
then the determinant of A is defined as:
a11 a12
det (A) = |A| = Δ = = a11 a22 – a21 a12
a21 a22

E 73

(iii) Determinant of a matrix of order three :
(a) Expansion along first row (R1) : The expression which has been expressed in a
determinant form is called the value of that determinant.
a11 a12 a13
Let   a 21 a 22 a 23 be a third order determinant.
a 31 a 32 a 33
To find its value we expand it by any row or column as the sum of three determinants
of order 2. If we expand it by first row then
a 22 a 23 a a a a
  (1)11 a11  (1)1 2 a12 21 23  (1)1 3 a13 21 22
a 32 a 33 a 31 a 33 a 31 a 32

a22 a23 a a23 a a


= a11  a12 21  a13 21 22
a32 a33 a31 a33 a31 a32

= a11a22a33 – a11 a32 a23 – a12 a21 a33 + a12 a31 a23 + a13 a21 a32 – a13 a31 a22 ... (1)
(b) Expansion along first column (C1) :
a22 a23 a a13 a a13
|A| = a11(–1)1+1 + a21(–1)2+1 12 + a31(–1)3+1 12
a32 a33 a32 a33 a22 a23

= a11 a22 a33 – a11 a23 a32 – a21 a12 a33 + a21 a32 a13 + a31 a12 a23 – a31 a13 a22 ... (2)
Values of |A| in (i) and (ii) are same and we can find the value of |A| by expanding
along any row(R1, R2, R3) and column (C1, C2, C3).
Note : For easier calculation we shall expand the determinant along that row or column
which contains maximum number of zeroes.
6 7 8
Illustration 1: Find the value of A = 1 3 1
2 1 4
3 1 1 1 1 3
Solution: A = 6(–1)1+1 + (–7) (–1)1+2 + 8 (–1)1+3
1 4 2 4 2 1
= 6(12 – 1) + 7 (–4 –2) + 8 (1 + 6) = 66 – 42 + 56 = 80
4. AREA OF TRIANGLE :
The area of a triangle whose vertices are (x1,y1), (x2,y2) and (x3,y3) is given by
1
[x1(y2 – y3) + x2(y3 – y1) + x3(y1 – y2)]. Now this expression can be written in the form of a
2
x1 y1 1 x1 x2 x3
1 1
determinant as   x 2 y 2 1 or y1 y2 y3
2 2
x3 y3 1 1 1 1
Note : (i) Since area is a positive quantity, so we always take the absolute value of the
determinant.
(ii) The area of the triangle formed by three collinear points is zero.
(iii) If area is given, then for calculation take both positive and negative value.
74 E

Illustration 2: Find the area of the triangle whose vertices are A(at12 , 2at1 ) , B(at 22 , 2at 2 ) and C(at 32 , 2at 3 )
at12 2at1 1  t12 t1 1 
1 1 
Solution: Area of ABC = at 22 2at 2 1 =  2a 2 t 22 t2 1 
2 2 2
at 3 2at 3 1
 t 32 t 3 1 

1 t1 t12 1 t1 t12
1 2 1 2
= (2a )  1 t 2 t 22 = (2a ) 1 t 2 t 22
2 2
1 t3 t 32 1 t3 t 32
Applying R1 R1 – R3 and R2 R2 – R3
0 t1  t 3 t12  t 32 0 t1  t 3 (t1  t 3 )(t1  t 3 )
2 2
= a 0 t2  t3 t2  t3 = a 0 t2  t3
2 2
(t 2  t 3 )(t 2  t 3 )
1 t3 t 32 1 t3 t 32
Take common (t1 – t3) and (t2 – t3 ) from R1 and R2 respectively
0 1 t1  t 3
2
= a 0 1 t2  t3
1 t3 t 32
Expanding along C1
= a2 | (t1 – t2) (t2 – t3) (t3 – t1)| square units.
5. MINORS :
The determinant that is left by cancelling the row and column intersecting at a particular element
is called the minor of that element. Minor of an element aij is denoted by Mij.
Note : Minor of an element of a determinant of order n (n  2) is a determinant of order (n – 1).
a11 a12 a13
a 22 a 23 a a
Example : If  = a 21 a 22 a 23 then minor of a11 is M11 = , similarly M32 = 11 13
a 32 a 33 a 21 a 23
a 31 a 32 a 33
6. COFACTORS :
The cofactor of an element aij is denoted by Cij and is equal to (–1)i + j. Mij where M is minor of
element aij.
a 22 a 23
Example : C11 = (–1)1 + 1 M11 = M11 = ,
a 32 a33
a11 a13
similarly C32 = (–1)3 + 2 M32 = – M32 = –
a 21 a 23
Note: (i) The sum of products of the elements of any row with their corresponding cofactors is
equal to the value of determinant i.e.  = a11C11 + a12C12 + a13C13
(ii) The sum of the product of elements of any row with corresponding cofactors of
another row is equal to zero i.e. a11C21 + a12C22 + a13C23 = 0
(iii) If order of a determinant () is 'n' then the value of the determinant formed by
replacing every element by its cofactor is n–1

E 75

Illustration 3 : Find the minors and the cofactors of each entry of the third row of the determinant A
and hence evaluate det A.

6 7 8
A = 1 3 1
2 1 4

–7 8 6 8
Solution: Now M31 = = –7 – (–24) = 17, M32 = = 6 – 8 = –2 and
–3 1 1 1

6 7
M33 = = 6 × (–3) – (–7) × 1 = –11.
1 3

Also, C31 = (–1)3+1 M31 = M31 = 17, C32 = (–1)3 + 2 M32 = – M32 = 2
and C33 = (–1)3+3 M33 = M33 = –11.
Further, det A = a31C31 + a32C32 + a33C33 (Expansion with the help of third row)
= 2 × 17 + 1 × 2 + (–4) × (–11) = 80
7. ADJOINT OF A MATRIX
If every element of a square matrix A be replaced by its cofactor in [A], then the transpose of the
matrix so obtained is called the adjoint of A and it is denoted by adj A
Thus if A = [aij] be a square matrix and Aij be the cofactor of aij in |A|, then adj A = [Aij]'
 a11 a12 .... a1n   A11 A12 .... A1n  '  A11 A 21 .... A n1 
a a 22 
.... a 2n  A A 22 .... A 2 n   A12 A 22 .... A n 2 
Hence ifA =  21 , then adj A =  21 
 .... .... .... ....   .... .... .... ....   .... .... .... .... 
     
a n1 a n 2 .... a nn   A n1 An2 .... A nn   A1n A2 n .... A nn 
Properties of Adjoint Matrix :
If A, B are square matrices of order n and n is corresponding unit matrix, then
(i) A (adj A) = |A| n = (adj A) A. (Thus A (adj A) is always a scalar matrix)
(ii) |adj A| = |A|n–1
(iii) |AB| = |A| |B|
(iv) A is singular  |A| = 0
(v) A is non-singular |A|  0
a a12 
Remark: For a square matrix of order 2, given by A =  11
a 21 a 22 
The adj A can also be obtained by interchanging a11 and a22 and by changing sign of a12 and a21.
Example :
a11 a12 a22 –a12
adj A = =
a21 a22 –a21 a11
Change sign Interchange elements

76 E

 4 6 1 
Illustration 4 : Find the adjoint of the matrix A =  1 1 1  and verify that
 4 11 1
A( adj A) = (adj A) A = | A | I3.
4 6 1
Solution: Here, |A| = 1 1 1 = 4(1 –11) – (–6) (1 + 4) + 1 (–11 –4) = – 40 + 30 – 15 = –25
4 11 1
Cofactors of the elements of first row are :
1 1 –1 1 1 1
C11  (–1)11 = – 10, C12 = (–1)1+2 = – 5, C13  (–1)13 = –15
11 1 –4 -1 4 11
Cofactors of the elements of second row are :
6 1 4 1 4 6
C21  (–1)2 1 = 5, C22  (1)22 = 0, C23  (–1)2 3 = –20
11 1 4 1 4 11
Cofactors of the elements of third row are :
6 1 4 1 4 6
C31  (1)31 = –5, C32  (1)3 2 = –5, C 33  ( 1)3 3 = –10
1 1 1 1  1 1
T
 10 5 15   10 5 5 
 adj A =  5 0 20    5 0 5 
 5 5 10   15 20 10 
   
4 6 1  10 5 5 
Verification : A (adj A) =  1 1 1  5 0 5 
4 11 1  15 20 10 
   
 40  30  15 20  0  20 20  30  10 
  10  5  15 5  0  20 5  5  10 
 40  55  15 20  0  20 20  55  10 

 –25 0 0  1 0 0 
 0 25 0  = –25
  0 1 0  = (–25) I3 = | A |I3
 0 0 0 1 
 0 25  
Similarly, we can show that (adj A) A = | A | I3
Hence (adj A) = | A | I3 = A(adj A).

8. INVERSE MATRIX (For non-singular matrix) :


If A is square matrix of order m and if there exists another square matrix B of the same order m,
such that AB = BA = I, then B is called the inverse matrix of A and it is denoted by A–1 and A is
called invertible matrix.
Also A adj A = |A| I = (adj. A) A
adj(A) adj(A) adj(A)
 A I A A–1 =
|A| |A| |A|
Thus A–1 exists  |A|  0.

E 77

Note : (1) Matrix A is called invertible if A–1 exists.
(2) A rectangular matrix does not have inverse matrix because BA and AB should be
defined and be equal. (So A and B should be square matrices of the same order)
(3) If B is the inverse of A, then A is also the inverse of B.
Properties of Inverse Matrix
(i) (A–1)–1 = A
(ii) (AB)–1 = B–1A–1
1
(iii) |A–1| = = |A|–1
|A|
2 1 4 5
Illustration 5 : If A    and B1    , compute (AB)–1.
 5 3 3 4
 4 5  –1
Solution: By reversal law, we have, (AB)–1 = B–1 A–1 =   A
3 4
In this case, we have to compute A–1 only
2 1
Here, det A = = 6 – 5 = 1 0, thus A–1 exists
5 3
Now, C11  3, C12  5, C21  1, C 22  2
T
1
–1 1  3 5  3 1
  A = (adjA)     
|A| 1  1 2   5 2 
 4 5   3 1 12  25 4  10  13 6 
Hence, (AB)–1 =     
 3 4   5 2   9  20 3  8   11 5 
9. APPLICATION OF DETERMINANTS AND MATRICES :
Consistent system : A system of equations is said to be consistent if its solution (one or more) exists
Inconsistent system : A system of equations is said to be inconsistent if its solution does not exists.
Solution of system of linear equations using inverse of a matrix (Matrix method) :
Consider the system of equations.
a1x + b1y + c1z = d1, a2x + b2y + c2z = d2, a3x + b3y + c3z = d3
 a1 b1 c1  x   d1 
Let A = a 2 b2 c 2 , X = y and B =  d 2  , then the system of equations can be written as
  
  z   
a3 b3 c3    d3 
AX = B.
 a1 b1 c1   x   d1 
a 2 b2 c2   y  = d 2 
   z  d 
a3 b3 c3     3

78 E

Case–I : If A is non-singular matrix (|A| 0) , then its inverse exists. Now AX = B
A–1 (AX) = A–1B  I X = A–1B
or X = A–1B ( AA–1 = I)
This matrix equations provides unique solution and the system of equations is called
consistent.
Case–II : If A is a singular matrix, then |A| = 0 then we calculate (adj A) B.
If (adj A) B  O [O, is zero matrix] then solution does not exist and the system of
equations is called inconsistent.
If (adj A) B = O, then system may be either consistent or inconsistent according as
the system have either infinitely many solution or no solution.
Illustration 6 : Solve using matrix method, for x, y and z :
2x – y – z = 7
3x + y – z = 7
x + y – z = 3.
Solution: The given system of equations can be written as AX = B …..(1)
2 1 1 x   7
 
where A   3 1 1 , X   y  and B   7 .
 
1 1 1  z  3 
2 1 1
Here, det A  3 1 1 = 2(–1 +1) – (–1) (–3 + 1) + (–1) (3 – 1) = –2 – 2 = –4  0
1 1 1
Therefore, the given system is consistent and has a unique solution given by X = A–1 B. [from (1)]
To find A–1, we have to find co-factors of each elements.
A11 = 0, A12 = 2, A13 = 2, A21 = –2, A22 = –1, A23 = –3, A31 = 2, A32 = –1 and A33 = 5
T
0 2 2 0 2 2 
1 1  1
  A 1  (adj A)   2 1 3   2 1 1

det A 4 4
 2 1 5   2 3 5 

0 2 2   7 
1
Then from (i), X = A–1B =   2 1 1  7 
4
 2 3 5   3 

x   0  14  6   8  2 
   y    1  14  7  3    1  4    1 x = 2, y = –1, z = –2
  4  4   
 z  14  21  15  8   2 

E 79

EXERCISE-I
 1 3
1. If A    , then the value of |A2 – 2A| is
 2 1
(A) 25 (B) –25 (C) 0 (D) 5

1 0 1 
2. If A  0 1 2  then |3A| is equal to :-
0 0 4 

(A) 9 |A| (B) 81 |A| (C) 27 |A| (D) 31 |A|


3. If A and B are square matrices each of order 3 and |A| = 5, |B| = 3, then the value of |3AB| is
(A)  (B)  (C)  (D) 
4. A triangle whose area is 3 sq. units and its vertices are (1, 3), (0, 0) and (k, 0) then the value of k is
(A)  2 (B)  1 (C)  4 (D)  8

5. If for any 2 × 2 square matrix A, A(adjA) =  8 0  , then the value of |A| is .


 0 8 

1 1
(A) 8 (B) 64 (C) (D)
64 8
6. If A is a skew-symmetric matrix of order 3, then prove that det A is
(A) 1 (B) –1 (C) 2 (D) 0

0 1 
 0 1 2   1 0  and M = AB, then M–1 is equal to-
7. If A =  , B =  
 2 2 0   1 1 

 2 2   1 / 3 1 / 3 1 / 3 1 / 3  1 / 3 1 / 3
(A)   (B)   (C)   (D)  
2 1   1 / 3 1 / 6  1 / 3 1 / 6   1 / 3 1 / 6 

ASSERTION-REASON BASED QUESTIONS


In the following questions, a statement of Assertion (A) is followed by a statement of
Reason (R). Choose the correct answer out of the following choices.
(A) Both A and R are true and R is the correct explanation of A.
(B) Both A and R are true but R is not the correct explanation of A.
(C) A is true but R is false.
(D) A is false but R is true.

80 E

8. Assertion (A) : If every element of a third order determinant of value is multiplied by 5,
then the value of the new determinant is 125.
Reason (R) : If k is a scalar and A is an n × n matrix, then |kA| = kn |A|

1 3   2 
9. Assertion (A) : If the matrix A  2 4 8  is singular, then  = 4.
 3 5 10 
 
Reason (R) : If A is a singular matrix, then |A| = 0
10. Assertion (A) : If A = [aij]2n where A' = – A, then |A| is a perfect square.

Reason (R) : The determinant of skew-symmetric matrix of odd order is equal to zero.

ANSWER KEY
Q. No. 1 2 3 4 5 6 7 8 9 10
Ans. A C A A A D C A A D
E 81

SOLUTIONS
 1 3  1 3   7 6  2 6 
1. (A) A2  A.A        ,2A   
2 1 2 1  4 7  4 2 

 7 6  2 6   5 0 
Now, A 2  2A     
 4 7   4 2  0 5 

5 0
A 2  2A   25
0 5

2. (C) |3A| = 33. |A| = 27 |A| [ |kA| = kn|A|]

3. (A) We know that |KA| = Kn|A|


Where n is the order of the matrix.
 |3AB| = 33 |A| |B| ( order given is 3)

 |3AB| = 33 × |A| |B|


= 27 × 5 × 3
= 27 × 15
= 405

1 3 1
1
4. (A) Area of triangle  0 0 1   3
2
k 0 1

1 (0 – 0) –3 (0 – k) + 1 (0 – 0) =  6 3k =  6  k =  2

5. (A) Given A(adjA)   8 0 


0 8 
We know,
A(adjA) = |A| I …..(1)
1 0 
A(adjA) = 8  = 8I …..(2)
0 1 
From equation (1) and (2)
|A| = 8
6. (D) By definition of skew-symmetric matrix A' = –A
 | A' | = |–A |
 |A| = (–1)3 |A| = – |A| [ |A'| = |A| and |kA| = kn|A|, where is order of matrix]
 2|A| = 0 |A| = 0

82 E

0 1 
 0 1 2  1 0 
7. (C) Given A =  , B =  
 2 2 0   1 1 

0 1 
 0 1 2    =  1 2
M = AB =    1 0   2 2 
 2 2 0  
 1 1 
 2 2 
|M| = 6 , adj (M) =  
2 1 
 1 2 2  1 / 3 – 1 / 3
 M–1 = · adj(M)   
|M| 6 2 1  1 / 3 1 / 6 

8. (A) Assertion : Using above property |5| = 53 = 125 


Hence A is true.
Reason : If k is a scalar and A is an n × n matrix, then |kA| = kn |A|.
This is a property of the determinant. Hence R is true.
Both A and R are true and R is the correct explanation of A.

1 3 2
9. (A) Assertion : For any singular matrix A, |A| = 0  2 4 8 0
3 5 10

 1(40 – 40) – 3(20 – 24) + ( + 2) (10 – 12) = 0 0 + 12 – 2 – 4 = 0  = 4


Hence A is true.
Reason : A matrix is said to be singular if |A| = 0
Hence R is true.
Both A and R are true and R is the correct explanation of A.
10. (D) Assertion : Let us assume a skew symmetric matrix of order 2 as follows
 0 a 
 A where a 
 a 0 
Now, let us find the determinant of the above matrix then we get
0 a
 |A| |A| = 0 – (–a2) = a2
a 0

Here, we can see that the determinant of matrix is a2.


Here, we can say that determinant of matrix is perfect square if and only if a   {0} .
We know that the value of a is a real number that is a  .

E 83

Here, we can see that there are some values of a for which the value a2 is not perfect square
for example a  3 .
Therefore we can conclude that the assertion is wrong.
Reason : Let us assume that the matrix A of odd order as n.
We know that the condition for skew-symmetric matrix given as A' = –A |A'| = |–A|
We know that the formulas of determinant of transpose and determinant of scalar multiple
of a matrix given as |A'| = |A| and |kA| = kn|A| |A| = (–1)n |A| [ |kA| = kn|A|]
We know that n is odd such that (–1)n = 1
By using this condition to above equation we get
 |A| = |–A| 2|A| = 0 |A| = 0
Here, we can see that the determinant of skew-symmetric matrix of odd order is 0.
Therefore we can conclude that the reason is correct.
Hence, the assertion is wrong and the reason is correct.

84 E

EXERCISE-II

Let A = 
3 7
and B = 
6 8
1.   . Verify that (AB)–1 = B–1 A–1.
 2 5   7 9 

 x  1 3 4 
2. 
Determine the values of x for which the matrix A   5 x  2 2  is singular.
 4 1 x  6 

3. If A(x1, y1), B(x2, y2) and C(x3, y3) are vertices of an equilateral triangle whose each side is equal
2
x1 y1 2
to a, then prove that x 2 y2 2  3a 4 .
x3 y3 2

cos   sin  0 
4. If A   sin  cos  0  , find adj A and verify that A (adj A) = (adj A) A = |A| I3.
 0 0 1 

5. Use matrix method to examine the following system of equations for consistency or
inconsistency :
4x – 2y = 3, 6x – 3y = 5

2 3
6. Show that A    satisfies the equation x2 – 6x + 17 = 0. Hence, find A–1.
3 4 

1 2 –3
7. If A =  2 3 2  then, find A–1 and solve the following system of equations using it :
 3 –3 –4 

x + 2y – 3z = –4, 2x + 3y + 2z = 2, 3x – 3y – 4z = 11

8. The management committee of a residential colony decided to award some of its members
(say x) for honesty, some (say y) for helping others and some others (say z) for supervising the
workers so keep the colony neat and clean. The sum of all the awardees is 12. Three times the
sum of awardees for cooperation and supervision added to two times the number of awardees for
honesty is 33. If the sum of the number of awardees for honesty and supervision is twice the
number of awardees for helping others, using matrix method, find the number of awardees of
each category.

E 85

CASE STUDY-I
9. Read the following passage and answer the questions given below:
Two schools A and B want to award their selected students on the values of sincerity,
truthfulness and helpfulness. The school A wants to award Rs. x each, Rs. y each and Rs. z each
for the three respective values to 3, 2 and 1 students respectively with a total award money of
Rs. 1,600 school B wants to spend Rs. 2,300 to award its 4, 1 and 3 students on the respective
values (by giving the same award money to the three values as before). If the total amount of
award for one prize on each value is Rs. 900.

(i) Express the given information in matrix multiplication form as AX = B.


(ii) Check that inverse of matrix A is exist or not.
(iii) Find adjoint of matrix A.
OR
Find the award money for each value.

CASE STUDY-II
10. Read the following passage and answer the questions given below:
Area of a triangle whose vertices are (x1, y1),(x2, y2) and (x3, y3) is given by the determinant :

x1 y1 1
1
  x2 y2 1
2
x3 y3 1

Since, area is a positive quantity, so we always take the absolute value of the determinant .
Also, the area of the triangle formed by three collinear points is zero.
(i) Find the area of the triangle whose vertices are (–2, 6), (3, –6) and (1, 5) :
(ii) If the points (2, –3) (k, –1) and (0, 4) are collinear, then find the value of 4k :
(iii) If the area of a triangle ABC ; with vertices A(1, 3), B(0, 0) and C(k, 0) is 3 sq. units ; then
find the value of k .

OR
If A = (11, 7) ; B = (5, 5) and C = (–1, 3) then show that A, B and C are collinear.

86 E

SOLUTIONS

Given; A = 
3 7
, B = 
6 8
1. 
2 5   7 9 

AB = 
3 7  6 8   67 87 
= , |AB| = 67 × 61 – 87 × 47 = – 2
 5   7 9   47 61
2

1 –1  61 –87
(AB)–1 = · adj(AB) = …..(1)
| AB | 2  –47 67 

Now, B–1 =
1 1  9 8 –1  9 –8
· adj(B)  
| B| 6  9 – 8  7  –7 6  2  –7 6 
1 1  5 –7  5 –7 
A–1 = · adj(A)  
|A| 3  5 – 2  7  –2 3   –2 3 
–1  9 –8  5 –7  –1  61 –87
B–1A–1 =  –7 6   –2 3  = 2  –47 67  …..(2)
2

From (1) and (2)


(AB)–1 = B–1A–1 Hence proved

 x  1 –3 4 
2. 
Given, A = –5 x  2 2 
 4 1 x – 6 

A is singular matrix if |A| = 0

x  1 –3 4
 –5 x  2 2 =0
4 1 x–6

(x + 1) [(x + 2) (x – 6) – 2] + 3 (–5x + 30 – 8) + 4 (–5 –4x –8) = 0

 (x + 1) (x2 – 4x – 14) + 3 (–5x + 22) + 4 (–4x – 13) = 0

 x3 – 3x2 – 49x = 0 or x(x2 – 3x – 49) = 0

1
Solving we get x = 0, (3 205 )
2

3. Let  be the area of triangle ABC. Then,


x y1 1 x1 y1 1
1 1
 x2 y 2 1  2   x 2 y2 1
2x y3 1 x3 y3 1
3

2
x1 y1 1 x1 y1 2 x1 y1 2
 4  2 x 2 y2 1  x2 y2 2  16  x 2
2
y2 2 …..(1)
x3 y3 1 x3 y3 2 x3 y3 2
E 87

3 2
But, the area of an equilateral triangle with each side equal to a is a .
4
3 2
  a  162 = 3a4 …..(2)
4
2
x1 y1 2
From (1) and (2), we obtain x 2 y2 2  3a 4
x3 y3 2

 cos  – sin  0 
4. Given A =  sin  cos  0 
 0 0 1 

|A| = cos  (cos – 0) + sin  (sin  – 0)

|A| = cos2  + sin2  = 1

Now, A11 = cos , A12 = –sin, A13 = 0

A21 = sin , A22 = cos  , A23 = 0

A31 = 0, A32 = 0, A31 = 1

 cos  sin  0 
 adj A =  – sin  cos  0 
 0 0 1 

cos  – sin  0   cos  sin  0 


Now, A(adj A) =  sin  cos  0  – sin  cos  0 
 0 0 1   0 0 1 

1 0 0 
= 0 1 0  = I3 = |A| · I3 [since |A| = 1]
0 0 1 

 cos  sin  0  cos  – sin  0 


( adj A)A =  – sin  cos  0   sin  cos  0 
 0 0 1   0 0 1 

1 0 0 
= 0 1 0  = I3 = |A| · I3 (since |A| = 1)
0 0 1 
5. The given system of equations can be written as
 4 2  x  3 
AX = B, where A    , X    and B   
 6 3  y
  5
4 2
Now, | A |  = –12 + 12 = 0
6 3

88 E

So, the given system of equations is inconsistent or it has infinitely many solutions according as
(adj A) B  O or, (adj A) B = O respectively.
Let Cij be the co-factors of elements aij in A = [aij]. Then,
C11 = –3, C12 = – 6, C21 = 2 and C22 = 4

  3 6    3 2 
adjA   
 2 4   6 4 

 3 2  3  9  10  1 
So, (adjA)B     O
 6 4  5  18  20  2 
Hence, the given system of equations is inconsistent.

Given; A = 
2 –3
6.
 3 4 

A2 = 
2 –3 2 –3  –5 –18

3 4  3 4  18 7 
 –5 –18  2 –3 1 0  0 0 
 A2 – 6A + 17 I =   – 6   17   =   =O
18 7  3 4  0 1  0 0 
 A2 – 6A + 17I = O or A2 – 6A = –17I
A.A.A–1 – 6 A.A–1 = –17 I.A–1 (Both sides on post multiplying by A–1)
A – 6I = – 17A–1 ( AA–1 = I)

–1 –1  2 –3  6 0   –1  –4 –3  1  4 3
 A–1 = (A – 6I) = – = =
17 17   3   
4  0 6   17  3 –2  17  –3 2 

 1 2 3 
7. Given; A =  2 3 2 
 3 3 4 
 
|A| = 1(–12 + 6) – 2(–8 – 6) – 3 (–6 – 9) = –6 + 28 + 45 = 67
 6 17 13 
adj A =  14 5 8 
 15 9 1
 
 6 17 13 
1 1
 A–1 = adj A   14 5 8
|A| 67  15 9 1
 
also given,
x + 2y – 3z = –4
2x + 3y + 2z = 2
3x – 3y – 4y = 11
by using matrix method
AX = B

E 89

 1 2 3  x   4 
where A =  2 3 2  , X = y  , B = 2
 3 3 4  z   
     11 

we know X = A–1 B
x   6 17 13   4   24  34  143   201 
 y  = 1  14 5 8  2  = 1  56  10  88 = 1 134 
 z  67  15 9 1 11  67  60  18  11  67  67 
        
x = 3, y = –2, z = 1

8. x = awarded members for honesty


y = awarded members for helping (cooperation)
z = awarded members for supervision.
Sum of all the awarders is 12
so x + y + z = 12 .....(1)
Three times the sum of awardees for y and z.
added to two times the x is 33
3(y + z) + 2x = 33
2x + 3y + 3z = 33 .....(2)
The sum of number of for x and z is twice the y
x + z = 2y
x – 2y + z = 0 .....(3)
above all three equation can be written as matrix form
1 1 1  x  12 
2 3 3  y   33
    
1 2 1 z  0 
1
Let A X = B X = A–1.B X =  adjA  .B
|A|
|A| = 1(3 + 6) –1(2 – 3) + 1(–4 – 3) = 9 + 1 – 7 = 3
adjA = [Cij]T
T
 9 1 7   9 3 0 

  3 0 3    1
 0 1
 0 1 1   7 3 1 

 9 3 0  12  108 99 0 


1 1
X   1 0 1 33   12 0 0 
  
3 3
 7 3 1  0   84 99 0 
 x  3 
 y    4  x = 3, y = 4, z = 5
   
z  5 

90 E

9. (i) Given Rs. x is award money for the value of sincerity, Rs. y for truthfulness and Rs. z for
helpfulness.
According to statement
3x + 2y + z = 1600
4x + y + 3z = 2300
x + y + z = 900
We can represent the given equation in matrix multiplication as
 3 2 1   x  1600 
 4 1 3  y   2300 
1 1 1   z   900 
    
AX =B
3 2 1
(ii) Let A =  4 1 3 
1 1 1 
 
|A| = 3(–2) – 2(1) + 1(3) = –5
|A|  0 therefore A–1 is exist.
(iii) adjA = [Cij]'
Now co-factors of matrix A are
C11 = – 2, C12 = – 1, C13 = 3, C21 = – 1, C22 = 2, C23 = –1, C31 = 5, C32 = – 5 C33 = –5

 2 1 3   2 1 5 
 adjA =  1 2 1   1 2 5 
   
 5 5 5  3 1 5 
OR
x 
The award money for each value i.e. X   y 
 z 

X = A–1.B
1
  A1   adjA 
A

 2 1 5  1600 
1
 X =  1 2 5 2300 
5  3 1 5  900 
  
 x   200 
 y    300   x = 200, y = 300, z = 400
 z   400 
   

E 91

10. (i) Required area is given as :
2 6 1
1 1
A  3 6 1 =  2(6  5)  6(3  1)  1(15  6)
2 2
1 5 1

1
 A =
2
 22  12  21 = 15.5 sq.units
(ii) Since the given points are collinear, area of triangle formed by them must be zero.
2 3 1
1
 k 1 1  0
2
0 4 1

or (–1 – 4) + 3(k – 0) + 1 (4k – 0) = 0


 –10 + 3k + 4k = 0
10 10 40
or k= 4k = 4  
7 7 7
(iii) As per given conditions, we have :
1 3 1
1 1
0 0 1  3  1(0  0)  3(0  k)  1(0  0)  3
2 2
k 0 1

or 3k = ± 6 k = ± 2 or k = 2 (as per question)


OR
Given points A, B and C are (11, 7), (5, 5) and (–1, 3) respectively. Hence, Area of ABC
11 7 1
1 1 1
= 5 5 1 = 11(5  3)  7(5  1)  1(15  5)  =  22  42  20 
2 2 2
1 3 1

Hence, Area of ABC = 0 sq. units

   Points A, B and C are collinear.

92 E

NCERT IMPORTANT QUESTIONS
1. Show that points A (a, b + c), B (b, c + a), C (c, a + b) are collinear. [Ex.4.2, Q.2]

2. Find values of k if area of triangle is 4 sq. units and vertices are

(i) (k, 0), (4, 0), (0, 2) (ii) (–2, 0), (0, 4), (0, k) [Ex.4.2, Q.3]

a11 a12 a13


3. If  = a 21 a 22 a 23 and Aij is Cofactors of aij, then value of  is given by [Ex.4.3, Q.5]
a 31 a 32 a 33

(A) a11 A31 + a12 A32 + a13 A33 (B) a11 A11+ a12 A21 + a13 A31

(C) a21 A11+ a22 A12 + a23 A13 (D) a11 A11+ a21 A21 + a31 A31

3 7 6 8 
. Verify that  AB  B1A1 .
1
4. Let A    and B    [Ex.4.4, Q.12]
2 5   7 9

For the matrix A  


3 2
5.  , find the numbers 'a' and 'b' such that A2  aA  bI  O . [Ex.4.4, Q.14]
1 1 

 2 1 1
6. If A  1 2 1 , verify that A3  6A2  9A  4I  O and hence find A1 . [Ex.4.4, Q.16]
 1 1 2 
 
7. Let A be a non-singular square matrix of order 3  3 , then adj A is equal to ? [Ex.4.4, Q.17]

(A) |A| (B) |A|2 (C) |A|3 (D) 3|A|


8. If A is an invertible matrix of order 2, then det (A–1) is equal to ? [Ex.4.5, Q.18]
1
(A) det  A  (B) (C) 1 (D) zero
det  A 

9. Solve the following system of linear equations, using matrix method


2x  3y  3z  5, x  2y  z  4, 3x  y  2z  3. [Ex.4.5, Q.13]

 2 3 5 
10. If A   3 2 4  , find A1 . Using A1 , solve the system of equations 2x  3y  5z  11,
 1 1 2 
 
3x  2y  4z  5, x  y  2z  3 . [Ex.4.5, Q.15]
11. The cost of 4 kg onion, 3 kg wheat and 2 kg rice is ` 60. The cost of 2 kg onion, 4 kg wheat and
6 kg rice is `90. The cost of 6 kg onion, 2kg wheat and 3 kg rice is ` 70. Find cost of each item
per kg by matrix method. [Ex.4.5, Q.16]

E 93

 3 1 1  1 2 2 
12. If A  15 6 5 and B   1 3 0  , find (A)–1.

1  [Misc. Ex., Q.3]
 5 2 2   0 2 1
  

 1 sin  1 
13. 
Let A =   sin  1 sin  , where 0  2. Then [Misc. Ex., Q.9]
 1  sin  1 

(A) Det (A) = 0 (B) Det (A)  (2, )

(C) Det (A)  (2, 4) (D) Det (A)  [2, 4]

3 x 3 2
14. Find values of x for which  . [Example 5]
x 1 4 1

 1 1 2   2 0 1
15. Use product 0 2 3  9 2 3 to solve the system of equations
 3 2 4   6 1 2 
  

x  y  2z  1
2y  3z  1 [Example 19]
3x  2y  4z  2

ANSWER KEY

2. (i) k = 0 , 8 ; (ii) k = 0, 8 3. (D)

 3 1 1
1
5. a = – 4, b = 1 6. 1 3 1
4 1 1 3
 

7. (B) 8. (B)

 0 1 2 
9. x = 1, y = 2, z = –1 10.  2 9 23 , x = 1, y = 2, z = 3
 1 5 13
 

11. Cost of onions per kg = Rs.5; Cost of wheat per kg = Rs.8; Cost of rice per kg = Rs.8

 9 3 5 
12.  2 1 0 13. (D)
 1 0 2
 

14. x  2 2 15. x = 0, y = 5 and z = 3

94 E

PREVIOUS YEAR QUESTIONS
1 MARK QUESTIONS
1. Three points P(2x, x + 3), Q(0, x) and R(x + 3, x + 6) are collinear, then x is equal to :
[CBSE 2022]
(A) 0 (B) 2 (C) 3 (D) 1
Sol. (D) As per the given condition ;
2x x  3 1
0 x 1 0  ar  PQR   0 
x3 x6 1
2x (x – x – 6) – 0(x + 3 – x – 6) + (x + 3) {x + 3 – x} = 0
or –12x + 3x + 9 = 0 –9x = –9  x = 1 [1]
1 1 2 
2. If Cij denotes the cofactor of element pij of the matrix P = 0 2 3 , then the value of

 3 2 4 
C31.C23 is : [CBSE 2022]
(A) 5 (B) 24 (C) –24 (D) –5
1 1 2 
 
Sol. (A) P = 0 2 3 (given)
 3 2 4 
 C31.C23 = (3 – 4).{ (–1) (2 + 3)} = 5 [1]
3. The system of linear equations
5x + ky = 5,
3x + 3y = 5
will be consistent if : [CBSE 2022]
(A) k  –3 (B) k = –5 (C) k = 5 (D) k  5
Sol. (D) The system of linear equations is given as :
5x + ky = 5
and 3x + 3y = 5
It can be written in matrix form as :
 5 k   x   5
3 3   y    5
    
or A X = B
So ; given system of linear equations is consistent if |A|  0
5 k
 0
3 3
  15 – 3k  0 k  5 [1]
E 95

  2 
4. If, for the matrix A =   , |A3| = 125; then the value of  is : [CBSE 2022]
 2  
(A) ±3 (B) –3 (C) ±1 (D) 1
Sol. (A) |A| = 2 – 4
Given ;|A3| = 125  (|A|)3 = 125 [ |An| = |A|n]

 (2 –4)3 = 125  (2 – 4) = 5 2 = 9 or  = ±3 [1]


1 1 2 
5. Let matrix X = [xij] is given by X =  3 4 5 . Then, the matrix Y = [mij], where mij = Minor
 
2 1 3 

of xij, is : [CBSE 2022]


 7 5 3  7 19 11  7 19 11  7 19 11
(A)  19 1 11 (B)  5 1 1  (C)  3 11 7  (D)  1 1 1 
      
 11 1 7  3 11 7   5 1 1   3 11 7 

1 1 2 
Sol. (D) X =  3 4 5
 
2 1 3 

Now; Y = [mij]
 m11 m12 m13   7 19 11
 Y = m 21 m 22 m 23  Y =  1 1 1  [1]
 
 m 31 m 32 m 33   3 11 7 

x 2 3
6. If x = –4 is a root of 1 x 1 = 0, then the sum of the other two roots is : [CBSE 2022]
3 2 x
(A) 4 (B) –3 (C) 2 (D) 5
x 2 3
Sol. (A) 1 x 1  0 (given)
3 2 x

 x (x2 – 2) – 2( x – 3) + 3(2 – 3x) = 0


 x3 – 2x – 2x + 6 + 6 – 9x = 0
 x3 – 13x + 12 = 0
 (x + 4) (x2 – 4x + 3) = 0 [ x = –4 is a root]

 (x + 4) (x – 1) (x – 3) = 0 [1]
Hence ; the sum of other two roots = 1 + 3 = 4

96 E

2 0 0 
7. The inverse of the matrix X = 0 3 0  is : [CBSE 2022]
 
0 0 4 
1 / 2 0 0  1 0 0  2 0 0  1 / 2 0 0 
  1  1 
(A) 24 0 1 / 3 0 (B) 0 1 0  (C) 0 3 0  
(D) 0 1 / 3 0 
  24  24   
 0 0 1 / 4  0 0 1  0 0 4   0 0 1 / 4 
2 0 0 
 
Sol. (D) Given ; X = 0 3 0 
0 0 4 
|X| = 2(12) – 0 + 0 = 24  0  X–1 exists
12 0 0 
  1
Now, adj X =  0 8 0  X–1 = . adjX
X
 0 0 6 
12 0 0  1 / 2 0 0 
1 
 X = –1

 
0 8 0    0 1 / 3 0  [1]
24
 0 0 6   0 0 1 / 4 
 2 0 0
8. If A is square matrix of order 3 such that A(adj A) =  0 2 0  , then find |A|. [CBSE 2021 C]
0 0 2 

 2 0 0
Sol. Given, A(adj A) =  0 2 0 
0 0 2 

We know that A(adj A) = |A|I
1 0 0 
Now, A(adj A) = –2 0 1 0  = – 2I = |A| I
0 0 1 
 
 |A| = – 2 [1]
2
9. If A is a non-singular square matrix of order 3 such that A = 3A, then value of |A| is? [CBSE 2020]
(A) –3 (B) 3 (C) 9 (D) 27
2
Sol. (D) Given, A = 3A, |A| 0, order of A is 3
 |A2| = |3A|
 |A|2 = 33|A| ( |A2| = |A|2 & |KA| = Kn|A| )
or |A| = 27
10. If A is a square matrix satisfying A'A = I, write the value of |A|. [CBSE 2019]
Sol. A'A = I (given)
 |A'A| = |I|
2
 A' A  I  A  1 ( A' = A) [½]
 A  1 or A  –1 [½]

E 97

2 MARKS QUESTIONS

 2 3 
11. Given A    , compute A–1 and show that 2A–1 = 9I – A. [CBSE 2018]
 4 7 
 2 3 
Sol. Given, A   
 4 7 
|A| = 14 – (12) = 2  0
Hence, A is invertible.
7 3
adjA    [½]
4 2
1
Hence, A1  . adj.(A)
A
1 7 3 7 3
A–1  or 2A–1 = …..(1) [½]
2  4 2  4 2
 
Now, R.H.S. = 9I – A
1 0   2 3  9 0   2 3  7 3 
 9          = 2A–1 = L.H.S. [from (1)] [1]
0 1   4 7  0 9   4 7   4 2 
Hence, proved

5 MARKS QUESTIONS

3 4 2
12. If A = 0 2 3 , find A–1. Hence, solve the following system of equations :

 
1 2 6 

3x + 4y + 2z = 8
0x + 2y – 3z = 3
x – 2y + 6z = –2 [CBSE 2021 C]
OR
 3 1 1  1 2 2 
If A = 15 6 5  and B =
–1  1 3 0  , find (AB) .
–1
  
 5 2 2   0 2 1 

3 4 2
Sol. Given, A = 0 2 3

 
1 2 6 

 |A| = 3(12 – 6) – 4(0 + 3) + 2(0 – 2) = 18 – 12 – 4 = 2  0 [½]


Hence, A–1 exists.

98 E

Now, co-factors are given as:
C11 = 6, C12 = –3, C13 = – 2,
C21 = – 28, C22 = 16, C23 = 10,
C31 = – 16, C32 = 9, C33 = 6 [1]

 6 3 2 
T
 6 28 16 
 
Hence, adj A =  28 16 10  =  3 16
 9  [½]
 16 9 6   2 10 6 

 6 28 16 
1 1
9 
–1
A = (adjA) = 3 16 …..(1) [1]
A 2 
 2 10 6 

The given system of linear equations is given as :


3x + 4y + 2z = 8
0x + 2y – 3z = 3
x – 2y + 6z = –2
This system is written in matrix form as :
3 4 2  x   8
 0 2 3  y    3  [½]
    
1 2 6   z   2

 AX = B  X = A–1 . B ……(2) [1]


 6 28 16  8   48  84  32 
1 1
 X= 3 16 9  . 3  =  24  48  18  [from (1) and (2)] [½]
2  2 10 6   2  2  16  30  12 

x   4 
1
  y    6   x = –2, y = 3, z = 1 [1]
z  2  2 
   
OR
 3 1 1 
Given, A = 15 6 5 

–1
……(1)
 
 5 2 2 

1 2 2 

and B = 1 3 0 

 0 2 1 

|B| = 1(3 – 0) – 2(– 1 – 0) – 2(2 – 0) = 3 + 2 – 4 = 1  0


Hence, B–1 exists. [1]

E 99

Now, the co-factors of matrix B are given as :
C11 = 3, C12 = 1, C13 = 2,
C21 = 2, C22 = 1, C23 = 2,
C31 = 6, C32 = 2, C33 = 5 [1]
t
3 1 2 3 2 6 
 adj (B) =  2 1 2  = 1 1 2 
 
 6 2 5   2 2 5 

3 2 6 
1
 B–1 = (adj B)  1 1 2  …..(2) [1]
| B| 2 2 5 
 
 (AB)–1 = B–1.A–1
3 2 6  3 1 1  9  30  30 3  12  12 3 10  12 
  
= 1 1 2 . 15 6 5  =  3  15  10 1  6  4 1  5  4  [from (1) and (2)] [1]
    
 2 2 5   5 2 2  6  30  25 2  12 10 2 10 10 

 9 3 5 
 (AB) =  2 1 0 
–1
[1]
 
 1 0 2 

6 MARKS QUESTIONS

 5 1 4 
13. If A  2 3 5  , find A–1 and use it to solve the following system of equations : [CBSE 2020]
 5 2 6 
5x – y + 4z = 5
2x + 3y + 5z = 2
5x – 2y + 6z = –1
 5 1 4 
Sol. Given, A  2 3 5 
 5 2 6 
 
5 1 4
 |A| = 2 3 5 = 5(18+10) + (12 – 25) + 4 (–4 – 15) = 140 – 13 – 76 = 51 0. [½]
5 2 6
Hence; A–1 exists
Now co-factors of elements of A are :
A11 = 28, A12 = 13, A13 = – 19
A21 = –2, A22 = 10, A23 = 5
A31 = – 17, A32 = – 17, A33 = 17 [1]

100 E

T
 28 13 19   28 2 17
 adj A =  2 10 5    13 10 17 [1]
 17 17 17   19 5 17 
   
28 2 17 
1 1 
13 10 17 
–1
 A  (adj A) = [1]
A 51  
 19 5 17 
Given system of equations are
5x – y + 4z = 5
2x + 3y + 5z = 2
5x – 2y + 6z = –1
This system is written in matrix form as :
5 1 4  x   5 
2 3 5   y    2  [½]
5 2 6  z   1 
    
 AX = B X = A–1B
 x  1  28 2 17  5 
  y    13 10 17  2 
z  51  19 5 17   1 
    
x  140  4  17 
or  y  = 1 65  20  17  [1]
  51  
 z   95  10  17

 x  1  153   3 
  y    102    2 
z  51  102   2 
     
 x = 3 , y = 2, z = –2 [1]
1 1 1 
14. Show that, for the matrix A = 1 2 3 , A3 – 6A2 + 5A + 11 I = O. Hence, find A–1. [CBSE 2019]
 2 1 3 
 
OR
Using matrix method, solve the following system of equations :
3x – 2y + 3z = 8
2x + y – z = 1
4x – 3y + 2z = 4
1 1 1 
Sol. Given; A = 1 2 3
 2 1 3 
 
To prove : A3 – 6A2  5A  11I  O

E 101

1 1 1  1 1 1 1  1  2 1  2 –1 1 – 3  3 
A = A.A = 1
2
2  
–3 . 1 2 –3= 1  2 – 6 1  4  3 1 – 6 – 9 
2 –1  
3 2 –1 3  2 –1  6 2 – 2 – 3 2  3  9 
  

 4 2 1
 A =  –3
2
8 –14  [1½]
 7 –3 14 

 4 2 1 1 1 1  4  2  2 4  4 –1 4–63 
and A3 = A2.A =  –3 8 –14  . 1 2 – 3  =  –3  8 – 28 – 3  16  14 – 3 – 24 – 42 
 7 –3  
14  2 –1 3   7 – 3  28 7 – 6 –14 7  9  42 

 8 7 1 
 A3 =  –23 27 –69  [1½]
 32 –13 58 

L.H.S. = A3 – 6A2 + 5A + 11I

 8 7 1  4 2 1  1 1 1  1 0 0 
  –23 27 –69 – 6  –3 8 –14  5 1 2 –3  11 0 1 0 
 32 –13 58   7 –3 14  2 –1 3  0 0 1 
       

 8 – 24  5  11 7 –12  5  0 1– 6  5  0  0 0 0 
= –23  18  5  0 27 – 48 10 11 –69  84 –15  0   0 0 0  = O (Zero Matrix) [1]

32 – 42  10  0 –13 18 – 5  0 58 – 84 15 11  0 0 0 
   
Now, A3–6A2 + 5A + 11I = O
 A3A–1 –6A2A–1+5AA–1+11IA–1 = OA–1 (Post multiplying both sides by A–1)
 A2 (AA–1) –6A (AA–1) +5 (AA–1) +11 IA–1 = OA–1

 A2 – 6A + 5 I + 11 A–1 = O [ AA–1 = I and OA–1 = O]

 4 2 1  1 1 1  1 0 0  
–1 –1   
 A–1 =  (A 2  6A  5I)   –3 8 –14  – 6 1 2 –3  5 0 1 0   [1]
11 11   7 –3 14  2 –1 3  0 0 1  
    

–1  4 – 6  5 2 – 6  0 1– 6 0   –3 / 11 4 / 11 5 / 11 
 A –1   –3 – 6  0 8 – 12  5 –14  18  0   A –1   9 / 11 –1 / 11 –4 / 11 [1]
11  7 – 12  0 –3  6  0 14 – 18  5   5 / 11 –3 / 11 –1 / 11
 

OR
Given system of equations are :
3x –2y + 3z = 8
2x + y – z = 1
and 4x – 3y + 2z = 4

102 E

By using matrix method; the given system of equations can be written as :
AX = B

 3 –2 3  x  8 
where A =  2 1 –1 , X =  y  and B = 1 
 4 –3 2  z  4
     

3 –2 3
Now; |A|  2 1 –1 = –3 + 16 – 30 = –17  0 [1]
4 –3 2

Hence; A–1 exists.

 3 –2 3 
Now; A =  2 1 –1
 4 –3 2 
 
Co-factors are given as :
C11 = –1, C12 = –8, C13 = –10, C21 = –5, C22 = –6, C23 = 1, C31 = –1, C32 = 9, C33 = 7 [1]
Hence, adj(A) = [Cij]T

 –1 –5 –1
   adj(A)  –8 –6 9 
 [1]
 –10 1 7 
 

adj A
Now; AX = B  X  A–1 .B = ·B [1]
|A|

1 
–8 – 5 – 4 
1 
x  –17
1 
–1 –5 –1 8 
   y   –  –8 –6 9  . 1   –  –64 – 6  36   –  –34  [1]
  17  –10 1 7   4  17  –80  1  28  17  –51
z         
 x  1 
  y    2   x = 1, y = 2, z = 3 [1]
 z  3
   

2 3 5 
15. If A   3 2 4  , find A–1. Hence, using A–1, solve the system of equations :
 
1 1 2 

2x – 3y + 5z = 11, 3x + 2y – 4z = –5, x + y – 2z = –3. [CBSE 2017, 2018]


2 3 5 
Sol. Given, A   3 2 4 
 
1 1 2 

A is invertible if |A|  0

E 103

Now, |A| = 2(–4 + 4) + 3 (–6 + 4) + 5(3 – 2)
= 0 – 6 + 5 = –1  0 [1]
Co-factors are given as :
C11 = 0, C12 = 2, C13 = 1
C21 = –1, C22 = –9, C23 = –5
C31 = 2, C32 = 23, C33 = 13 [1]
T
 adj(A) =  C ij 
T
0 2 1 0 1 2 
or
 
adj(A)   1 9 5  2 9 23 [2]
 
 2 23 13  1 5 13 

 0 1 2 
adj(A) 
  2 9 23
1
 A  [½]
|A|
 1 5 13

Now, given equations are


2x – 3y + 5z = 11
3x + 2y – 4z = –5
and x + y – 2z = –3
2 3 5   x  11 
  3 2 4  y    5
1 1 2  z   3
    
 AX = B or X = A–1B [1]
 0 1 2   11
 X   2 9 23  5
 1 5 13  3

 0  11  (1)  (5)  (2)  (3) 


or X  (2)  11  9  (5)  (23)  (3)  [½]
 (1)  11  5  (5)  (13)  (3) 

 x  1 
  y   2  x = 1, y = 2, z = 3 [1]
   
 z  3 

104 E

CHAPTER-5 : CONTINUITY AND DIFFERENTIABILITY
CONTINUITY
1. CONTINUOUS FUNCTION :
A function for which a small change in the independent variable causes only a small change and
not a sudden jump in the dependent variable is called continuous function. Naively, we may say
that a function is continuous at a fixed point if we can draw the graph of the function around that
point without lifting the pen from the plane of the paper.

2. CONTINUITY OF A FUNCTION AT A POINT :


A function f(x) is said to be continuous at x = a, if lim f (x)  f (a) . Symbolically, f is continuous at
x a

x = a if lim f (a  h)  lim f (a  h)  f  a  , h > 0 i.e L.H.L.x = a = R.H.L.x = a = f(a), h > 0


h 0 h 0

3. REASONS OF DISCONTINUITY :
(a)Limit does not exist
i.e. lim f (x)  lim f (x)
x a x a

(b)f(x) is not defined at x = a


0 1 2 3 4
(c)lim f (x)  f (a)
x a lim f(x) f(1)
x1

Geometrically, the graph of the function will exhibit a break lim f(x) does not exist
x2
f(x) is not defined at x = 3
at x = a, if the function is discontinuous at x = a. The graph as
shown is discontinuous at x = 1, 2 and 3.
4. CONTINUOUS FUNCTION:
A function is said to be continuous function if it is continuous at every point in its domain.
Function f(x) Interval in which f(x) is continuous
Constant function (–)
n
x , n is an integer  0 (–)
x–n, n is a positive integer (–) – {0}
|x – a| (–)
p(x) = a0 xn + a1xn – 1 + a2xn – 2 + ..... + an (–)
p(x) (–) – {x : q(x) = 0}
, where p(x) and q(x) are polynomial in x
q(x)
sin x, cos x, ex (–)
tan x, sec x (–) – {(2n + 1)/2 : n  I}
cot x, cosec x (–) – {n : n  I}
log x (0)

E 105

1  cos 4x
 , x0
Illustration 1: If f(x)   8x 2 is continuous at x = 0, then find value of k.
 k , x0
Solution: It is given that the function f is continuous at x = 0. Therefore, lim f (x)  f (0)
x 0

2
1  cos 4x 2 sin 2 2x  sin 2x 
 lim 2
 k  lim 2
 k   lim    k k = 1
x 0 8x x  0 8x x 0
 2x 
Thus, f is continuous at x = 0 if k = 1.
 1  cos 4x
 ,x0
x2

Illustration 2: Let f(x)   a ,x0

 x
 ,x 0
 16  x  4
For what value of a, is 'f' continuous at x = 0 ? [Exemplar]
2
1  cos 4x 2 sin 2 2x  sin 2x 
Solution: Now, f(0) = a; lim f(x)  lim  lim  lim 8   =8
x 0 x 0 x 2
x 0 
x 2
x 0  2x 

and lim f(x)  lim


x
 lim
x  16  x  4 
x 0 x 0
16  x  4 x 0
 16  x  4  16  x  4 
= lim
x  16  x  4   lim  
16  x  4  16  4  8
x 0 16  x  16 x  0

Hence, lim f(x)  lim f(x)  8 = f(0)  f is continuous at x = 0 only if a = 8.


x 0 x 0

Illustration 3: Find the value of a and b such that the function f defined by
 x4
 | x  4 |  a , if x  4

f (x)  a  b , if x  4 is a continuous function at x = 4.
 x4
  b , if x  4
 | x  4 |
x4  x4
Solution: lim f (x) = lim  b = lim    b  1 b {x > 4, |x – 4| = (x – 4)}
x 4 x 4 | x 4| x 4  x  4 

x4 x4
and lim f(x) = lim  a = lim  a  1  a {x < 4, |x – 4| = –(x – 4)}
x 4 x 4 | x 4| x 4 (x  4)

Now; f(4) = a + b
Thus, lim f (x)  lim f (x)  f (4) (For Continuity)
x 4 x 4

 1 + b = – 1 + a = a + b a – b = 2 and b = –1. So; a = 1, b = –1

106 E

2 cosx 1  
Illustration 4: If f(x)  , x  , find the value of f   so that ƒ(x) becomes continuous at
cot x 1 4 4

x= . [Exemplar]
4
2 cos x  1 
Solution: Given; f(x) = ,x
cot x  1 4
 2 cos x  1 
Therefore, lim f(x)  lim  
x
 
x  cot x  1 
4 4

( 2 cos x  1)sin x
 lim
x
 (cos x  sin x)
4

( 2 cos x  1) ( 2 cos x  1) (cos x  sin x)


 lim . . .sin x
x

4
( 2 cos x  1) (cos x  sin x) (cos x  sin x)

 lim
 2 cos 2
x 1  .  cos x  sin x  .(sin x)
x

4
 cos 2
x  sin 2
x   2 cos x  1

cos 2x  cos x  sin x 


 lim .  .(sin x)
x  cos 2x 
4

2 cos x  1 

1  1 1 

 lim
 cos x  sin x  sin x  2  2 2   1

x

4 
2 cos x  1  
 2.
1 
 1
2
 2 
1
Thus, lim f(x) 
x
 2
4

  1
Hence, for f to be continuous at x  ,f 
4  4  2
5. SOME IMPORTANT POINTS :
Suppose f(x) and g(x) be two real functions continuous at a real number 'a', then
(i) f(x)  g(x) is continuous at x = a.
(ii) f(x) . g(x) is continuous at x = a.

 f (x) 
(iii)  g(x)  is continuous at x = a, (provided g(a) 0).
 
(iv) Suppose f and g are real valued functions such that (fog) is defined at x = a. If g is
continuous at x = a and if f is continuous at x = a, then (fog) is continuous at x = a.

E 107

Illustration 5: Show that the function f defined by f(x) = |1 – x + |x| |, where x is any real number, is a
continuous function.
Solution: Let us define g by g(x) = 1 – x + |x|
and h by h(x) = |x| for all real numbers x. Then;
(h o g) (x) = h(g (x))
= h(1 – x + |x|)
= |1 – x + |x| | = f(x)
We have seen that h is a continuous function. Also, g being a sum of a polynomial function and
the modulus function is continuous.
Hence, f being a composite of two continuous functions is continuous.

 1  kx  1  kx
 , if 1  x  0
Illustration 6: Find the value of k, for which f(x)   x continuous at
 2x  1 , if 0  x  1
 x  1
x = 0.
Solution: f(x) is continuous at x = 0
So, R.H.L. = L.H.L. = f(0)
0 1
f(0) =  1
0 1
1  kh  1  kh
RHL = lim f(0  h)  lim
h 0 h 0 h
(1  kh)  (1  kh) 2kh
 lim  lim
h 0
h( 1  kh  1  kh ) h 0
h( 1  kh  1  kh )
2k
 k
2
Hence, f(0) = RHL
 k = –1

108 E

DIFFERENTIABILITY & DIFFERENTIATION
1. MEANING OF DERIVATIVE :
The instantaneous rate of change of a function with respect to the independent variable is called
derivative. Let ‘f’ be a given function of one variable and let x denote a number

(positive or negative) to be added to the number x. Let f denote the corresponding change of

' f ' then f = f(x +x) – f(x)


f f (x  x)  f (x)
  
x x

If f/x approaches a limit as x approaches zero, this limit is the derivative of 'f' at the point x.
The derivative of a function ‘f’ is a function; this function is denoted by symbols such as
df d dy
f '(x), ,  f (x)  or if y = f(x) by or y'.
dx dx dx
df f f (x  x)  f (x)
  lim  lim
dx x 0 x x 0 x

The process of finding derivative of a function is called differentiation. We also use the phrase
differentiate f(x) with respect to x to mean find f '(x).
2. DIFFERENTIABILITY :
(a) Right hand derivative : The right hand derivative of f(x) at x = a denoted by f '(a+) is
defined as :
f(a + h) – f(a)
Rf'(a) = f'(a  ) = lim , provided the limit exists and is finite. (h > 0)
h 0 h
(b) Left hand derivative : The left hand derivative of f(x) at x = a denoted by f '(a–) is defined as :
f (a  h)  f (a)
Lf'(a) = f '(a  )  lim , provided the limit exists and is finite. (h > 0)
h 0 h
Hence f(x) is said to be derivable or differentiable at x = a, if R.H.D. = L.H.D. = finite
quantity and it is denoted by f '(a); where f '(a) = f '(a–) = f '(a+) and it is called derivative or
differential coefficient of f(x) at x = a.
Note : (i) Right hand and left hand derivative at x = a is also denoted by Rf '(a) and Lf '(a)
respectively.
(ii) The common value of Rf '(a) and Lf '(a) is denoted by f '(a).

E 109

 x[x] , if 0  x  2
Illustration 7: Consider the function f (x)   . Discuss derivability of f at x = 2.
(x  1)x , if 2  x  3
Solution: Here, f(2) = (2 – 1) 2 = 1 × 2 = 2
f (a  h)  f (a) (2  h)[2  h]  2 (2  h)  2
Lf (2) = lim   1
h 0 h h h
f (a  h)  f (a) f (2  h)  f (2) (2  h  1) (2  h)  2
Rf (2) = lim   lim
h 0 h h h  0 h
h 2  3h h(h  3)
 lim  lim  lim  h  3  3
h 0 h h 0 h h 0

Thus, Lf (2)  Rf (2) f is not derivable at x = 2.


Illustration 8: Prove that the function f given by f(x) = |x – 1|, x  R is not differentiable at x = 1.
Solution: Given f(x) = |x – 1|
 x  1, if x  1
We redefine the function as f (x)  
 x  1 , if x  1
Now, at x = 1
f (1  h)  f (1) –(1 – h)  1 – (1 –1) 1  h  1  0 h
Lf (1) = lim  lim  lim  lim  1
h 0 h h 0 –h h 0 h h 0  h

f (1  h)  f (1) 1 h 1 0 h
Rf (1)  lim  lim  lim  1
h 0 h h 0 h h 0 h

 Lf (1)  Rf (1) f(x) is not differentiable at x = 1.

3. THEOREM :
If a function f(x) is derivable at x = a, then f(x) is continuous at x = a. i.e. every differentiable
function is continuous.
f (a  h)  f (a)
Proof : f '(a)  lim exists.
h 0 h
f (a  h)  f (a)
Also, f (a  h)  f (a)  .h [h  0]
h
f (a  h)  f (a)
 lim [f (a  h)  f (a)]  lim .h  f '(a).0  0
h 0 h 0 h
 lim [f (a  h)  f (a)]  0  limf (a  h)  f (a)  f (x) is continuous at x = a.
h 0 h 0

Note :(i) Converse of above theorem is NOT true.


(ii) All polynomial, trigonometric, logarithmic and exponential functions are continuous
and differentiable in their domains.
(iii) If f(x) and g(x) are differentiable at x = a, then the functions f(x) + g(x), f(x) – g(x),
f(x).g(x) will also be differentiable at x = a and if g(a) 0 then the function f(x)/g(x)
will also be differentiable at x = a.

110 E

4. DERIVABILITY OVER AN INTERVAL :
(a) f(x) is said to be derivable over an open interval (a, b) if it is derivable at each & every
point of the open interval (a, b).
(b) f(x) is said to be derivable over the closed interval [a, b] if :
(i) f(x) is derivable in (a, b) and
(ii) for the points a and b, f '(a+) and f '(b–) exist.
1  x, if x  2
Illustration 9: Consider the function f (x)   . Check the continuity and differentiability
5  x, if x  2
of f(x) at x = 2.
Solution: Here, f(2) = 1 + 2 = 3.
lim f(x) = lim (1 + x) = 1 + 2 = 3 and lim f(x) = lim (5 – x) = 5 – 2 = 3.
x  2 x2 x2 x2

Thus, f(2) = lim f(x) = lim f(x) = 3


x2 x2

  f is continuous at x = 2.
f (2  h)  f (2) 1  (2  h)  3 h
Also, Lf '(2)  lim  lim  lim 1
h 0 h h  0 h h  0 h
f (2  h)  f (2) 5 – (2  h)  3 h
and, Rf '(2)  lim  lim  lim  1
h 0 h h  0 h h  0 h
  Lf '(2) Rf '(2)f is not derivable at x = 2.
Illustration 10: Examine the differentiability of the function ƒ defined by
2x  3 ; if 3  x  2

f (x)   x  1 ; if 2  x  0
 x  2 ; if 0  x  1

Solution : The only doubtful points for differentiability of ƒ(x) are x = –2 and x = 0. For
differentiability at x = – 2; we have :
f ( 2  h)  f ( 2) 2(2  h)  3  (2  1) 2h
Lf ( 2)  lim  lim  lim 2
h 0 h h 0 h h 0  h

and Rf (2)  lim


f ( 2  h)  f ( 2)
 lim
 2  h  1  (2  1)  lim  h  1  (1)  lim h  1
h 0 h h 0 h h 0 h h 0 h

Thus, Rf'(–2)  Lf'(–2). Therefore, f is not differentiable at x = –2.


Similarly, for differentiability at x = 0, we have :

Lf (0)  lim
f (0  h)  f (0)
 lim
 0  h  1  (0  2)
h 0 h h 0 h
h  1  1
 lim  lim 1   = does not exist.
h 0 h h  0
 h
Hence, f is not differentiable at x = 0.

E 111

5. DIFFERENTIATION :
Fundamental Theorems :
If f and g are derivable functions of x, then :
d
(a) (f (x)  g(x))  f '(x)  g '(x)
dx
d
(b) (cf (x))  cf '(x) , where c is any constant
dx
d
(c) (f (x) g(x))  f (x) g '(x)  g(x) f '(x) , known as “PRODUCT RULE”
dx
d  f (x)  g(x) f '(x)  f (x) g '(x)
(d)    , where g(x) 0; known as “QUOTIENT RULE”
 g(x) 
2
dx  g(x) 

dy dy du
(e) If y = f(u) and u = g (x), then  . , known as “CHAIN RULE”
dx du dx
dy du
Note : In general if y = f(u), then  f ' u . .
dx dx

6. DERIVATIVE OF STANDARD FUNCTIONS :


f(x) f '(x)
(i) xn nxn–1
(ii) ex ex
(iii) ax ax log a, a > 0
(iv) log x 1/x
(v) logax (1/x) logae, a > 0, a  1
(vi) sinx cosx
(vii) cosx – sinx
(viii) tanx sec2x
(ix) cotx – cosec2x
(x) secx secx tanx
(xi) cosecx – cosecx cotx
(xii) constant 0

Illustration 11: Differentiate tan x with respect to x. [Exemplar]

Solution: Let y = tan x


dy 1 d 1 d
   ·  tan x   · sec 2 x  x
dx 2 tan x dx 2 tan x dx

1  1  sec 2 x
     · sec2 x   
2 tan x  2 x  4 x tan x

112 E

dy
Illustration 12: If y = tan (x + y), find . [Exemplar]
dx
Solution: Given y = tan (x + y). Differentiating both sides w.r.t. x, we have
dy d  dy 
  sec 2 (x  y) (x  y)  sec2 (x  y)  1  
dx dx  dx 
dy
 1  sec 2 (x  y)   sec 2 (x  y)
dx
dy sec (x  y)
2
sec 2 (x  y)
    cosec 2 (x  y)
dx 1  sec (x  y) tan (x  y)
2 2

dy
Illustration 13: If y  log  x  x 2  a 2  , then find .
  dx
Solution: We have, y  log  x  x 2  a 2 
 
dy 1 d
    x  x2  a 2  (Differentiating w.r.t. x)
dx  x  x 2  a 2  dx  
 
dy 1  1 d 
    1   (x 2  a 2 ) 
dx  x  x 2  a 2   2 x 2  a 2 dx 
 
dy 1  1 
    1   2x 
dx  x  x 2  a 2   2 x  a 2 2

 
dy 1  x2  a2  x  dy 1
      =
dx  x  x 2  a 2   x  a 
2 2 dx x  a2
2
 
1  sin x dy
Illustration 14: If y  log , then find ?
1  sin x dx
1  sin x 1  sin x 1  sin x
Solution: y  log  y  log  
1  sin x 1  sin x 1  sin x
(1  sin x) 2 (1  sin x) 2
  y  log   y  log
1  sin 2 x cos 2 x
 1  sin x 
  y  log   y = log(sec x + tan x)
 cos x 
dy 1 d
    (sec x  tan x) (Differentiating w.r.t. x)
dx (sec x  tan x) dx
dy 1
    (sec x tan x  sec2 x)
dx (sec x  tan x)
dy sec x(tan x  sec x)
  
dx (sec x  tan x)
dy
  sec x
dx
E 113

7. DIFFERENTIATION OF IMPLICIT FUNCTION :
Let function is (x, y) = 0, then, to find dy/dx; in the case of implicit functions, we differentiate
each term w.r.t. x regarding y as a functions of x and then collect terms in dy/dx together on one
side to finally find (dy/dx).
dy y
Illustration 15 : If y = x sin y; prove that x =
dx 1  x cos y
Solution: Given, y = x sin y; differentiating both sides w.r.t. x.
dy dy dy dy
 x cos y  sin y.1   x cos y  sin y 
dx dx dx dx
dy sin y dy x sin y
     x  , (multiplying both sides by x)
dx 1  x cos y dx 1  x cos y
y
= ( x sin y = y)
1  x cos y

dy 1  y2
Illustration 16 : If 1  x  1  y  a(x  y) , then show that
2 2
 [Exemplar]
dx 1 x2

Solution: Given 1  x 2  1  y2  a(x  y)

Putting x = sin  = sin–1x, and y = sin  = sin–1y, we get

1  sin 2   1  sin 2   a(sin   sin )

  cos + cos = a(sin – sin)


(  )            
  2cos cos    a.2cos   sin  
2  2   2   2 
     
  cot  a  cot 1 a      2cot 1 a
 2  2
  sin–1x – sin–1y = 2cot–1a
Differentiating both sides w.r.t. x, we get

1 1
dy dy 1  y2
 0 
1 x2 1  y 2 dx dx 1 x2

dy
Illustration 17: If ex + ey = ex+y, prove that   eyx
dx
x y x+y
Solution: Given that, e + e = e (Differentiating w.r.t. x, we get)
dy  dy  dy
ex  ey  e x  y 1    (e y  e x  y )  exy  ex
dx  dx  dx
dy ex  y  ex ex  e y  ex
which implies that  y x  y  y x y  e y  x . [ ex + ey = ex+y]
dx e  e e e e

114 E

8. DIFFERENTIATION OF INVERSE TRIGONOMETRIC FUNCTIONS USING
TRIGONOMETRICAL SUBSTITUTION :
Standard Formulae :
f(x) f '(x)
1
(i) sin–1 x , 1  x  1
1 x2
(ii) cos–1 x 1
, 1  x  1
1 x2
1
(iii) tan–1 x , xR
1 x2
1
(iv) cot–1 x , xR
1 x2
1
, | x | 1
(v) sec–1 x | x | x 2 1
1
–1
(vi) cosec x , | x | 1
| x | x 2 1

Some standard substitution


Expression Substitution
a2  x2 x = a tan  or, a cot 
a2  x2 x = a sin  or, a cos 
x2  a2 x = a sec  or, a cosec 
ax ax
or , x = a cos 2
ax ax

x2  a2 a2  x2
or , x2 = a2 cos 2
a x2 2
a2  x2

Illustration 18: Differentiate sin 1  2x 1  x2  with respect to x, if 


1 1
x
2 2

Solution: Let y = sin 1  2x 1  x2  . Putting x = sin , we get


y = sin–1 (2 sin cos ) = sin–1 (sin 2)
1 1
If  x , then
2 2
1 1
   sin  
2 2
   
        2 
4 4 2 2

E 115

  
 y = sin–1 (sin 2)   2  2  2 

 y = 2    [ sin   x    sin 1 x] 


 y = 2 sin–1 x
dy 2
  (Differentiating w.r.t. x)
dx 1  x2
 2x  dy
Illustration 19: sin–1  2 
, |x| 1, find .
 1 x  dx

 2x 
Solution: Let y = sin–1  2  , put  = tan
–1
x, i.e., x = tan     
 1 x 

 2 tan  
then y = sin–1  1  tan 2   = sin–1 (sin 2) = 2 = 2 tan–1x

Differentiating w.r.t. x, we get


dy 1 2
 2. 
dx 1 x 2
1 x2

 1 x  1 x 
 dy
Illustration 20: If y  sin 1   , then find .

 2 
 dx

 1 x  1 x 
 
Solution: y  sin 1  

 2 

1
Put x  cos 2;   cos 1 x
2
 1  cos 2  1  cos 2 
 
  y  sin 1   

 2 

 2 cos 2   2sin 2  
1
  y  sin  
 2 
 1 1     
  y  sin 1  cos   sin    y  sin 1 sin cos   cos sin 
 2 2   4 4 
     1
  y  sin 1 sin       y     y   cos 1 x
 4  4 4 2

dy 1 1
   (Differentiating w.r.t. x.)
dx 2 1 x2

dy 1
  =
dx 2 1 x2

116 E

 cos x 
Illustration 21: y  tan 1 
1  sin x 
 2 x 2 x

  cos  sin  
 cos x 
 y  tan 1  
2 2 
Solution: y  tan 1 
1  sin x   x 
2
x
  cos  sin  
 2 2 
 x x   x
  cos 2  sin 2    1  tan 
  y  tan 1      y  tan 1

2

  cos x  sin x    1  tan
x

  2

2    2
   x   x dy –1
  y  tan 1  tan      y     (Differentiating w.r.t x)
  4 2  4 2 dx 2
9. LOGARITHMIC DIFFERENTIATION :
To find the derivative of:
(a) A function which is the product or quotient of a number of functions.
(b) A function of the form [f(x)] g(x) where f & g are both derivable;
[it is convenient to take the logarithm of the function first and then differentiate]
Consider y = f(x) = [u(x)]v (x)
By taking logarithm (to base e) the above may be re-written as log y = v(x) log[u(x)]
Using chain rule we may differentiate this to get
1 dy 1
.  v(x) . . u '(x)  v '(x) . log[u(x)]
y dx u(x)
dy  v(x) 
which implies that  y . u '(x)  v '(x) . log[u(x)]
dx  u(x) 
dy v  x   v(x) 
=   u  x    . u '(x)  v '(x) . log[u(x)] 
dx  u(x) 
dy log x
Illustration 22 : If xy = ex–y, show that 
dx log(xe)
2

Solution: We have, xy = ex – y
 eylogx = ex – y  ylogx = x – y
  ylogx + y = x  y(1 + logx) = x
x
 y
1  log x
On differentiating both sides w.r.to x, we get
 1
(1  log x).1  x.  0  
dy  x log x log x log x
   
dx (1  log x) 2
(1  log x) 2
(log e  log x) 2
[log(ex)]2

E 117

Illustration 23: Differentiate the function (sin x)tan x + (cos x)sec x w.r.t. x.
Solution: Let y = (sin x)tan x + (cos x)sec x and u = (sin x)tan x ; v = (cos x)sec x
dy du dv
Now, y = u + v and   ......(1) (Differentiating w.r.t. x)
dx dx dx
We have, u = (sin x)tan x
Taking log on both sides, we have : log u = tan x log (sin x)
Differentiating both sides w.r.t. x, we have
1 du 1
 tan x . cos x  log sin x.sec 2 x
u dx sin x
du  1 
 u  tan x . cos x  sec2 x.log sin x 
dx  sin x 

 (sin x) tan x 1  sec 2 x.log sin x 


du
dx
Again; v = (cos x)sec x
Taking log on both sides; we have : log v = sec x log (cos x)
Differentiating both sides w.r.t. x, we have
1 dv  1 
 (sec x)     ( sin x)  log(cos x)(sec x .tan x)
v dx  cos x 
  sec2 x.sin x  (sec x.tan x).log(cos x)
dv
    v[ sec 2 x.sin x  (sec x.tan x).log(cos x)]
dx
dv
 (cos x)sec x [ sec 2 x.sin x  (sec x.tan x).log(cos x)]
dx
Putting these values in (1), we have

 (sin x) tan x 1  sec 2 x.log sin x   (cos x)sec x [  sec 2 x.sin x  (sec x.tan x).log(cos x)]
dy
dx
10. PARAMETRIC DIFFERENTIATION :
dy (dy / d)
If y  f () and x  g() ; where  is a parameter, then  .
dx (dx / d)
dy  y log x
Illustration 24: If x = ecos2t and y = esin2t, prove that  . [Exemplar]
dx x log y
Solution: x = ecos2t
  log x = cos2t (taking log on both sides) .....(1)
1 dx dx
  .  2 sin 2t   2x sin 2t (Differentiating w.r.t. t)
x dt dt
118 E

and y = esin2t
 logy = sin2t (taking log on both sides) .....(2)
1 dy
  .  2cos 2t      (Differentiating w.r.t. t)
y dt
dy
   2y cos 2t
dt
dy dy / dt 2y cos 2t y log x
Now,    (from equation (1) and (2))
dx dx / dt 2x sin 2t x log y
11. SECOND ORDER DERIVATIVE :
dy
Let y = f(x). Then,  f '(x) .....(1)
dx
If f '(x) is differentiable, we may differentiate (i) again w.r.t. x. Then, the left hand side becomes
d  dy  d2 y
  which is called the second order derivative of y w.r.t. x and is denoted by .
dx  dx  dx 2
The second order derivative of f(x) is denoted by f "(x). It is also denoted by D2y or y" or y2 if
y = f(x). Similarly, higher order derivatives may be defined.
Illustration 25: (i) Find the second derivative of log x + cos x.
d2 y cos x
(ii) If y = sec x + tan x, prove that  . [Exemplar]
dx 2
(1  sin x) 2

Solution: (i) Let y = log x + cos x, differentiating w.r.t. x, we get


dy 1
  sin x , differentiating again w.r.t. x, we get
dx x
d  dy  d 1  d2 y 1
  =  – sin x    (–1) x –2  (– cos x)   2  cos x
dx  dx  dx  x  dx 2
x
(ii) Given y = sec x + tan x, differentiating w.r.t. x, we get
dy 1  sin x  1 sin x  1 1  sin x
= sec x tan x + sec2 x =    
dx cos x  cos x  cos x
2
cos 2 x 1  sin 2 x


1  sin x  
1
(1  sin x) (1  sin x) 1  sin x

d  dy  d  1 
Again differentiating w.r.t. x, we have;  
dx  dx  dx  1  sin x 

d2 y d cos x
    {(1 – sin x)–1} = (–1) (1 – sin x)–2 (0 – cos x) = .
dx 2
dx (1  sin x) 2

E 119

1 d2y dy
Illustration 26: If y  emsin x
, prove that (1  x 2 ) 2
 x  m 2y  0
dx dx
1
Solution : We have, y  emsin x

Differentiating with respect to x, we obtain


dy m dy my  em sin  y 
1 1
 e m sin x    x

dx 1  x2 dx 1  x2
2 2
 dy  m 2y2  dy  dy
      (1  x 2 )    m 2 y 2  (1  x 2 )y12  m 2 y 2 where y1 
 dx  1  x 2
 dx  dx
d 2 d d 2
Differentiating with respect x, we obtain : (1  x 2 ) (y1 )  (y12 ) (1  x 2 )  m 2 (y )
dx dx dx
 d 2 d 2 
  (1  x 2 )2y1y 2  y12 (2x)  m 2 (2yy1 )  dx (y1 )  2y1y 2 and dx (y )  2yy1 

  2y1 (1  x2 )y2  xy1  m 2 y  0  (1  x 2 )y 2  xy1  m 2 y  0  y1  0


120 E

EXERCISE–I
1– cos4x
 , x0
1. If f(x) =  x 2 is continuous then the value of a -
a , x=0

(A) 0 (B) 1 (C) 4 (D) 8


 | x –1 |
 1– x + a , x > 1

2. If f(x) = a + b , x =1 is continuous at x = 1, then the values of a and b are respectively:
 | x –1 |
 + b , x <1
 1– x
(A) 1, 1 (B) 1, –1 (C) 2, 3 (D) None of these
 x3 + x 2 –16x + 20
 , x2
3. If f(x) =  (x – 2)2 is continuous for all values of x, then the value of k is:
 k , x=2

[Exemplar]
(A) 5 (B) 6 (C) 7 (D) 8
a  bx 2 ;x 1
4. If f(x)   , then find a and b so that f(x) become differentiable at x = 1?
3ax  b  2 ; x  1
(A) a = 3, b = 2 (B) a = 2, b = 3 (C) a = 0, b = 3 (D) a = 2, b = –3

d  2  1 1  x  
5. sin  cot   is :
dx   1  x  

1 1
(A)  (B) 0 (C) (D) –1
2 2
1 d2y
6. If y = 2 2 , then equals-
x –a dx 2

3x 2 + a 2 3x 2 + a 2 2(3x 2 + a 2 ) 2(3x 2 + a 2 )
(A) (B) (C) (D)
(x 2 – a 2 )3 (x 2 – a 2 )4 (x 2 – a 2 )3 (x 2 – a 2 )4

 1  x2  dy
7. If y  log  2 
, then is equal to [Exemplar]
1 x  dx
4x 3 4x 1 4x 3
(A) (B) (C) (D)
1  x4 1  x4 4  x4 1  x4
8. If y = xn – 1 log x then x2y2 + (3 – 2n) xy1 is equal to
(A) –(n – 1)2 y (B) (n – 1)2 y (C) –n2y (D) n2y

E 121

1  sin 3 x
 2
, x /2
 3cos x
9. If f(x) =  a , x   / 2 is continuous at x = /2, then value of a and b are-
 b(1  sin x)
 , x / 2
 (  2x)
2

(A) 1/2, 1/4 (B) 2, 4 (C) 1/2, 4 (D) 1/4, 2


–1 2 –1
10. The derivative of cos (2x – 1) with respect to cos x is
1 2
(A) 2 (B) (C) (D) 1 – x2
2 1  x2 x

dy
11. If cos(xy) = x, then is equal to-
dx
y  cosec(xy) y  sin(xy) y  cos(xy)  y  cosec(xy) 
(A) (B) (C) (D) –  
x x x  x 
d2y
12. If x = a(+ sin), y = a(1 – cos) then the value of at  = /2 is-
dx 2
1 1
(A) a (B) – (C) (D) –a
a a

ASSERTION-REASON BASED QUESTIONS

In the following questions, a statement of assertion (A) is followed by a statement of Reason (R).
Choose the correct answer out of the following choices.
(A) Both A and R are true and R is the correct explanation of A.
(B) Both A and R are true but R is not the correct explanation of A.
(C) A is true but R is false.
(D) A is false but R is true.
x   for x , 0

 
13. f(x)   cos x, for x  0, 
 2
 2
  
 x  2  for x   , 
 2 
Consider the following statements
Assertion (A) : The function f(x) is continuous at x = 0

Reason (R) : The function f(x) is continuous at x  .
2

122 E

ANSWER KEY
Q. No. 1 2 3 4 5 6 7 8 9 10
Ans. D B C B A C B A C A
Q. No. 11 12 13
Ans. D C B
SOLUTIONS
1. (D) Since the given function is continuous at x = 0 ; therefore lim f(x)  f(0)
x0

1  cos 4x 2sin 2 2x 4
 lim = a  lim × =a
x0 x2 x0 x 2
4
2
 sin 2x 
 lim 2   × 4 = a 8 = a
x0  2x 
2. (B) f(x) is continuous at x = 1; therefore, limf(x)  f(1)
x1

f(1) = a + b (given)
|1  h 1|
RHL : lim f(1 + h) = lim + a = –1 + a
h0 h0 1  (1  h)
 a + b = –1 + a b = –1
|1  h 1|
LHL : lim f(1 – h) = lim +b=1+b
h0 h0 1  (1  h)
 a + b = 1 + b a = 1
3. (C)  f(x) is continuous at x = 2; therefore, lim f(x)  f(2)
x2

 f(2–) = f(2+) = f(2) = k .......(1)


(2  h)3  (2  h)2  16(2  h)  20 h 3  7h 2
Now, f(2+) = lim = lim =7
h 0 (2  h  2)2 h 0 h2
From (1) k = 7
4. (B) f(x) will become differentiable at x = 1 if R.H.D. at (x = 1) = L.H.D. at (x = 1) (should be finite)
f(1  h)  f(1)
R.H.D. : f '(1+) = lim
h 0 h
3a(1  h)  b  2  3a  b  2 3ah
= lim = lim = 3a
h 0 h h 0 h

– f(1  h)  f(1) a  b(1  h)  3a  b  2


2
L.H.D. : f '(1 ) = lim = lim
h 0 h h  0 h
(2a  2b  2)  bh 2  2bh
= lim
h 0 h
Hence for this limit to be defined

E 123

– 2a + 2b – 2 = 0 b = a + 1
f'(1–) = – lim (bh – 2b) = 2b
h0

 f'(1–) = f'(1+)


3a = 2b 3a = 2(a + 1) a = 2, b = 3
 1 x 
5. (A) Let y = sin2  cot 1 
 1  x 

Put x = cos 2
 1  cos 2 
 y = sin2  cot 1 2 –1
 = sin (cot (cot ))
 1  cos 2 

1  cos2  1  x 1 x
 y = sin2  =    (Differentiating w.r.t. x)
2 2 2 2
dy 1
 
dx 2

dy 2x d2y  (x 2  a 2 )2 .2  2x.2(x 2  a 2 ).2x  2(3x 2  a 2 )


6. (C) = 2 2 2  2 = –   = (x 2  a 2 )3
dx (x  a ) dx  (x 2  a 2 )4 
 1  x2 
7. (B) We have, y  log    log(1  x )  log(1  x )
2 2

 1  x2 
dy 1 1
  · (2x)  · (2x)
dx 1  x 2
1  x2
2x[1  x 2  1  x 2 ] 4x
 
(1  x 2 ) · (1  x 2 ) 1  x4
8. (A) We have y = xn – 1 log x
Diff. w.r.t. x
1
y1  (n  1)x n 2 log x  x n 1 ·
x
 xy1  (n  1)x n 1 log x  x n 1

 xy1  (n  1)y  x n 1 …..(1)


Diff. w.r.t. x again :
y1  xy 2  (n  1)y1  (n  1)x n 2

 x 2 y 2  xy1  x(n  1)y1  (n  1)x n 1

 x2 y2  xy1 (2  n)  (n 1) xy1  (n 1)y [from (1)]

 x 2 y 2  xy1 (2  n)  xy1 (n  1)  (n  1) 2 y

 x 2 y 2  xy1 (3  2n)  (n  1)2 y

124 E

     –  
9. (C) f(x) is continuous at x = therefore f   = f   = f  
2  2   2  2
 
   1  sin 3   h 
LHL : f   = lim 2 
2  h 0   
3cos2   h 
2 
1  cos3 h (1  cosh)(1  cosh  cos 2 h) 1
= lim = lim =
h0 3sin 2 h h0 3(1  cosh)(1  cosh) 2
  
   b 1  sin   h 
2 
RHL : f   = lim  2
2  h0
  
   2  2  h  
 
2
h
2
2b sin  
b(1  cosh) 2 = b
= lim 2
= lim 2
h0 4h h0
2 h 
8
4h   
2

Now f(x) is continuous at x =
2
       1 b
 f   = f   = f     a
2  2  2 2 8
  a = 1/2, b = 4
10. (A) Let u = cos–1 (2x2 – 1) and v = cos–1 x
For u = cos–1 (2x2 – 1)
Let x = cos  = cos–1x
u = cos–1(2 cos2– 1) = cos–1 (cos 2) = 2 = 2 cos–1x
du 2 dv 1
  and 
dx 1  x2 dx 1  x2
du du dx
  2
dv dv dx
11. (D) cos(xy) = x (Differentiating w.r.t. x)

 dy 
 –sin(xy) x.  y.1  1
 dx 
dy
x sin(xy).  y sin(xy)  1
dx
dy 1  ysin(xy)  y  cosec(xy) 
 = = – 
dx  xsin(xy)  x 

E 125

12. (C) We have x = a( + sin), y = a(1 – cos)
dx dy
Here, = a(1 + cos) and = asin
d d
dy dy / d  a sin  dy 2sin  / 2 cos  / 2 dy 
 = =  = = tan/2   tan
dx dx / d  a(1  cos ) dx 2 cos  / 2
2
dx 2

d2y 2  1 d 1 2 1 1 / 2sec 2  / 2 1
again  sec . .  sec  = = sec4/2
dx 2
2 2 dx 2 2 a(1  cos ) a.2cos  / 2
2
4a

 d2 y  1 1 4 1
 2 =
4
  2 = sec4/4 = =
 dx /2 4a 4a 4a a

13. (B) Assertion :


LHL = lim f(x) = lim(x  )  
x 0 x 0

RHL = lim f(x) = lim  cos x =  cos (0) = 


x 0 x 0

Also, f(0) =  cos(0) = 


Hence, f(x) is continuous at x = 0
 Assertion is true.

Reason : Now, for x 
2

LHL = lim  f(x)  lim  cos x   cos 0
 
x   x
 2
2 2

2 2
   
RHL = lim  f(x)  lim  x        0

x   x 
 2 2 2
2 2

 
Also, f     cos  0
2 2

Hence, f(x) continuous at x 
2
 Reason is true.
Hence, A is true, R is true; R is not a correct explanation for A

126 E

EXERCISE–II

 log(1  mx) – log 1 – nx 


 if x  0
1. If f(x) =  x , is continuous at x = 0, then find value of k.
 k if x  0

2. Determine the value of 'k' for which the following function is continuous at x = 3 :

 (x  3)2  36
 , x3
f(x)   x 3
 k , x 3

d2 y
3. If y = a sin x + b cos x, then prove that + y = 0.
dx 2

 x2  y2  dy y
4. If cos1  2 2 
 tan 1 a , prove that 
x y  dx x

d2y
5. If x = a(cos2t + 2t sin2t) and y = a(sin2t – 2t cos2t), then find the .
dx 2
6. Discuss the continuity and differentiability of the function f(x) = |x| + |x – 1| in the interval (–1, 2).
dy
7. Find the value of if xy + yx = ab
dx
2
y/x  d 2 y   dy 
8. If (ax + b)e = x, then show that x  2    x  y 
3

 dx   dx 

 a cos x – bsin x 
9. Differentiate tan –1   with respect to x. [Exemplar]
 b cos x  a sin x 

CASE STUDY-I

10. Consider the following function :

 1 1
 x tan , x0
f(x)   x
 0 , x0

Based on the above information answer the following ?

(i) Check the continuity and differentiability of the function f(x) at x = 0.

(ii) Find the value of f "(0)

E 127

CASE STUDY-II
11. A potter made a mud vessel, where the shape of the pot is based on f(x) = |x – 3| + |x – 2|, where
f(x) represents the height of the pot.

Based on the above information, answer the following questions :


(i) When x > 4, what will be the height in terms of x.
(ii) Find f (x) at x = 3
(iii) When the value of x lies between (2,3), then find the value of function f(x).
OR
(iii) If the potter is trying to make a pot using the function f(x) = [x], will he get a pot or not?
Why?

128 E

SOLUTIONS
 log(1  mx)  log(1  nx)
 ,x  0
1. Given, f(x)   x
 k ,x  0
Since f(x) is continuous at x = 0
So, RHL = LHL = f(0)
log(1  mx)  log(1  nx)
 lim k
x 0 x

 lim
m log(1  mx)
 lim n log
1  nx   k  (1  x) 
 1
mx nx  lim log
x
x 0 x 0
 x 0 
 m+n
 (x  3)2  36
 , x3
2. f(x)   x 3
k , x 3

Since function is continuous at x = 3
So RHL = LHL = ƒ(3) or limf(x)  f(3)
x 3

(x  3)  36
2
(x  3)2  (6)2
 lim  k  lim k
x 3 x 3 x 3 (x  3)
(x  9)(x  3)
 lim  k 12 = k  k = 12
x 3 (x  3)
3. y = a sin x + b cos x …..(1)
dy
  a cos x  b sin x (Differentiating w.r.t x)
dx
d2y
  a sin x  b cos x (again Differentiating w.r.t. x)
dx 2
d2y
  (a sin x  b cos x)
dx 2
d2y
 y 0 (from eq. (1))
dx 2
4. We have
 x2  y2  x2  y2
cos1  2 2 
 tan 1 a   cos(tan 1 a)   , say
 x  y  x2  y2
2x 2  1
  [Applying Componendo and dividendo]
2y 2
 1
x2 1  
 
y2 1  

E 129

Differentiating both sides with respect to x, we get
d 2 d 2 dy
y2 (x )  x 2 (y ) y 2  2x  x 2  2y
 dx dx  0  dx  0
2 2 4
(y ) y

dy dy dy 2xy 2 dy y
 2xy 2  2x 2 y  0  2x 2 y  2xy 2   2  
dx dx dx 2x y dx x
Aliter
 x2  y2 
We have cos1  2 2 
 tan 1 a
 x  y 
1   y x 2  y
 cos1  2 
 tan 1 a  2 tan 1    tan 1 a 
1   y x   x

y 1 y 1 
  tan 1    tan 1 a   tan  tan 1 a  
x 2 x 2 
dy
x  y 1
d y dx
     0  0
dx  x  x2
dy dy y
  x  y  0   
dx dx x
5. Given : x = a(cos2t + 2tsin2t) and y = a(sin2t – 2tcos2t)
dx
  a[ 2 sin 2t  2(2t cos 2t  sin 2t)] (differentiating w.r.t. t)
dt
dx
  4at cos 2t ......(1)
dt
and y = a(sin2t – 2tcos2t) (differentiating w.r.t. t)
dy
  a 2 cos 2t  2  2t sin 2t  cos 2t  
dt
dy
  4at sin 2t .......(2)
dt
dy dy / dt 4at sin 2t
  
dx dx / dt 4at cos 2t
dy
  tan 2t
dx
d2y dt
Now, 2
 2sec2 2t  (Again differentiating w.r.t. x)
dx dx
2
dy 1
 2
 2sec2 2t 
dx 4at cos2t
2
dy 1
 2
  sec3 2t
dx 2at

130 E

6. f(x) = |x| + |x – 1|
1  2x , 1  x  0

f (x)  1 , 0  x 1
2x  1 , 1  x  2

The possible point of discontinuity and non-differentiability are x = 0 and x = 1
(i) We have to check continuous at x = 0
RHL = lim f (x)  lim 1  1
x 0 x 0

LHL = lim ƒ(x)  lim  2x  1  1


x 0 x 0

f(0) = 1
Here RHL = LHL = f(0)

So, function is continuous at x = 0

(ii) We have to check continuous at x = 1

RHL = lim f (x)  lim 2x  1  1


x 1 x 1

LHL = lim f (x)  lim1



1
x 1 x 1

f(1) = 1

Here, RHL = LHL = f(1)

So, function is continuous at x = 1

(iii) We have to check differentiability at x = 0


f (0  h)  f (0) 1 1
R.H.D. = lim  lim 0
h 0 h h 0 h

f (0  h)  f (0) 2(h)  1  1
L.H.D. = lim  lim  2
h 0 h h  0 h
Here, R.H.D.  L.H.D.
So f(x) is not differentiable at x = 0.
(iv) We have to check differentiable at x = 1
f (1  h)  f (1) [2(1  h)  1]  1
R.H.D. = lim  lim 2
h 0 h h 0 h
f (1  h)  f (1) 11
L.H.D. = lim  lim 0
h 0 h h  0 h
Here, R.H.D.  L.H.D.
So f(x) is not differentiable at x = 1

E 131

7. Given, xy + yx = ab
Let u + v = ab Where u = xy & v = yx (Differentiating w.r.t. x)
du dv d(a b ) du dv
    0 ....(1)
dx dx dx dx dx
Now, u = xy
 log u = y log x (taking log on both sides)
d d 1 du 1 dy
  log u   (y log x)   y  log x
dx dx u dx x dx
du  1 dy  du y dy 
  u  y  log x    x y   log x  …..(2)
dx  x dx  dx x dx 
Again v = yx
log v = x log y (taking log on both sides)
d d
log v  (x log y)
dx dx
1 dv 1 dy
x  log y.1
v dx y dx
dv  x dy 
v  log y 
dx  y dx 
dv  x dy 
 yx   log y  …..(3)
dx  y dx 
Putting values from equation (2) and (3) in equation (1)
y dy   x dy  dy dy
 x y   log x   y x   log y  = 0  yx y 1  x y log x  xy x 1  y x log y  0
x dx   y dx  dx dx

dy y dy  x y 1y  y x log y 
  x log x  y x 1x   x y 1y  y x log y    y x 1 
dx dx  x log x  y x 
y
8. (ax  b)e x  x
x
e y/ x  .......(1) (taking log on both sides)
ax  b
y
  log x  log(ax  b)  (Differentiating w.r.t. x)
x
 dy   dy 
 x. dx  y  1 a  x. dx  y  ax  b  ax
     
 x ax  b  x(ax  b)
2 2
 x  x
   
dy bx
x y
dx (ax  b)
132 E

from equation (1)
dy
x  y  be y/ x .......(2)
dx
again differentiate w.r.t. x
 dy 
2 be y/x  x.  y 
d y dy dy  dx 
x. 2   
dx dx dx x2
from equation (2)
2
 d 2 y   dy 
x  2   x y
3

 dx   dx 
 a cos x  bsin x 
9. Let z = tan–1  
 b cos x  a sin x 
 a 
  tan x 
 z  tan 1  b  [Each term dividing by b cos x]
a
1  tan x 
 b 
a   xy  
 z = tan–1    tan 1 (tan x)  tan 1  1 1
  tan x  tan y 
b   1  xy  
a
 z = tan 1    x (Differentiating w.r.t. x)
b
dz
  1
dx
10. (i) We have to check the continuity at x = 0.
L.H.L. at x = 0 is given as :
 1   1 
 lim(0  h) tan 1    lim  h tan 1   0   0
h 0
 0  h  h 0  h 2
and R.H.L. at x = 0 is given as :
 1   1 
lim(0  h) tan 1    lim  h tan 1   0   0
h 0
0h  h 0
 h 2
and f(0) = 0 f(x) is continuous at x = 0.
Now, we have to check the differentiability at x = 0
 1 
( h) tan 1  –0
f(0  h)  f(0)   h    1   
L.H.D. = lim  lim = lim  tan 1    =
h 0 ( h) h 0 ( h) h 0
  h  2
1
h tan 1    0
f(0  h)  f(0) h 1 
and R.H.D. = lim  lim = lim tan 1   =
h 0 h h 0 h h0
h 2
 f '(0) does not exist [ L.H.D.  R.H.D.]

E 133

1
(ii) f(x) = x tan–1   (given)
x
1  1  1 1 .  x3 1  1 x 1
f '(x) = x.   2 
 tan 1    . 2   tan 1    tan 1
1 x 1 x x 
2
x 1 x 2
x
1 2  x 
x
x 1
f '(x) =  + tan 1
1 x 2
x
 (1  x 2 ).1  x.(2x)  1  1 
 f "(x) =    +  2 
 (1  x )2 2
 1  1   x 
 x2 
 
1  x 2  2x 2  x2  1  (1  x 2 ) 1
= – 2 2 
 2 
 2
  
 (1  x )  1  x  x  (1  x ) 1  x2
2 2

  f "(0) = –2
11. Given that f(x) = |x – 3| + |x – 2|
(x  3)  (x  2), x  2 2x  5 , x 2
 
f(x) = (x  3)  (x  2), 2  x  3  f(x) =  1 , 2x3
 (x  3)  (x  2), x  3  2x  5 , x3
 
(i) When x > 4 ;
f(x) = |x – 3| + |x– 2|
= (x – 3) + (x – 2) = 2x – 5 {according to the property of modulus function}
 f(x) = 2x – 5, which represents the height.
(ii) f(x) = |x – 3| + |x – 2| (given)
2x  5 , x2

f(x) =  1 , 2 x3
 2x  5 , x3

We have to check differentiability at x = 3
f(3  h)  f(3) {2(3  h)  5}  {2  3  5} 2h
  R.H.D. = lim , h  0 = lim = lim 2
h0 h h0 h h0 h

f(3  h)  f(3) 1  {2  3  5} 0
and L.H.D. = lim , h  0 = lim = lim 0
h0 h h 0 h h 0  h

 R.H.D. L.H.D.

 The given function is not differentiable at x = 3.


(iii) f(x) = 1 when 2 < x < 3
OR
(iii) If f(x) = [x] ; then the potter is trying to make a pot using this function. So, he will not be
able to get a pot as the greatest integer function is not continuous at integral points.

134 E

NCERT IMPORTANT QUESTIONS
1. Find all points of discontinuity of f, where f is defined by [Ex.5.1, Q.7]
 x  3, if x  3

f(x)  2x, if  3  x  3
6x  2, if x  3

2. Find the values of k so that the function f is continuous at the indicated point. [Ex.5.1, Q.26]
 k cos x 
   2x , if x  2 
f(x)   at x 
3,  2
if x 
 2
5, if x  2

3. Find the values of a and b such that the function defined by f(x)  ax  b, if 2  x  10 , is a
21, if x  10

continuous function. [Ex.5.1, Q.30]
4. Prove that the function f is given by f(x) = |x – 1|, x  R is not differentiable at x = 1.
[Ex.5.2, Q.9]
dy
5. Find : sin2x + cos2y = 1 [Ex.5.3, Q.8]
dx
 2x 
6. Differentiate w.r.t. 'x' : y  cos1  2 
, 1  x  1 [Ex.5.3, Q.13]
 1 x 
 1  1
7. Differentiate w.r.t. 'x' : y = sec 1  2  ,0x [Ex.5.3, Q.15]
 2x  1  2
dy
8. Find , if yx + xy + xx = ab [Example 30]
dx
x  1
 1 1 
9. Differentiate function w.r.t. 'x' :  x    x x  [Ex.5.5, Q.6]
 x
10. Differentiate function w.r.t. 'x' : xsin x + (sin x)cos x [Ex.5.5, Q.9]
dy y
11. Find : x + yx = 1 [Ex.5.5, Q.12]
dx
dy
12. Find : xy = e(x – y) [Ex.5.5, Q.15]
dx
13. If x and y are connected parametrically by the given equations, without eliminating the parameter,
dy
Find . x = a ( – sin ), y = a (1 + cos) [Ex.5.6, Q.6]
dx
14. If x and y are connected parametrically by the given equations, without eliminating the parameter,
dy  t
Find . x  a  cos t  log tan  , y  a sin t [Ex.5.6, Q.8]
dx  2
E 135

15. If x and y are connected parametrically by the given equations, without eliminating the parameter,
dy
Find . x = a (cos + sin), y = a(sin –  cos) [Ex.5.6, Q.10]
dx
1 1 dy y
16. If x  asin t , y  a cos t
, show that  [Ex.5.6, Q.11]
dx x

d2y dy
17. If y = sin–1x, show that (1  x 2 ) 2
x 0 [Example 38]
dx dx
d2y
18. If y = cos–1x, Find in terms of y alone. [Ex.5.7, Q.12]
dx 2
19. If y = 3 cos (log x) + 4 sin (log x), show that x2y2 + xy1 + y = 0 [Ex.5.7, Q.13]

mx nx d2y dy
20. If y = Ae + Be , show that 2
 (m  n)  mny  0 [Ex.5.7, Q.14]
dx dx
2
d2 y  dy 
21. If ey (x + 1) = 1, show that  [Ex.5.7, Q.16]
dx2  dx 
22. If y = (tan–1x)2, show that (x2 + 1)2 y2 + 2x (x2 + 1) y1 = 2 [Ex.5.7, Q.17]
23. Differentiate the following w.r.t. 'x' [Example 40]

 sin x   2x1 
(i) cos1 (sin x) (ii) tan 1   (iii) sin 1  x 
 1  cos x   1 4 
1 a
dy t  1
24. For a positive constant a, find , where y  a t , and x   t   [Example 42]
dx  t
 1  sin x  1  sin x  
25. Differentiate w.r.t. 'x' : cot 1  , 0  x  [Misc.Ex.,Q.6]
 1  sin x  1  sin x  2

26. Differentiate w.r.t. 'x' : xx + xa + ax + aa, for some fixed a > 0 and x > 0 [Misc.Ex., Q.10]
dy
27. Find , if y = sin–1x + sin 1 1  x 2 , 0 < x < 1 [Misc.Ex.,Q.13]
dx
dy 1
28. If x 1  y  y 1  x  0 , for –1 < x < 1, prove that  [Misc.Ex., Q.14]
dx (1  x)2
3
  dy 2  2
1    
  dx  
29. If (x – a)2 + (y – b)2 = c2, for some c > 0, Prove that  is a constant independent of
d2y
dx 2
'a' and 'b'. [Misc.Ex., Q.15]

136 E

dy cos2 (a  y)
30. If cos y = x cos (a + y), with cos a ±1, prove that  [Misc.Ex., Q.16]
dx sin a
d2y
31. If x = a(cost + t sint) and y = a (sint – t cost), find . [Misc.Ex., Q.17]
dx 2

a cos1 x d2y
2 dy
32. If y = e , 1  x  1 , show that (1 – x ) 2
 x  a 2y  0 [Misc.Ex., Q.22]
dx dx

ANSWER KEY
sin 2x
1. Discontinuous at x = 3 2. k=6 3. a = 2, b = 1 5.
sin 2y

2 2 [y x log y  y.x y1  x x (1  log x)]


6. 7.  8.
1  x2 1  x2 x.y x1  x y (log x)

x
 1   x2  1  1   1 1x  x  1  log x 
9.  x  x   x 2  1  log  x  x    x  x2 
      

 sin x 
10. xsin x   cos x log x   (sin x)cosx [cos x cot x  sin x logsin x]
 x 

yx y1  y x log y y(x  1) 


11.  12. 13.  cot 14. tan t
x y log x  xy x 1 x(y  1) 2

15. tan     18. –cot y cosec2y

1 2x1 log2
23. (i) –1 (ii) (iii)
2 1  4x
1
t
a t
loga 1
24. a 1
25. 26. x x (1  log x)  axa1  a x loga
 1 2
at  
 t

sec3 t
27. 0 31.
at

E 137

PREVIOUS YEAR QUESTIONS
1 MARK QUESTIONS

1. Differential of log [log (log x5)] w.r.t. x is : [CBSE 2022]


5 5
(A) (B)
5
x log(x ) log(log x 5 ) x log(log x 5 )
5x 4 5x 4
(C) (D)
log(x 5 ) log(log x 5 ) log(log x 5 )
Sol. (A) Let y = log [log (log x5)]
dy 1 1 1 4 5
= 5
 5
 5 × 5x = [1]
dx log(log x ) log(x ) x 5
x log(x ) log(log x 5 )
dx
2. If siny = x cos (a + y), then is: [CBSE 2022]
dy
cosa  cosa cosa  cosa
(A) (B) (C) (D)
cos2 (a  y) cos2 (a  y) sin 2 y sin 2 y
Sol. (A) Given ; siny = x cos (a + y)
sin y
 x =
cos(a  y)
dx cos(a  y).cos y  sin y.sin(a  y) dx cos[(a  y)  y] cosa
    = [1]
dy cos (a  y)
2
dy cos (a  y)
2
cos2 (a  y)
dy
3. If (x2 + y2)2 = xy, then is : [CBSE 2022]
dx
y  4x(x 2  y 2 ) y  4x(x 2  y 2 ) y  4x(x 2  y 2 ) 4y(x 2  y 2 )  x
(A) (B) (C) (D)
4y(x 2  y 2 )  x x  4(x 2  y 2 ) 4y(x 2  y 2 )  x y  4x(x 2  y 2 )
Sol. (C) Given ; (x2 + y2)2 = xy
 dy  dy
2 (x2 + y2) 2x  2y  = x  y.1
 dx  dx
dy dy y  4x(x 2  y 2 )
  4(x 2 y  y 3 )  x   y  4(x 3  xy 2 )     
dx dx 4y(x 2  y 2 )  x
 e3x  e 5x
 ; if x 0
4. The function f(x) =  x is continuous at x = 0 for the value of k, as :
 k ; if x  0

[CBSE 2022]
(A) 3 (B) 5 (C) 2 (D) 8
Sol. (D) The given function is continuous at x = 0, therefore
limf(0  h)  f(0)
h0

e3h  e 5h  e3h  1   e 5h  1 


 lim k  lim3    lim(5)   =k  k=3+5=8 [1]
h0 h h0
 3h  h0  5h 

138 E

dy
5. If x = 2 cos – cos2 and y = 2 sin – sin 2, then is : [CBSE 2022]
dx
cos   cos 2 cos   cos 2 cos   cos 2 cos 2  cos 
(A) B) (C) (D)
sin   sin 2 sin 2  sin  sin   sin 2 sin 2  sin 
dx
Sol. (B) x = 2 cos – cos2   2sin   2sin 2 
d
dy
y = 2 sin – sin 2  2 cos   2 cos2
d
dy cos   cos2
Hence ;  [1]
dx sin 2  sin 
 k cosx 
   2x ; if x  2 
6. If a function f defined by : f(x)   is continuous at x = then the value of
 3  2
; if x =
 2
k is : [CBSE 2022]
(A) 2 (B) 3 (C) 6 (D) –6

Sol. (C) Given ; f(x) is continuous at x =
2
 
k cos   h 
   2  3  k sinh
 R.H.L.  f    lim f   h   3  lim  lim 3
2 h 0
2  h 0   h  0 2h
  2  h 
2 
k sinh   sinh 
 lim =3 k = 6  lim  1 [1]
2  h0 h 

 h0 h 
7. If y = sin (m sin–1x), then which one of the following equations is true ? [CBSE 2022]
d2y dy d2y dy
(A) (1  x 2 ) 2
 x  m2y  0 (B) (1  x 2 ) 2
 x  m2y  0
dx dx dx dx
d2y dy d2y dy
(C) (1  x 2 ) 2
 x  m2y  0 (D) (1  x 2 ) 2
x  m2x  0
dx dx dx dx
Sol. (B) y = sin (m sin–1x)
dy  m  2 dy
   cos(m sin 1 x) ×    1  x  m cos(m sin 1 x) 
dx  1 x 
2 dx

d 2 y  dy  1 1
   1  x
2
2
  . .(2x)  m 2 sin(m sin 1 x)  
dx  dx  2 1  x 2 1  x2
d2y dy
   (1  x 2 ) 2
 x  m2y  0    
dx dx

E 139

8. The function f : R R given by f(x) = – |x – 1| is : [CBSE 2020]
(A) continuous as well as differentiable at x = 1
(B) not continuous but differentiable at x = 1
(C) continuous but not differentiable at x = 1
(D) neither continuous nor differentiable at x = 1
Sol. (C) Given; f : R  R, f(x) = –|x – 1|
Here, f(x) is a modulus function.
We know that the modulus function is always continuous in R but not differentiable at that point
where it value becomes zero.
Hence; f(x) is continuous but not differentiable at x = 1. [1]
2 2 2
9. Differentiate sec (x ) with respect to x . [CBSE 2020]
OR
dy
If y = f(x2) and f '(x)  e x
, then find .
dx
Sol. Let u = sec2 (x2) and v = x2
2 2 2
du du / dx du 2sec(x ).sec(x ).tan(x ).2x du
Now,      2sec 2 (x 2 ) tan(x 2 )
dv dv / dx dv 2x dv

d(sec2 (x2 ))
So, 2
 2sec2 (x2 ).tan(x2 ) [1]
d(x )
OR
Given, y = f(x2) and f '(x) = e x

dy 2 x2
Now;  f '(x 2 )2x and f '(x ) = e
dx
dy
 e x2
.2x  2x.e x [1]
dx

 kx2  5 if x 1
10. Find the value of k, so that the function f(x)   is continuous at x = 1.
 2 if x 1

[CBSE 2020]
 kx 2  5 , if x  1
Sol. Given that f(x)  
2 , if x  1
f(x) is continuous at x = 1
so lim f(x)  lim f(x)  f(1)  lim 2  lim kx2  5  k  5 2 = k + 5 k = –3 [1]
x 1 x 1– x 1 x 1–

dy
11. If y  cos  
3x , then find
dx
. [CBSE 2019]

Sol. y = cos  
3x 
dy
dx
 – sin  3x  1
2 3x
 3 
 3
2 3x
sin  3x  [1]

140 E

2 MARKS QUESTIONS
ax  1 if x  3
12. Find the relationship between a and b so that the function f defined by f(x) =  .
 bx  3 if x  3
Is continuous at x = 3. [CBSE 2021 C]
OR
Check the differentiability of f(x) = |x – 3| at x = 3.
ax  1 if x  3
Sol. Given, f(x) =  is continuous at x = 3.
 bx  3 if x  3
 L.H.L = R.H.L [1]
 lim f(3 – h) = lim f(3 + h)  lim a(3 – h) + 1 = lim b(3 + h) + 3
h 0 h 0 h 0 h 0

2
 3a + 1 = 3b + 3 3a – 3b = 2 or a – b = [1]
3
OR
(x  3); x  3
Given, f(x) = |x – 3|, f(x) = 
(x  3) ; x  3
L.H.D. at x = 3
f (3  h) – f (3) 3 h 3 – 0 h
= lim = lim = lim  1 [1]
h 0 h h 0 h h 0 (  h)

And R.H.D. at x = 3
f (3  h) – f (3) 3 h 3 – 0 h
= lim = lim = lim  1
h 0 h h 0 h h  0 h
 L.H.D  R.H.D

  Given function f(x) is not differentiable at x = 3. [1]


sec x  1 
13. If f(x)  , find f '   [CBSE 2020]
sec x  1 3
OR
tan x
Find f '(x) if f(x) = (tan x) .
sec x  1 1  cos x 2 sin 2 x / 2
Sol. Given; f(x) =  f(x)   f(x)  [1]
sec x  1 1  cos x 2 cos2 x / 2
x
f(x)  tan ( x lie in 1st quard.)
2
f(x) = tan x/2
1
f '(x) = sec2 x/2 × (Differentiating w.r.t. x)
2
1 4 1 2
f '(/3) = sec2 /6 ×  f '(/3) =   [1]
2 3 2 3

E 141

OR
tan x
Given, f(x) = (tan x)
Taking log on both sides; we get :
log f(x) = log (tan x) tan x  log f(x) = tan x log (tan x)
Differentiating w.r.t. x; we get
 1
.f '(x)  tan x   sec 2 x  log tan x  sec 2 x [1]
f(x) tan x
f '(x)
 sec2 x  sec2 x log tan x
f(x)
f '(x) = f(x).[sec2x (1 + log tanx)]
f '(x) = (tan x) tan x . [sec2x (1 + log tanx)] [1]
 1  cos x 
14. Differentiate tan 1   with respect to x. [CBSE 2018]
 sin x 
1  cos x 
Sol. Let y  tan 1 
 sin x 
 x 
 2 cos 2
 y  tan 1  2 
x x
 2sin cos 
 2 2
 x
 cos
 y  tan 1  2   tan 1 cot x  [1]
x  2 
 sin 
 2
   x   x
Hence, y  tan 1  tan      y    
  2 2  2 2
dy 1 1
  0   (Differentiating w.r.t. x) [1]
dx 2 2

3 MARKS QUESTIONS
dy x
15. If x = acos + bsiny = asin – bcosthen show that   and hence show that
dx y
d2 y dy
y 2
2
x y 0 [CBSE 2021 C]
dx dx
OR
dy y(1  log y)
If ey – x = yx; Prove that 
dx x log y
Sol. Given, x = acos + bsin, and y = asin – bcos
Now,x2 + y2= (acos + bsin)2 + (asin – bcos)2
  a
= a 2 cos2   b2 sin 2   2ab cos  sin  + 2
sin 2   b2 cos2   2absin  cos  [1]
= a2 + b2

142 E

dy
 x2 + y2 = a2 + b2  2x + 2y =0
dx
(Differentiating both sides w.r.t.x)
dy
dy  x 2 y(1)  x
d y dx
    [1]
dx y dx 2 y2
(Again, differentiating w.r.t.x)
d2 y dy
 y2 2
x y 0 [1]
dx dx
OR
Given, ey – x = yx
 (y – x)log e = x log y
(Taking log on both the sides)
(y – x) = xlog y …..(1)
dy 1 dy
   1  x.  log y.1
dx y dx
(Differentiating both sides w.r.t. x) [1]
dy  x 
  1  – (1  log y)
dx  y 
dy (1  log y) y(1  log y)
  = [1]
dx  x (y  x)
1  y 
 
dy y(1  log y)
  (from 1) [1]
dx x log y
16. Differentiate sin2x w.r.t. ecosx. [CBSE 2021 C]
2 cosx
Sol. Let y = sin x and z = e
dy
  2sin x.cos x …...(1)
dx
(Differentiating w.r.t.x) [1]
dz
and  e cos x (  sin x) …...(2)
dx
(Differentiating w.r.t.x) [1]
dy dy / dx 2sin x.cos x
  =
dz dz / dx ecos x ( sin x)
[From equations (1) and (2)]
dy 2 cos x
  = – 2cosx e – cosx [1]
dz ecos x

E 143

4 MARKS QUESTIONS
y dy x  y
17. If tan 1    log x 2  y 2 , prove that  . [CBSE 2020]
x dx x  y

OR
1 d2 y dy
If y  eacos x ,  1  x  1 , then show that (1  x2 ) 2
x  a2y  0
dx dx

Sol. Given; tan 1    log x 2  y 2


y
x

Differentiating w.r.t. x; we get :


 dy 
 dy   2x  2y 
1 x y 1
  dx   dx  [1]
 y2    x y
2 2
2 x  y2
2
1  2   x 
2

 x 
x2
 dy  1 1  dy 
   x.  y   2  2 2 
xy  [1]
x  y  
2 2
dx  x x  y   dx 

dy dy
 x yxy [1]
dx dx
dy dy x  y
 x  y xy   [1]
dx dx x  y

OR
1
Given y = ea cos x
.....(1)
differentiating w.r. t. x
dy 1 1
 ea cos x  a  .....(2) [1]
dx 1  x2
dy
1  x2 .  ay [from (1)]
dx
2
(1–x )    a 2 y 2
dy
2
 (Squaring on both sides) [1]
 dx 
Again, differentiating w.r. t. x; we get :
2
 dy   d 2 y   dy  dy
2(1 – x 2 )    2   2x    2a 2 y
 dx   dx   dx  dx

dy   
 2  
 1 x 2
d2 y dy
 x  a2 y = 0 [1]
 dx   dx 2
dx 
d2 y dy  dy 
 (1–x2) 2
 x  a2y  0   0, as y is not constant  [1]
dx dx  dx 

144 E

dy x  y
18. If x = aet (sin t + cos t) and y = aet (sin t – cos t), then prove that  . [CBSE 2019]
dx x  y
OR
sin x cos x
Differentiate x + (sin x) with respect to x.
x  ae t  sin t  cos t 
Sol.
y  ae t  sin t  cos t 
given (Differentiating w.r.t. t)

dx
  a e t  cos t  sin t    sin t  cos t  e t  = aet (sin t + cos t) – aet (sin t – cos t)
dt
dx
 =x–y ......(1) [1½]
dt
dy
and, = a [et (cost + sint) + (sint – cost) et] = aet (cost + sint) + aet (sint – cost)
dt
dy
 = x+ y ......(2) [1½]
dt
dy dy / dt xy
Hence;  = From (1) and (2) [1]
dx dx / dt x–y
OR
sin x cos x
Let y = x + (sinx)
y=u+v Where u = xsinx & v = (sinx)cosx
dy du dv
   ......(1) (Differentiating w.r.t. x) [1]
dx dx dx
Now; u = xsin x
 log u = sinx. logx (Taking log on both sides)
1 du 1
 sin x.  log x.cos x (Differentiating w.r.t. x)
u dx x
du  sin x  du  sin x 
 u  log x.cos x    xsin x   log x.cos x  ......(2) [1½]
dx  x  dx  x 
and v = (sinx)cos x
 log v = cosx. log (sinx) (Taking log on both sides)
1 dv 1
  cos x. .cos x  log  sin x  .   sin x 
v dx sin x
1 dv
  [cos x cot x – sin x log(sin x)]
v dx
dv
  v[cos x cot x  sin x log(sin x)] 
dx
dv
  sin x  cos x cot x  sin x log  sin x  
cos x
 .....(3) [1]
dx

E 145

From equations (1), (2) and (3); we get :
dy du dv
= 
dx dx dx
dy  sin x 
 log x.cos x    sin x  cos x.cot x  sin x.log sin x 
cosx
= xsin x  [½]
dx  x 
3/2
  dy  2 
1    
  dx  
19. If (x – a)2 + (y – b)2 = c2, for some c > 0, prove that  is a constant independent of
d2y
dx 2
a and b. [CBSE 2019]
2 2 2
Sol. Given, (x – a) + (y – b) = c ; for c > 0
Differentiating w.r.t 'x'; we get,
dy dy (x  a)
2(x – a) + 2(y – b) 0   ...... (1) [1½]
dx dx (y  b)
Again, differentiating w.r.t 'x'; we get :
 dy 
d2y  (y  b)(1  0)  (x  a) dx 
   ...... (2)
dx 2  (y  b) 2 
 
Putting the value of dy/dx from (1) to (2) ;
 (x  a) 
 (y  b)  (x  a)
2
d y (y  b)   (x  a)2  (y  b)2 
        = – c2/(y – b)3 ..... (3)
dx 2
 (y  b) 2
  (y  b) 3

 
3/2
  dy  2   (x  a) 2 
3/2

1     1  (y  b) 2 
  dx  
Now;  =   {from (1) and (3)} [1½]
d2y c 2

dx 2 (y  b)3
3/2
 (x  a) 2  (y  b) 2 
 (y  b) 2  c3
=   =  c; which is a constant independent of a and b. [1]
c2 / (y  b)3 c 2
d2 y dy
20. If y = sin(sinx), prove that 2
 tan x  y cos 2 x  0 . [CBSE 2018]
dx dx
Sol. Given y = sin(sinx) .....(1)
dy
= cos(sinx).cosx .....(2) (Differentiating. w.r.t. x) [1]
dx
d  dy  d d
   cos(sin x) (cos x)  cos x cos(sin x) (Again differentiating w.r.t. x)
dx  dx  dx dx
d2 y
= –sinx.cos(sinx) + cosx.(–sin{sinx}.cosx) ….(3) [1]
dx 2

146 E

1 dy
Put sin(sinx) = y from (1) and cos(sin x)  . from (2) in (3) [1]
(cos x) dx
d 2 y  sin x dy d2 y dy
2
 .  y cos x  2  tan x
2
 y cos 2 x  0 [1]
dx cos x dx dx dx
dy
21. If (x2 + y2)2 = xy, find . [CBSE 2018]
dx
OR
dy 
If x = a(2 – sin2) and y = a(1 – cos2), find , when   .
dx 3
Sol. Given, (x2 + y2)2 = xy  x4 + y4 + 2x2y2 = xy
(Differentiating both sides w.r.t. x)
dy  dy  dy
4x 3  4y3  2 2xy2  x 2 .2y   x y [2]
dx  dx  dx
dy
(4y3 + 4x2y – x) = (y – 4x3 – 4xy2) [1]
dx
dy y  4x 3  4xy 2
  [1]
dx 4y3  4x 2 y  x
OR
Given : x = a(2 – sin2) and y = a(1 – cos2)
dy dy / d
Function is parametric, therefore  ....(1)
dx dx / d
Now, x = a(2 – sin2)
Differentiating; w.r.t. ; we get :
dx dx
 a  2  cos 2 2    2a 1  cos 2 
d d

 2a  2sin 2   
dx dx
  4a sin 2  ....(2) [1]
d d
y = a(1 – cos2)
Differentiating w.r.t. ; we get :
dy dy
 2a sin 2    4a sin  cos  ....(3) [1]
d d
Put the value of equation (2) and (3) in (1); we get :
dy 4a sin  cos  dy cos 
   = cot [1]
dx 4a sin 
2
dx sin 
 dy    1
    cot    [1]
 dx  3 3
3

E 147

IMPORTANT NOTES
__________________________________________________________________________________
__________________________________________________________________________________
__________________________________________________________________________________
__________________________________________________________________________________
__________________________________________________________________________________
__________________________________________________________________________________
__________________________________________________________________________________
__________________________________________________________________________________
__________________________________________________________________________________
__________________________________________________________________________________
__________________________________________________________________________________
__________________________________________________________________________________
__________________________________________________________________________________
__________________________________________________________________________________
__________________________________________________________________________________
__________________________________________________________________________________
__________________________________________________________________________________
__________________________________________________________________________________
__________________________________________________________________________________
__________________________________________________________________________________
__________________________________________________________________________________
__________________________________________________________________________________
__________________________________________________________________________________
__________________________________________________________________________________
__________________________________________________________________________________
__________________________________________________________________________________
__________________________________________________________________________________
__________________________________________________________________________________
__________________________________________________________________________________

148 E

CHAPTER-6 : APPLICATION OF DERIVATIVES
1. RATE MEASUREMENT :
dy
Whenever one quantity y varies with another quantity x, satisfying some rule y = f(x), then
dx
dy 
(or f '(x)) represents the rate of change of y with respect to x and (or f'(a)) represents the
dx  x a
rate of change of y with respect to x at x = a.

Illustration 1 : Find the rate of change of volume of a sphere with respect to its surface area when the
radius is 2 cm.
Solution : Let r be the radius, V be the volume and S be the surface area of the sphere. Then,
4 3
V r and S  4r 2
3
dV
We have, to find when r = 2
dS
4 3
Now, V  r and S  4r 2
3
dV dS
  4 r 2 and  8r
dr dr
dV
dV dr dV 4r 2 r
    
dS dS dS 8r 2
dr
 dV  2
Hence,    1
 dS  r 2 2
Illustration 2 : The volume of a cube is increasing at a constant rate. Prove that the increase in surface
area varies inversely as the length of the edge of the cube. [Exemplar]
Solution : Let x be the length of each edge of the cube, S be its surface area and V be its volume at any
dV
time t. Then, S = 6x2 and V = x3. It is given that  k (constant)
dt
Now, V = x3
dV dx dx dx k
  3x 2  k  3x 2   2 …..(1)
dt dt dt dt 3x
and S = 6x2
dS dx dS  k 
   12x   12x  2  [Using (1)]
dt dt dt  3x 
dS 4k dS 1
   
dt x dt x

E 149

Illustration 3 : A man is walking at the rate of 6.5 km/hr towards the foot of a tower 120 m high. At what
rate is he approaching the top of the tower when he is 50 m away from the tower?
Solution : Let at any time t, the man be at distances of x and y metres
from the foot and top of the tower respectively. Then, B

y 2  x 2  (120)2 …..(1)
dy dx y
 2y  2x 120 m
dt dt

dy x dx
  x metre
dt y dt
A(man) O
dx
We are given that = – 6.5 km/hr (negative sign due to decreasing x).
dt
dy 6.5x
Therefore,  …..(2)
dt y

Putting x = 50 m in (1), we get : y  502  1202  130 m


dy 6.5  0.050
Putting x = 0.050 km, y = 0.130 km in (ii), we get,   2.5
dt 0.130
Thus, the man is approaching the top of the tower at the rate of 2.5 km/hr.
Illustration 4 : Water is dripping out at a steady rate of 1 cm3/sec through a tiny hole at the vertex of the
conical vessel, whose axis is vertical. When the slant height of water in the vessel is 4 cm, find the rate of

decrease of slant height, where the semi vertical angle of the conical vessel is . [Exemplar]
6
dv
Solution : Given that  1 cm3/s ….(1)
dt
where v is the volume of water in the conical vessel.
r
 3  1
From the Figure l = 4 cm, h = l cos  l and r  l sin  l .
6 2 6 2
1  l2 3 3 3 h
Therefore, v  r 2 h  l l . l
3 3 4 2 24 /6
dv 3 2 dl
 l
dt 8 dt
3 dl
Therefore, l  16. [Using (1)]
8 dt
dl 1
  cm/sec.
dt 2 3
1
Therefore, the rate of decrease of slant height  cm / sec .
2 3

150 E

2. INCREASING AND DECREASING FUNCTIONS :
1. Monotonicity At A Point :
(a) A function f(x) is called an increasing function at point x = a, if in a sufficiently small
neighbourhood of x = a ; f(a – h) < f(a) < f(a + h).
f(a + h)
f(a) f(a) f(a + h)
f(a – h)

f(a – h)

a–h a a+h a–h a a+h


(b) A function f(x) is called a decreasing function at point x = a, if in a sufficiently small
neighbourhood of x = a ; f(a – h) > f(a) > f(a + h)
f(a – h)
f(a – h)
f(a)
f(a +h)
f(a)
f(a + h)

a–h a a+h a – h x=a a+h


Note : If x = a is a boundary point, then use the appropriate one sides inequality to test
monotonicity of f(x).
f(a) f(a + h)

f(a – h) f(a)

x=a x=a
f(a) > f(a – h) f(a + h) > f(a)
increasing at x = a increasing at x = a
(c) Testing of monotonicity of differentiable function at a point :
(i) If f '(a) > 0, then f(x) is increasing at x = a.
(ii) If f '(a) < 0, then f(x) is decreasing at x = a.
+ –
(iii) If f '(a) = 0, then examine the sign of f '(a ) and f '(a ).
+ –
(1) If f '(a ) > 0 and f '(a ) > 0, then increasing.
+ –
(2) If f '(a ) < 0 and f '(a ) < 0, then decreasing.
(3) Otherwise neither increasing nor decreasing.
3
Illustration 5: Let f(x) = x – 3x + 2. Examine the nature of function as increasing and decreasing at
points x = 0 and 2.
3 2
Solution: Given f(x) = x – 3x + 2  f '(x) = 3(x – 1)

(i) f '(0) = –3  f (0)  0 decreasing at x = 0

(ii) f '(2) = 9  f (2)  0 increasing at x = 2

E 151

2. Monotonicity Over An Interval :
(a) A function f(x) is said to be monotonically increasing (MI) in (a, b) if f'(x)  0 where
equality holds only for discrete values of x i.e. f '(x) does not identically become zero
for x  (a, b) or any sub interval.
(b) f(x) is said to be monotonically decreasing (MD) in (a, b) if f '(x)  0 where equality
holds only for discrete values of x i.e. f '(x) does not identically become zero for
x  (a, b) or any sub interval.
Note :
(i) A function is said to be monotonic if it is either increasing or decreasing.
(ii) If a function is invertible, it has to be either increasing or decreasing.

4x3
Illustration 6: Determine, for which values of x, the function y  x 4  is increasing and for
3
which values, it is decreasing? [Exemplar]

4x3 dy
Solution: y  x 4    4x3  4x 2  4x 2 (x  1)
3 dx – – – + +
dy 0 1
For increasing  0 4x2(x – 1)  0 x  [1, )
dx
dy
For decreasing  0 4x2(x – 1)  0 x  (–, 0]  [0, 1].
dx
Therefore, f is decreasing in (–, 1] and f is increasing in [1, ).
x 2
Illustration 7: Find the intervals in which f  x   , x 0, is strictly increasing or decreasing.
2 x

x 2 1 2 x2  4
Solution: We have, f(x) =   f ' (x) =  
2 x 2 x2 2x 2
For strictly increasing, f ' (x) > 0
+ – +
x 4
2
(x  2)(x  2) – –2 2 
 2
> 0  0
2x x2
 x  (–, –2) (2, )
So, f(x) is strictly increasing on (–, –2) (2, ).
For strictly decreasing, f ' (x) < 0
x2  4 (x  2)(x  2)
 2
< 0   0 x  (–2, 2) – {0}
2x x2
Therefore, ƒ is strictly decreasing in (–2, 2) – {0} and ƒ is strictly increasing in (–, –2) (2, ).
152 E

4 4
Illustration 8: Separate the interval [0, /2] into sub-intervals in which f(x) = sin x + cos x is
increasing or decreasing.
4 4
Solution: We have, f(x) = sin x + cos x
3 3 2 2
 f '(x) = 4 sin x cos x – 4 cos x sin x  f '(x) = – 4 sin x cos x (cos x – sin x)
 f '(x) = – 2 (2 sin x cos x) (cos 2x) f '(x) = – 2 sin 2x cos 2x

 f '(x) = – sin 4x
n
Put f '(x) = 0  –sin4x = 0 sin 4x = 0 4x = nx =
4
    
 x  for n = 1  x  0,  
4   2 
   
When, x  0,  then, f '(x) < 0. So, f(x) is decreasing on 0, 4  .
 4
   
When, x   ,  then, f '(x) > 0So, f(x) is increasing on  ,  .
4 2 4 2
3. MAXIMA-MINIMA
1. Introduction : Some of the most important applications of differential calculus are
optimization problems, in which we are required to find the optimal (best) way of doing
something. Here are examples of such problems that we will solve in this chapter :
 What is the shape of a vessel that can with-stand maximum pressure ?
 What is the maximum acceleration of a space shuttle ? (This is an important question
to the astronauts who have to withstand the effects of acceleration)
 What is the radius of a contracted windpipe that expels air most rapidly during a cough?
These problems can be reduced to finding the maximum or minimum values of a function.
Let's first explain exactly what we mean by maxima and minima.
(a) Maxima (Local/Relative maxima) : A function f(x) is said to have a maxima
at x = a if there exists a neighbourhood (a – h, a + h) of a such that
f(a) > f(x) x (a – h, a + h) – {a}.
In such a case, f(a) is called the local maximum value of f(x) at x = a.
(absolute
absolutemaximum)
maximum (Relative maximum)
relative maximum
A No greater value of
of ff.
near
nearby.
by
A Q

y = f(x) B

P
relative minimum
(Relative minimum)
No R
Nosmaller
smallervalue
valueofoff.f (absoluteminimum
absolute minimum)
near
near by
by. NoNo smallervalue
smaller value of
of f.
Alsoa arelative
Also relative
minimum.
minimum
x=a x=b

E 153

(b) Minima (Local/Relative minima) : A function f(x) is said to have a minima
at x = a if there exists a neighbourhood (a – h, a + h) of a such that
f(a) < f(x)  x  (a – h, a + h) {a}. The value of the function at x = a i.e. f(a) is called
the local minimum value of f(x) at x = a.
(c) Absolute maxima (Global maxima) : A function f has an absolute maxima (or
global maxima) at c if f(c) f(x) for all x in D, where D is the domain of f. The
number f(c) is called the maximum value of f on D.
(d) Absolute minima (Global minima) : A function f has an absolute minima at c if
f(c) f(x) for all x in D and the number f(c) is called the minimum value of f on D.
where D is the domain of f.
Note: (i) The maximum and minimum values of a function are also known as local/relative
maxima or local/relative minima as these are the greatest and least values of the
function relative to some neighbourhood of the point in question.
(ii) The term 'extrema' is used both for maxima or minima.
(iii) The maximum (minimum) value of a function may not be the greatest (least) value in
a finite interval.
(iv) A function can have several extreme values and a local minimum value may even
be greater than a local maximum value.
2. Derivative Test for local Maxima/Minima :
(a) First derivative test : If ƒ '(x) = 0 at a point (say x = a) and
(i) If f'(x) changes sign from positive to negative in the neighbourhood of x = a;
then x = a is said to be a point local maxima.
(ii) If f'(x) changes sign from negative to positive in the neighbourhood of x = a;
then x = a is said to be a point local minima.
Y Y
dy
=0
dy dx dy
>0 <0
dx dx
dy
dy >0
<0 dx
dx dy
=0
dx
X X
O x=a O x=a
Note : If f '(x) does not change sign at a point x = a i.e. has the same sign in a certain
complete neighbourhood of a, then f(x) is either increasing or decreasing throughout
this neighbourhood implying that x=a is not a point of extremum of f. In this case;
x = a is a point of inflexion (neither maxima nor minima).

154 E

Illustration 9: Let f(x) = x + ; x 0. Discuss the local maximum and local minimum values of f(x).

Using the first derivative test. [Exemplar]


+ – +
1 x2  1 (x  1)(x  1)
Solution: Here, f' (x) = 1 – 2 =  –1 1
x x2 x2
Put f' (x) = 0 x = –1, 1
at x = –1
(1  h  1)(1  h  1) h(2  h)
f' (–1 – h) = f' (–1 – h) = 0
(1  h) 2
(1  h)2
(1  h  1)(1  h  1) h(2  h)
and f' (–1 + h) = f' (–1 + h) = 0
(1  h) 2
(1  h)2
Here, f' (x) change its sign from +ive to –ive at neighbourhood of x = –1, so x = –1 is point of
maxima.
 Local maximum value of f(x) = –2 at x = –1
at x = 1
(1  h  1)(1  h  1)  h(2  h)
f' (1 – h) = f' (1 – h) = 0
(1  h) 2
(1  h)2
(1  h  1)(1  h  1) h(2  h)
and f' (1 + h) = f' (1 + h) = 0
(1  h) 2
(1  h)2
Here, f' (x) change its sign from –ive to +ive at neighbourhood of x = 1, so x = 1 is point of
minima.
 Local minimum value of f(x) = 2 at x = 1
Illustration 10: Find the local maxima or local minima, if any, of the function f(x) = sin x + cos x,

0<x< using the first derivative test.
2
dy
Solution: We have, y = f(x) = sin x + cos x  = cos x – sin x
dx
dy + –
For a local maximum or a local minimum, we have =0 0 /4 /2
dx
  
 cos x – sin x = 0 tan x = 1 x =  0  x  2
4  
 dy
when 0 < x < cos x > sin x cos x – sin x > 0   0
4 dx
  dy
when <x< cos x < sin x cos x – sin x < 0  0 
4 2 dx
dy 
  Thus, changes its sign from positive to negative as x increases through .
dx 4

So, f(x) attains a local maximum at x = .
4

E 155

3 2
Illustration 11: Show that the function ƒ(x) = 4x – 18x + 27x – 7 has neither maxima nor minima.
[Exemplar]
3 2
Solution: f(x) = 4x – 18x + 27x – 7
2 2 2
f '(x) = 12x – 36x + 27 = 3(4x – 12x + 9) = 3(2x – 3)
3
f '(x) = 0  x  (critical point)
2 + +
x = 3/2
3 3
Since ƒ '(x) > 0 for all x  and for all x  .
2 2
3
Here, f' (x) does not change its sign at neighbourhood point of x =
2
3
Hence, x  is a point of inflexion i.e., neither a point of maxima nor a point of minima.
2
3
x is the only critical point, and ƒ has neither maxima nor minima.
2
(b) Second derivative test :
If f(x) is continuous and differentiable at x = a; where f '(a) = 0 and f''(a) also exists; then,
for ascertaining maxima/minima at x = a, second derivative test can be used :
(i) If f ''(a) > 0  x = a is a point of local minima.
(ii) If f ''(a) < 0  x = a is a point of local maxima.
(iii) If f ''(a) = 0  Second derivative test fails. To identify maxima/minima or point of
inflexion at this point; use first derivative test.
3 2
Illustration 12: If f (x) = 2x – 3x – 36x + 6 has local maximum and minimum at x = a and x = b
respectively, then find ordered pair (a, b)
3 2
Solution: f(x) = 2x – 3x – 36x + 6
2
f '(x) = 6x – 6x – 36 & f ''(x) = 12x – 6
2
Now f '(x) = 0  6(x – x – 6) = 0 
  (x – 3) (x + 2) = 0  x = –2, 3
f ''(–2) = –30 < 0
 x = –2 is a point of local maximum
f ''(3) = 30 > 0
 x = 3 is a point of local minimum
Hence, (–2, 3) is the required ordered pair.

156 E

Illustration 13: Find the point of maxima of f(x) = sin x (1 + cos x) in x  (0, /2).
1
Solution: Let f(x) = sin x (1+ cos x)  sin x + sin 2x
2
f '(x) = cos x + cos 2x
f ''(x) = – sin x – 2sin 2x
Now f '(x) = 0  cos x + cos2x = 0  cos 2x = cos (–x)  x = /3
Also f ''(/3) = – 3 /2 – 3 0
 f(x) has a maxima at x = /3

Illustration 14: Prove that f(x)  sin x  3 cos x has maximum value at x  . [Exemplar]
6
Solution: We have, f(x)  sin x  3 cos x
 f (x)  cos x  3( sin x)  cos x  3 sin x
For f (x)  0 , cos x  3 sin x
1  
 tan x   tan  x 
3 6 6
Again, differentiating f (x) , we get
f (x)   sin x  3 cos x
   1 3 1 3
At x  , f (x)   sin  3 cos    3 ·   =–2<0
6 6 6 2 2 2 2
 
Hence, at x  , f(x) has maximum value at is the point of local maxima.
6 6
Illustration 15: Find the maximum and minimum values of f(x) = sec x + log cos2 x, 0 < x < 2
               [Exemplar]
Solution: f(x) = sec x + 2 log cos x
Therefore, f (x)  sec x tan x  2 tan x  tan x(sec x  2)
1
f (x)  0 tan x = 0 or sec x = 2 or cos x =
2
 5
Therefore, possible values of x are x = 0, or x =  and x  or x 
3 3
Again, f (x)  sec 2 x(sec x  2)  tan x(sec x tan x)
= sec3 x + sec x tan2 x – 2sec2 x
= sec x (sec2 x + tan2 x – 2 sec x). We note that
f (0) = 1 (1 + 0 – 2) = – 1 < 0. Therefore, x = 0 is a point of maxima.
f () = –1 (1 + 0 + 2) = – 3 < 0. Therefore, x =  is a point of maxima.
 
f    = 2 (4 + 3 – 4) = 6 > 0. Therefore, x  is a point of minima.
3 3

E 157

 5  5
f    = 2 (4 + 3 – 4) = 6 > 0. Therefore, x  is a point of minima.
 3  3
Maximum value of y at x = 0 is 1+0=1
Maximum value of y at x =  is –1 + 0 = –1
 1
Minimum value of y at x = is 2  2 log  2(1  log 2)
3 2
5 1
Minimum value of y at x = is 2  2 log  2(1  log 2)
3 2
6. ABSOLUTE MAXIMUM AND MINIMUM VALUES :
Absolute maximum (minimum) value of any continuous function on [a, b] exist at any one of
local maximum [minimum] or at boundary points.
Illustration 16: Find the maximum and minimum values of f(x) = x + sin 2x in the interval [0, 2].
Solution: We have, f(x) = x + sin 2x
 f '(x) = 1 + 2 cos 2x f '(x) = 0 1 + 2 cos 2x = 0
1 2 2 
cos 2x =   cos  2x  2n   x  n 
2 3 3 3
 2 4 5
 x= , x= , x , x
3 3 3 3
Now, f(0) = 0 + sin 0 = 0,

  2  3  4  4 8 4 3
f     sin =  ; f     sin  
3 3 3 3 2  3  3 3 3 2
 2  2  4 2 3  5  5 10  5 3
f  =  sin   ; f     sin  
 3  3 3 3 2  3  3 3 3 2
and, f(2) = 2 + sin4 = 2 + 0 = 2.
Of these values, the maximum value is 2 and the minimum value is 0.
Thus, the maximum value of f(x) is 2 and the minimum value is 0.
4 3 2
Illustration 17: Find both the maximum and the minimum value of 3x – 8x + 12x – 48x + 1 on the
interval [1, 4].
4 3 2
Solution: Let f(x) = 3x – 8x + 12x – 48x + 1.
3 2
Then, f '(x) = 12x – 24x + 24x – 48
Now, f '(x) = 0
3 2 3 2
 12 x – 24 x + 24 x – 48 = 0  x – 2x + 2x – 4 = 0
2
 x (x – 2) + 2(x – 2) = 0 (x – 2) (x2 + 2) = 0
 x=2 [ x2 + 2 0]

Now, f(2) = – 63,


f(1) = – 40 and f(4) = 257.
So, the minimum and maximum values of f(x) on [1,4] are –63 and 257 respectively.
158 E

7. USEFUL FORMULAE OF MENSURATION :
(a) Volume of a cuboid = bh.
(b) Surface area of a cuboid = 2 (b + bh + h).
(c) Curved surface area of a cylinder = 2  rh.
2
(d) Total surface area of a cylinder = 2  rh + 2  r .
2
(e) Volume of cylinder = r h
(f) Curve surface area of cone = r
2
(g) Total surface area of cone = r + r
1 2
(h) Volume of a cone =  r h.
3
2
(i) Surface area of a sphere = 4  r
4 3
(j) Volume of a sphere =  r
3
1 2
(k) Area of a circular sector = r , when  is in radians.
2
8. SUMMARY OF WORKING RULES FOR SOLVING REAL LIFE OPTIMIZATION
PROBLEM :
First : When possible, draw a figure to illustrate the problem & label those parts that are
important in the problem. Constants & variables should be clearly distinguished.
Second : Write an equation for the quantity that is to be maximized or minimized. If
this quantity is denoted by ‘y’, it must be expressed in terms of a single
independent variable x. This may require some algebraic manipulations.
Third : If y = f (x) is a quantity to maximize or minimize then find those values of x for
which dy/dx = f (x) = 0.
Fourth : Using derivative test, test each value of x for which f (x) = 0 to determine whether
it provides a maximum or minimum or neither.
Fifth : If the derivative fails to exist at some point, examine this point as possible maximum
or minimum point.
Sixth : If the function y = f(x) is defined only for x  [a, b] then examine x = a and x = b
also for possible extreme values.
Illustration 18: Amongst all pairs of positive numbers with product 256, find those whose sum is the
least.
Solution: Let the required numbers be x and y. Then,
xy = 256 (given) ......(1)
Let S = x + y.

E 159

256
Then, S = x + [Using (1)]
x
dS 256 d 2S 512
 = 1  2 and  3
dx x dx 2 x
dS
For maximum or minimum values of S, we must have 0
dx
256
 1– = 0  x2 = 256
x2
 x = 16 [ x > 0]
 d 2S  512 1
Now,  2    0
 dx x 16 16 
3
8

Thus, S is minimum when x = 16. Putting x = 16 in (i) we get y = 16.


Hence, the required numbers are both equal to 16.
Illustration 19 : An Open box with a square base is to be made out of a given quantity of cardboard of

2 c3
area c square units. Show that the maximum volume of the box is cubic units.
6 3
Solution : Let each side of the square base be x and height by y then
c2 = total surface area of the box = 4 xy + x2 ..... (1)
Let v be the corresponding volume of the box, then
y
Volume of open box = Area of base × height
x
2 c x 
x
2
2 2
1 2 3
v=x y=x    v = (c x – x ), 0 < x < c ......(2)
 4x  4
dv 1 d 2v 1 3
= (c2 – 3x2) and = (–6x) = – x
dx 4 dx 2
4 2
For maximum value of volume
dv 1 c
= (c2 – 3x2) = 0 c2 – 3x2 = 0 x =
dx 4 3

 d 2v  3 c
 2 =– × <0
 dx  x  c 2 3
3

c
Therefore v is maximum at x = and maximum volume will be
3

1 2 c c3  1  2c3  c3
v  c      v = cubic units
4 3 3 3  43 3  6 3

160 E

x2 y2
Illustration 20 : Find the area of greatest rectangle that can be inscribed in an ellipse   1.
a 2 b2
Solution : Let ABCD is rectangle that can be inscribed in an ellipse
Y

(–a cos , b sin ) D C (a cos  , b sin )

X' X
(0, 0)O
(–a cos , –b sin ) A B (a cos  , –b sin
)
Y'
Area of rectangle (A) = (AB) × (BC) = (2acos)(2bsin)
A = 2absin2       
dA
 2  2ab.cos 2
d
dA
for maxima or minima 0
d
 
cos2 = 0  cos 2  cos  
2 4
d2A  d2A 
 8absin 2    2 0
d2  d at  
4


So, area is maximum at  
4
 
Thus area ABCD is 2ab.sin 2   2ab.sin = 2ab
4 2
Aliter
x2 y2
Given  1 .......(1)
a 2 b2 Y
Let point A be (x,y) for fig. AB = 2y and BC = 2x.
(–x,y) D A(x,y)
 Area of ABCD is
Let A = (AB) × (BC) X' X
(0,0)
A = 4xy  A2 = 16x2y2
C B(x,–y)
2 b2 2 2
2
(–x,–y)
Let S = A = 16x × 2 (a – x ) from equation (1)
a Y'
2 2
16b 16b
S
a 2
[x2.(a2 – x2)]  2 a 2 x 2  x 4
a
 
E 161

ds 16b2
 2 2a 2 x  4x3 
dx a
ds a
For Critical point 0x
dx 2

d 2s 16b2  d 2s  16b2
2
 
2 
2a 2
 12x 2

   2  2a 2  6a 2   0
2 
dx a  dx x  a a
2

a b 2 a2 b
for maximum area x  and y  a  
2 a 2 2
a b
Hence, maximum area = 4xy  4    2ab
2 2
Illustration 21: If the sum of lengths of the hypotenuse and a side of a right triangle is given, show
that the area of the triangle is maximum, when the angle between them is 60°.
Solution : Let y be the length of hypotenuse
A
and x be the length of base
Given = x + y y
–——
y2 –x2
1
A   x  y2  x2 
2 C x B

x2 2 2 x2
Let z = A2  (y  x ) z = A2  [ 2
 x2  2 x  x2 ]
4 4
1
z = A  [ x 2 x ]
2 2 2 3

4
dz dA 1 2 dz
 2A  [ (2x)  2 (3x 2 )]  0 (for maxima 0)
dx dx 4 dx
 = 3x  x = /3 and y = 2 /3

x 1  
cos     cos   
y 2 6 6

d2z 1
Now   2
 6 x 
dx2 2 
 d 2z  1
2

 2     2     0
2 2

 dx x  2 2
3

Thus area of triangle is maximum at  = /6

162 E

Illustration 22 : Tangent to the circle x2 + y2 = 4 at any point on it in the first quadrant makes
intercepts OA and OB on x and y axes respectively, O being the centre of the circle. Find the minimum
value of (OA + OB).
Solution : Circle x2 + y2 = 4
2 y
In  OPA, cos  
OA B
OA = 2 sec  P
2 x'  x
In OPB, cos(90  )  (0,0) O A
OB
OB = 2cosec.
Let Z = OA + OB Z = 2(sec + cosec) y'
dZ dZ
 2  sec  tan   cosec cot    0 (For minima 0)
d d

sin  cos 
 
cos  sin 2 
2


 sin3 = cos3tan3 = 1   
4
d2 Z
 2 sec3   sec  tan 2   cosec3  cosec cot 2 
d 2

d2 Z 
 0 at   Z is minimum.
d 2
4
  
Minimum value of Z  2  sec  cosec   Z  4 2
 4 4
Illustration 23: The three sides of a trapezium are equal each being 6 cm long; find the area of
trapezium when it is maximum.
Solution: Let ABCD be the given trapezium.

Let AM = BN = x cm then DM = CN =  36  x2 
 Area of the trapezium ABCD is
1  36  x2  = (6 + x)  36  x2 
S= (6 + x + 6 + x) ×
2
2 2 2
or S = (6 + x) (36 – x )
2 2 D 6 C
Let y = (6 + x) (36 – x )
dy 2 2
 = (6 + x) (–2x) + (36 – x ).2(6 + x) 6 6
dx
2 2
= 2(6 + x) (6 – 2x) = 4(3 – x)(6 + x)
A x M 6 N x B

E 163

d2y
and = – 12x(6 + x)
dx 2
dy
For maxima or minima of y, = 0 then x = 3 (x  – 6)
dx

d2y
  324  0
dx 2 x 3

 y is maximum at x = 3 then S is also maximum at x = 3

 S = (6 + 3) 36  9  27 3 cm 2

Illustration 24 : An isosceles triangle of vertical angle 2 is inscribed in a circle of radius a. Show that

the area of triangle is maximum when   . [Exemplar]
6
Solution: Let ABC be an isosceles triangle inscribed in the circle with radius a such that AB = AC.
AD = AO + OD = a + acos2 and BC = 2BD = 2asin2 (see figure)
1
Therefore, area of the triangle ABC i.e.   BC.AD
2
1 2
 2a sin 2.(a  a cos 2) = a sin2(1 + cos2)
2
1
   a 2 sin 2  a 2 sin 4
2
d
Therefore,  2a 2 cos 2  2a 2 cos 4
d
 2a 2 (cos 2  cos 4)
d
 0  cos2 = –cos4 = cos( – 4)
d

Therefore, 2 =  – 4   
6

d2  
 2a 2 (2sin 2  4sin 4)  0  at   
d2
 6

Therefore, Area of triangle is maximum when   .
6

164 E

EXERCISE–I
1. The function f(x) = 2 log (x – 2) – x2 + 4x + 1 strictly increases on the interval

(A) (1, 2) (B) (2, 3) (C) (1, 3) (D) (2, 4)

2. The maximum value of f(x) =(x – 2) (x – 3)2 is :

7 4
(A) (B) 3 (C) (D) 0
3 27

3. If the sides of an equilateral triangle are increasing at the rate of 2 cm/s then the rate at which
the area increases, when side is 10 cm, is

10
(A) 10 cm2/s (B) 3 cm2/s (C) 10 3 cm2/s (D) cm2/s
3
3 2
4. If f (x) = 2x – 3x – 36x + 6 has local maximum and minimum value at x = a and x = b
respectively; then ordered pair (a, b) is :

(A) (3, –2) (B) (2, –3) (C) (–2, 3) (D) (–3, 2)

5. A cylindrical vessel of radius 0.5m is filled with oil at the rate of 0.25 m3/minute. The rate at
which the surface of the oil is rising, is

(A) 1 m/minute (B) 2 m/minute (C) 5 m/minute (D) 1.25 m/minute

6. Two parts of 10 such that the sum of the twice of first with the square of second is minimum,
are-

(A) 9, 1 (B) 5, 5 (C) 4, 6 (D) 1, 9

7. For all values of x, function f(x) = 2x3 + 6x2 + 7x – 19 is -

(A) Increasing (B) Decreasing (C) Not monotonic (D) None of these

8. For what values of x is the rate of increase of x3 – 5x2 + 5x + 8 is twice the rate of increase of x?

1 1 1 1
(A) –3, – (B) – 3, (C) 3,  (D) 3,
3 3 3 3

9. If x = p and x = q are respectively the maximum and minimum points of the function
x5 – 5x4 + 5x3 – 10, then-

(A) p = 0, q = 1 (B) p = 1, q = 0 (C) p = 1, q = 3 (D) p = 3, q = 1

E 165

ASSERTION-REASON BASED QUESTIONS

In the following questions, a statement of assertion (A) is followed by a statement of Reason (R).
Choose the correct answer out of the following choices.
(A) Both A and R are true and R is the correct explanation of A.
(B) Both A and R are true but R is not the correct explanation of A.
(C) A is true but R is false.
(D) A is false but R is true.
10. Assertion (A) : The absolute maximum value of the function 2x3 – 24x in the interval [1, 3]
is 89.
Reason (R) : The absolute maximum value of the function can be obtained from the value of the
function at critical points and at boundary points.

ANSWER KEY
Q. No. 1 2 3 4 5 6 7 8 9 10
Ans. B C C C A A A D C D
166 E

SOLUTIONS
1. (B) We have, f(x) = 2 log (x – 2) – x2 + 4x + 1
2
 f (x)   2x  4
x2
– + – + – +
(x  1)(x  3)
 f (x)  2 1 2 3
x2
Since domain of f(x) is x – 2 > 0 x > 2
 f(x) is strictly increasing on (2, 3).
2. (C) Given, f(x) = (x – 2) (x – 3)2
f '(x) = (x – 2) 2(x – 3) + (x – 3)2 (1) = (x – 3) (3x – 7)
7
For maxima or minima f '(x) = 0 x = 3,
3
Now, f"(x) = (x – 3).3 + (3x – 7) . 1 = 6x – 16
 7
f"(3) = 2 > 0 and f"   = –2 < 0
 3
7
 Maxima at f"(x) < 0 at x =
3
2
7  7  4
Maximum value =   2   3  
3  3  27
3. (C) Let the side of an equilateral triangle be x cm.
3 
 Area of equilateral triangle, A  x …..(1)
4
dx
Also,  2 cm / s
dt
On differentiating Eq.(1) w.r.t. t, we get
dA 3 dx 3  dx 
 · 2x ·  · 2 ·10 · 2  x  10 and  2
dt 4 dt 4 dt 
 10 3 cm 2 / s
3 2
4. (C) Given; f(x) = 2x – 3x – 36x + 6
2
 f '(x) = 6x – 6x – 36 and f''(x) = 12x – 6
2
Now, f '(x) = 0 6(x – x – 6) = 0
 (x – 3) (x + 2) = 0  x = –2, 3
f ''(–2) = –30
 x = –2 is a point of local maxima ( a = –2)
Also; f ''(3) = 30
 x = 3 is a point of local minima ( b = 3)
Hence; (–2, 3) is the required ordered pair.

E 167

5. (A) V = r2h (for cylindrical vessel)
dV
Now, = 0.25  m3/minute
dt
and r = 0.5 m (given)
dV  dh 
  r 2  
dt  dt 
dh
or 0.25 =  × (0.5)2
dt
dh
 = 1 m/minute
dt
6. (A) Let two parts be x and (10 – x). If y = 2x + (10 – x)2
dy
Then = 2 – 2 (10 – x) = 2x – 18
dx
dy
Now = 0 x = 9
dx
d2y
Also then = 2 > 0. Hence when x = 9
dx 2
Value of y is minimum. So required two parts of 10 are 9 and 1.
7. (A) Given; f(x) = 2x3 + 6x2 + 7x – 19
 f '(x) = 6x2 + 12x + 7 = 6(x2 + 2x) + 7 = 6(x + 1)2 + 1;
which is positive for all values of x. Hence, f(x) is an increasing function.
8. (D) Let y = x3 – 5x2 + 5x + 8
dy dx
 (3x 2  10x  5) ….(1)
dt dt
dy dx
Given, 2
dt dt
2dx dx
= (3x2 – 10x + 5) [Using (1)]
dt dt
 3x2 – 10x + 3 = 0  3x2 – 9x – x + 3 = 0  3x(x – 3) – 1(x – 3) = 0
1
 (x – 3) (3x – 1) = 0  x = 3,
3
9. (C) Let f(x) = x5 – 5x4 + 5x3 – 10, then f '(x) = 5x4 – 20x3 + 15x2 = 5x2 (x – 1) (x – 3) and
f"(x) = 20x3 – 60x2 + 30x
for maxima and minima
f '(x) = 0 5x2 (x – 1) (x – 3) = 0
 x = 0, 1,3 Also f "(1) = –10 < 0
 x = 1 is a point of maxima p = 1 and f"(3) = 90 > 0
 x = 3 is a point of minima q = 3

168 E

10. (D) Let f(x)  2x3  24x

 f (x)  6x 2  24  6(x 2  4) = 6(x + 2) (x – 2)


For maxima or minima put f (x)  0
 6(x + 2) (x – 2) = 0
 x = 2, –2
We first consider the interval [1, 3]
So, we have to evaluate the value of f at the critical point x = 2 [1, 3] and at the end
points of [1, 3]
At x = 1, f(1) = 2 × 13 – 24 × 1 = – 22
At x = 2, f(2) = 2 × 23 – 24 × 2 = – 32
At x = 3, f(3) = 2 × 33 – 24 × 3 = – 18
 The absolute maximum value of f(x) in the interval [1, 3] is – 18 occurring at x – 3.
Hence, Assertion is false and Reason is true.

E 169

EXERCISE–II
4x 2  1
1. Find the intervals in which f(x)  is increasing or decreasing.
x

2. Find the minimum value of f(x), where f(x) = (2x2 – 3) + 3(3 – x) + 4

3. Show that the function f(x) = x3 – 3x2 + 6x – 100 is increasing on R.

4. Find the points of local maxima, local minima and the points of inflection of the function

f(x) = x5 – 5x4 + 5x3 – 1. Also, find the corresponding local maximum and local minimum

values. [Exemplar]


5. Show that f(x) = sin x (1 + cos x) is maximum at x  in the interval [0, ].
3

  
6. Prove that the function f(x) = tan x – 4x is strictly decreasing on   ,  . [Exemplar]
 3 3

7. For the curve y = 5x – 2x3, if x increases at the rate of 2 units/sec, then how fast is the slope of

the curve changing when x = 3? [Exemplar]

8. A metal box with a square base and vertical sides is to contain 1024 cm3 of water, the material

for the top and bottom costs Rs. 5 per cm2 and the material for the sides costs Rs.2.50 per cm2.

Find the least cost of the box. [Exemplar]

CASE STUDY-I
x
9. The sum of the surface areas S of a rectangular parallelopiped with sides x, 2x and and a
3
sphere of radius y is given to be constant.

x y

2
x
On the basis of above information answer the following questions :
(i) Find the combined volume V in terms of x and y.
(ii) If V is minimum, then find the relation between x and y.

170 E

CASE STUDY-II
10. A building has front gate has the figure as shown below.

10m

10m h 10m

x 10m x

It is in the shape of trapezium whose three sides other than base is 10 m. Height of the gate is h m.
On the basis of above figure answer the following questions.
(i) Express the area of gate A as a function of x.
(ii) Find the value of x when A is maximum.

E 171

SOLUTIONS
4x 2  1
1. We have, f(x) 
x
1 1 4x 2  1 + – +
Now, f(x)  4x   f (x)  4  2  
x x x2 –
For f(x) to be increasing, we must have Signs of f'(x) for different values of x

f (x)  0

4x 2  1
 2
 0 4x2 – 1 > 0
x
 1  1
 (2x – 1) (2x + 1) > 0   x   x    0
 2  2
1 1  1 1 
 x or x   x   ,     ,  
2 2  2 2 

 1 1  + – +
So, f(x) is increasing on  ,     ,   – 
 2 2 
For f(x) to be decreasing, we must have Signs of f'(x) for different values of
f (x)  (0)
4x 2  1
 2
 0 4x2 – 1 < 0 [ x 2  0]
x
1 1  1 1
 (2x – 1) (2x + 1) > 0    x   x    , 
2 2  2 2
 1   1
But domain (f) = R – {0}. So, f(x) is decreasing on   , 0    0,  .
 2   2
2. Given: f(x) = (2x – 3)+ 3(3 – x) + 4
2
…..(1)
f '(x) = 4x – 3
for maxima or minima f '(x) = 0
3
  4x – 3 = 0 x =
4
f "(x) = 4
3
So, x = is point of minima
4
 3  9   3
Now, f   =  2   3   3 3    4
4    16  4  
 3  15 9
f   9 4
4 8 4
 3  33 71
f    13  (min imum value)
4 8 8
172 E

3. f(x) = x3 – 3x2 + 6x – 100
f'(x) = 3x2 – 6x + 6 f'(x) = 3(x2 – 2x + 2)
f'(x) = 3[(x – 1)2 + 1] f'(x) > 0  x  R
So ƒ(x) is an increasing function on R
4. Let y = f(x) = x5 – 5x4 + 5x3 – 1. Then
dy
 5x 4  20x 3  15x 2 = 5x2 (x2 – 4x + 3) = 5x2 (x – 1) (x – 3)
dx
dy
The critical points of y = f(x) are given by 0
dx
dy
Now,  0  5x2 (x – 1) (x – 3) = 0 x = 0, x = 1, x = 3
dx
dy
Clearly, does not change its sign as x increases through 0. So, x = 0 is a point of inflection.
dx
dy
It is evident from figure that changes it sign from positive to negative as x increases through
dx
1. So, x = 1 is a point of local maximum.
+ + – +
– 0 1 3 
Signs of for different values of x.

The local maximum value of f(x) is f(1) = 1 – 5 + 5 – 1 = 0


dy
We observe, from figure, that changes its sign from negative to positive as x increases
dx
through 3. So, x = 3 is a point of local minimum.
The local minimum value of f(x) is f(3) = 35 – 5 × 34 + 5 × 33 – 1 = –28
5. We have, f(x) = sin x (1 + cos x)
 f(x) = sin x (1 + cos x)  f (x)  cos x(1  cos x)  sin 2 x
 f (x)  cos x  cos2 x  (1  cos2 x)  f (x)  2cos2 x  cos x  1  (2 cos x  1)(cos x  1)
At stationary points, we have
f (x)  0
1 
 (2 cos x – 1) (cos x + 1) = 0  cos x  or, cos x = – 1  x  or x = 
2 3
Let us now compute the values of x at these stationary points and at the end-points of the interval.
     3 3
Now, f(0) = 0, f    sin   1  cos   and f() = 0
3  3  3 4
3 3 3 3
Of these values, the maximum value is . Hence, f(x) attains the maximum value at
4 4

x .
3

E 173

6. We have, f(x) = tan x – 4x
 f (x)  sec 2 x  4
1  4 cos2 x 4 1  4 1  1 
 f (x)     cos x  =
2
  cos x   cos x 
2
cos x cos x  4
2
 cos x  2
2
 2 
  
Now, x    , 
 3 3
  1
   x    cos x  1
3 3 2
1 1 1 1 1
  cos x and   cos x   1
2 2 2 2 2
1 1 3
  cos x  0 and 1   cos x 
2 2 2
1 1
  cos x  0 and  cos x  0
2 2
1  1  4 1  1 
   cos x   cos x   0    cos x   cos x   0
2  2  cos x  2
2
 2 
 f (x)  0
  
Hence, f is strictly decreasing on   , 
 3 3
7. Let 'm' be the slope of the curve at an arbitrary point (x, y) on it. Then,
dy  dy 2
m=  m = 5 – 6x2  y  5x  2x  dx  5  6x 
3

dx  
dx dm
It is given that = 2 units/sec and we have to find when x = 3.
dt dt
dm  dx 
Now, m = 5 – 6x2  = – 12x  
dt  dt 
 dm   dx 
  dt  = – 12 × 3 × 2 = – 72 units/sec  x  3 and dt  2 
  x 3  
Thus, the slope of the curve is decreasing at the rate of 72 units/sec, when x is increasing at the
rate of 2 units/sec.
8. Let the length, breadth and height of the metal box be x cm, x cm and y cm respectively. It is
given that the box can contain 1024 cm3 of water.
1024
 1024  x 2 y   y  ….(1)
x2
Let C be the total cost in Rs. of material used to construct the box. Then,
5
C  5x 2  5x 2   4xy  C = 10x2 + 10xy
2

174 E

We have to find the least value of C.
Now, C = 10x2 + 10xy
1024 y
 C  10x 2  10x  2 [Using (1)] y y
x x
10240 y
 C  10x 2  x x
x x
2
dC 10240 dC 20480
  20x  2
and 2
 20x 
dx x dx x3
dC
The critical numbers for C are given by 0
dx
dC 10240
Now,  0  20x  2
 0 x3 = 5012 x3 = 83 x = 8
dx x

 d 2C  20480
Also,  2   20  0
 dx x 8 83
Thus, the cost of the box is least when x = 8. Putting x = 8 in (i), we obtain y = 16. So, the
dimensions of the box are 8 × 8 × 16.
Putting x = 8 and y = 16 in C = 10x2 + 10xy, we obtain C = 1920
Hence, the least cost of the box is Rs.1920.
 x x 
9. (i) Given, S = 2  x  2x  2x    x + 4y2 = 6x2 + 4y2 …..(1)
 3 3 
4 3 x 4 2
We have, V = y  x  2x  = y3  x3
3 3 3 3
4 3 2 3
v= y  x
3 3
From equation (1)
3/2
4  S  6x 2  2
v =    x3 …..(2)
3  4   3
dv 1 3 2
(ii)   (S  6x 2 )1/2 (12x)   3x 2
dx 6  2 3
3
=  (S  6x 2 )1/2 x  2x 2

dv
For minimum 0
dx
3 3x 3x
  (S  6x 2 )1/2 x  2x 2  0 2x2 = (S  6x 2 )1/2  2x 2  (4 y 2 )1/2
  
 2  x  3(4y 2 )1/2 4x2 = 9 × 4y2  x2 = 9y2 x = 3y

E 175

D 10 C

10. 10m h 10m

A B
x 10m x
P Q
(i) In APD,
We have, AP2 + PD2 = AD2
 x2 + h2 = (10)2  x2 + h2 = 100
We have, x2 + h2 = 100

 h  100 – x2
Now, Area of gate = Area of Trapezium ABCD
1 1
 A= (AB + CD) × DP  (10  10  2x)  100 – x 2  (10  x) 100 – x2
2 2

(ii) We have, A = (10 + x) 100  x2


2
dA x(10  x) 100 –10x – 2x
  100 – x 2 – 
dx 100 – x 2 100 – x2
dA
For maxima or minima, 0
dx
100 –10x – 2x2
 0
2
100 – x
 100 – 10x – 2x2 = 0 x2 + 5x – 50 = 0
 (x + 10) (x – 5) = 0
 x=5 [x > 0  x + 10  0]

(100 – 10x – 2x 2 )x
100 – x 2 (–10 – 4x) 
d2 A 100 – x 2 2x3 – 300x –1000
Again, 2   3
dx 100 – x 2 2 2
(100 – x )

d2 A 30
 2
– 0
dx x 5 75

So, A is maximum at x = 5m

176 E

NCERT IMPORTANT QUESTIONS
1. The length x of a rectangle is decreasing at the rate of 5 cm/minute and the width y is increasing
at the rate of 4 cm/minute. When x = 8cm and y = 6cm, find the rates of change of (a) the
perimeter, and (b) the area of the rectangle. [Ex.6.1, Q.7]
2. A balloon, which always remains spherical has a variable radius. Find the rate at which its
volume is increasing with the radius when the later is 10 cm. [Ex.6.1, Q.9]
3. A ladder 5 m long is leaning against a wall. The bottom of the ladder is pulled along the ground,
away from the wall, at the rate of 2 cm/s. How fast is its height on the wall decreasing when the
foot of the ladder is 4 m away from the wall? [Ex.6.1, Q.10]
4. Sand is pouring from a pipe at the rate of 12 cm3/s. The falling sand forms a cone on the ground
in such a way that the height of the cone is always one-sixth of the radius of the base. How fast is
the height of the sand cone increasing when the height is 4 cm? [Ex.6.1, Q.14]
5. Find the intervals in which the following functions are strictly increasing or decreasing:
(a) 6 – 9x – x2 [Ex.6.2, Q.6(d)]
(b) (x + 1)3 (x – 3)3 [Ex.6.2, Q.6(e)]
2x
6. Show that y = log (1 + x) – , x > – 1, is an increasing function of x throughout its
2x
domain. [Ex.6.2, Q.7]
7. Find the values of x for which y = [x(x – 2)]2 is an increasing function. [Ex.6.2, Q.8]
4sin   
8. Prove that y    is an increasing function of θ in  0, 2  . [Ex.6.2,Q.9]
(2  cos)
 
9. Find intervals in which the function given by f (x) = sin 3x, x 0,  is [Example 12]
 2
(a) increasing (b) decreasing.
10. Find the intervals in which the function f given by f (x) = sin x + cos x, 0  x  2 is strictly
increasing or strictly decreasing. [Example 13]
11. If length of three sides of a trapezium other than base are equal to 10 cm, then find the area of the
trapezium when it is maximum. [Example 25]
12. Prove that the radius of the right circular cylinder of greatest curved surface area which can be
inscribed in a given cone is half of that of the cone. [Example 26]
13. Show that of all the rectangles inscribed in a given fixed circle, the square has the maximum area.
[Ex.6.3, Q.19]
14. A wire of length 28 m is to be cut into two pieces. One of the pieces is to be made into a square
and the other into a circle. What should be the length of the two pieces so that the combined area
of the square and the circle is minimum? [Ex.6.3, Q.22]

E 177

8
15. Prove that the volume of the largest cone that can be inscribed in a sphere of radius R is of the
27
volume of the sphere. [Ex.6.3, Q.23]
16. Show that the right circular cone of least curved surface and given volume has an altitude equal
to 2 times the radius of the base. [Ex.6.3, Q.24]
17. Show that the semi-vertical angle of the cone of the maximum volume and of given slant height
is tan 1 2 . [Ex.6.3, Q.25]
18. Show that semi-vertical angle of right circular cone of given surface area and maximum volume
1
is sin 1   [Ex.6.3, Q.26]
3
19. A water tank has the shape of an inverted right circular cone with its axis vertical and vertex
lowermost. Its semi-vertical angle is tan–1 (0.5). Water is poured into it at a constant rate of 5
cubic metre per hour. Find the rate at which the level of the water is rising at the instant when the
depth of water in the tank is 4 m. [Example 31]
20. A man of height 2 metres walks at a uniform speed of 5 km/h away from a lamp post which is 6
metres high. Find the rate at which the length of his shadow increases. [Example 32]
21. Show that the function f given by f (x) = tan–1(sin x + cos x), x > 0 is always an strictly increasing
 
function in  0,  . [Example 34]
4
 
22. An open topped box is to be constructed by removing equal squares form each corner of
a 3 metre by 8 metre rectangular sheet of aluminium and folding up the sides. Find the volume of
the largest such box. [Example 36]
23. The two equal sides of an isosceles triangle with fixed base b are decreasing at the rate of 3 cm per
second. How fast is the area decreasing when the two equal sides are equal to the base ?
[Mis.Ex., Q.2]
4 sin x  2x  x cos x
24. Find the intervals in which the function f given by f(x)  is
2  cos x
(i) increasing (ii) decreasing [Mis.Ex., Q.3]
1
25. Find the intervals in which of the function f given by f(x)  x 3  , x  0 is
x3
(i) increasing (ii) decreasing [Mis.Ex., Q.4]
26. A window is in the form of rectangle surmounted by a semicircular opening. the total perimeter
of the window is 10 m. Find the dimensions of the window to admit maximum light through the
whole opening. [Mis.Ex., Q.8]
27. A point on the hypotenuse of a triangle is at distance a and b from the sides of the triangle.
Show that the minimum length of the hypotenuse is (a2/3 + b2/3)3/2. [Mis.Ex., Q.9]

178 E

28. Find the points at which the function f given by f(x) = (x – 2)4 (x + 1)3 has
(i) local maxima (ii) local minima (iii) point of inflexion [Mis.Ex., Q.10]
29. Show that altitude of the right circular cone of maximum volume that can be inscribed in a
4r
sphere or radius r is . [Mis.Ex., Q.12]
3
30. Show that the height of the cylinder of maximum volume that can be inscribed in sphere of
2R
radius R is . Also find the maximum volume. [Mis.Ex., Q.14]
3
31. Show that height of the cylinder of greatest volume which can be inscribed in a right circular
cone of height h and semi vertical angle '' is one-third of the cone and the greatest volume of
4
cylinder is h 3 tan 2  . [Mis.Ex., Q.15]
27

ANSWER KEY
1. (a) –2cm/min, (b) 2cm2/min 2. 400cm3/s
8 1
3. cm / s 4. cm / s
3 48
9 9
5. (a) S.I.  x <  ; S.D.  x >  (b) S.I. (1, 3) and (3, ) ; S.D.  (–, –1) and (–1, 1)
2 2
7. 0 < x < 1 and x > 2 9. (a) [0, /6] ; (b) [/6, /2]

10. S.I. (0, /4) (5/4, 2); S.D. (/4, 5)

112 28
11. 75 3 cm 2 14. cm, cm
4 4

19. 35/88 m/h 20. 5/2 km/hr.

200 3
22. m 23. b 3 cm 2 / s
27

 3  3
24. (i) 0  x  and  x  2 (ii) x
2 2 2 2
25. (i) x < –1 and x > 1 (ii) – 1 < x < 1
20 10
26. Length = m breadth = m
 
2
28. (i) local maxima at x  (ii) local minima at x= 2 (iii) point of inflexion at x = –1
7

E 179

PREVIOUS YEAR QUESTIONS

1 MARK QUESTIONS
1. A function f : R  R is defined as f(x) = x3 + 1. Then the function has [CBSE 2022]
(A) no minimum value (B) no maximum value
(C) both maximum and minimum values (D) neither maximum value nor minimum value
Sol. (D) Given ; f(x) = x3 + 1, f : R  R + +
– 0 +
f(x) = 3x2 = 0  x = 0
In (0, ) ; f'(x) > 0 and In (–, 0) ; f'(x) > 0 [1]
Hence ; the function has neither maximum nor minimum value.
2. The function y = x2e–x is decreasing in the interval : [CBSE 2022]
(A) (0, 2) (B) (2, ) (C) (–, 0) (D) (–, 0)  (2, )
Sol. (D) Given, y = x2 e–x
– + –
 y= 2xe–x + x2 (–e–x) = e–x {2x – x2} = e–x . x(2 – x) +
– 0 2
Put y' = 0  x = 0 or 2 [1]
3. The function f(x) = 2x3 – 15x2 + 36x + 6 is increasing in the interval : [CBSE 2022]
(A) (–, 2)  (3, ) (B) (–, 2) (C) (–, 2]  [3, ) (D) [3, )
Sol. (C)
 f(x) = 2x3 – 15x2 + 36x +6 + – +
2
 f'(x) = 6x – 30x + 36 = 6(x – 2) (x – 3) – 2 3 +
Put f '(x) = 0  x = 2 or 3
Hence ; f(x) is increasing in (–, 2]  [3, ) [f '(x)  0] [1]
x
1
4. The maximum value of   is : [CBSE 2022]
x
1/e
1/e 1
(A) e (B) e (C)   (D) ee
e
x
1 1
Sol. (A) Let y =    log y = x log  
x x
1 dy 1  1  1 1
  x   2  + log   .1 = – 1 + log   …(i)
y dx 1/ x  x  x x
x
dy  1    1 
    1  log   
dx  x    x 
dy 1   1 x 
If 0  log    1  log e     0
dx x   x  
1
 x
e

180 E

Now differentiating equation (i) w.r.to x, we get
2
1 d2 y 1  dy   1
2
 2    x  2 
y dx y  dx   x 
2
1 d 2 y 1  dy  1
 2
 2  
y dx y  dx  x
2
1 d2 y  1  1
 2
  log    1  [from (1)]
y dx  x  x

d2 y  1    1   1
x 2
 
   log    1  
dx 2  x   x
   x 

d2y 1
  2
at x  < 0
dx e

1
 Hence x  is a point of maxima.
e
x
1
 Maximum value of   = (e)1/e [1]
x
1 2  9
5. The absolute maximum value of the function f(x) = 4x – x in the interval  2, 2  is :
2  
[CBSE 2022]
(A) 8 (B) 9 (C) 6 (D) 10
1 2  9
Sol. (A) f(x) = 4x – x in  2, 
2  2
2x
 f '(x) = 4 –  4x
2
Put f '(x) = 0  x = 4
1
Now ; f(–2) = 4 × (–2) –  4 = –8 – 2 = –10,
2
1
f(4) = 4 × 4 –  16 = 16 – 8 = 8
2
2
9 9 1 9
and f    4     
2 2 2 2
36 81 144  81 63
=  = 
2 8 8 8
Hence ; absolute maximum value of f(x) = 8 at x = 4 [1]

E 181

6. In a sphere of radius r, a right circular cone of height h having maximum curved surface area is
inscribed. The expression for the square of curved surface of cone is : [CBSE 2022]
(A) 22rh (2rh + h2) (B) 2rh (2rh + h2) (C) 22r (2rh2 – h3) (D) 22r2 (2rh – h2)
Sol. (C) As per the given figure,
r2 = R2 + (h – r)2
A
 R2 = r2 – (h – r)2 = h(2r – h) ……(1)
r
 h2 + R2 = 2rh ……(2)
O  h
r
Now ; curved surface area of cone is :
R
S = R S2 = 2R22 B C
D

or S2 = 2 r 2  (h  r)2  (R 2  h 2 ) [from (1)]

= 2h (2r – h) (2rh) [from (2)]


= 22r (2rh2 – h3) [1]
3
7. Show that the function f(x) = + 7 is strictly decreasing for x  R-{0}. [CBSE 2021 C]
x
3 3
Sol. Given, f(x) =  7  f '(x) =  2
x x

 f '(x) < 0 For all x  R-{0}

Hence, The given functions strictly decreasing. [1]

8. The interval in which the function f given by f(x) = x2e–x is strictly increasing, is [CBSE 2020]
(A) (– , ) (B) (– , 0) (C) (2, ) (D) (0, 2)
Sol. (D) f(x) = x2.e–x (Differentiating w.r.t. x)
f '(x) = –x2e–x + 2xe–x
put f '(x) = 0
–x2.e–x + 2x.e–x = 0  e–x (2x – x2) = 0

e–x x (2–x) = 0  x = 0, 2 ( e–x 0, as e–x is always positive for all x  R )

– + –
Now – 0 2 

when x (0,2) , f '(x)  0

so f(x) is strictly increasing function in interval (0, 2) [1]

182 E

9. If the radius of the circle is increasing at the rate of 0.5 cm/sec, then the rate of increase of its
circumference is ............... . [CBSE 2020]
Sol. At any time t let r be the radius and c be the circumference of circle
dr
 0.5cm / sec (given)
dt
 c = 2r
dc dr
 2   2 (0.5) (Differentiating w.r.t. t)
dt dt
dc
  cm / sec [1]
dt
Rate of increase of its circumference is cm/sec.

4 MARKS QUESTIONS

CASE STUDY
10. In a residential society comprising of 100 houses,
there were 60 children between the ages of
10-15 years. They were inspired by their teachers to
start composting to ensure that biodegradable waste
is recycled. For this purpose, instead of each child
doing it for only his/her house, children convinced
the Residents welfare association to do it as a
society initiative. For this they identified a square area in the local park. Local authorities charged
amount of Rs. 50 per square metre for space so that there is no misuse of the space and Resident
welfare association takes it seriously. Association hired a labourer for digging takes it seriously.
Association hired a labourer for digging out 250 m3 and he charged Rs.400 × (depth)2. Association
will like to have minimum cost.
Based on this information, answer the any 4 of the following questions : [CBSE 2022]
(i) Let side of square plot is x m and its depth is h metres, then cost c for the pit is :
50 12500 250 250
(A)  400 h 2 (B)  400 h 2 (C)  h2 (D)  400h 2
h h h h
Sol. (B) Since depth of plot is h metres and side is x metres,
250
Volume of plot is x2 × h 250 = x2 × h x2 =  Area of plot
h
 Charge of space is `50 per m2
50  250 12500
So, cost of plot is `
h h
and charge of digging is 400 × h2 [1]
12500
So total cost is c =  400h 2 ……(1)
h

E 183

dc
(ii) Value of h(in m) for which  0 is :
dh
(A) 1.5 (B) 2 (C) 2.5 (D) 3
Sol. (C) From equation (1) ; we have
12500
c=  400h 2
h
Diff. w.r.t. to 'h'
dc 12500
  800h  0
dh h2
12500 125 5
 2
 800 h  h 3   h  = 2.5 m [1]
h 8 2
d 2c
(iii) is given by :
dh 2
25000 500 100 500
(A) 3
 800 (B) 3  800 (C)  800 (D) 2
h h h3 h3
dc 12500
Sol. (A)    800h
dh h2
Again diff. w.r.t. to h ; we get
d 2c 25000
  800 [1]
dh 2 h3
(iv) Value of x (in m) for minimum cost is :
5
(A) 5 (B) 10 (C) 5 5 (D) 10
3
Sol. (D)  h = 2.5,
d 2c
at h = 2.5 is > 0
dh 2
So h = 2.5 is point of minima.
250
 x 2 
h
250
x2 =  x2 = 100  x = 10 m [1]
2.5
(v) Total minimum cost of digging the pit (in Rupees) is :
(A) 4,100 (B) 7,500 (C) 7,850 (D) 3,220
Sol. (B)
12500
c =  400h 2 and h = 2.5
h
12500
 c=  400  (2.5)2
2.5
12500  25 
 c=   400  
25  4 
c = 5000 + 2500 = `7500 [1]

184 E

CASE STUDY
11. A factory makes an open cardboard box for a jewellery shop from a square sheet of side 18 cm by
cutting off squares from each corner and folding up the flaps

x
x

Based on the above information answer any four of the following five questions, if x is the
length of each square cut from corners. [CBSE 2021 C]
(i) The volume of the open box is :
(A) 4x(x2 – 18x + 81) (B) 2x(2x2 + 36x + 162)
(C) 2x(2x2 + 36x – 162) (D) 4x(x2 + 18x + 81)
(ii) The condition for the volume (V) to be maximum is :
dV d2V dV d2V
(A)  0 and 0 (B)  0 and 0
dx dx 2 dx dx 2
dV d2V dV d2V
(C)  0 and 0 (D)  0 and 0
dx dx 2 dx dx 2
(iii) What should be the side of square to be cut off so that the volume is maximum ?
(A) 6 cm (B) 9 cm (C) 3 cm (D) 4 cm
(iv) Maximum volume of the open box is
(A) 423 cm3 (B) 432 cm3 (C) 400 cm3 (D) 216 cm3
(v) The total area of the removed squares is :
(A) 324 cm2 (B) 144 cm2 (C) 36 cm2 (D) 64 cm2
Sol. Let the length of side of cutting square is x cm, then the length and the breadth of the box will be
(18 – 2x) cm each and the height of the box is x cm.
Let V be the volume of the open box formed by folding up the flaps, then
V = x(18 – 2x) (18 – 2x) = 4x(9 – x)2 = 4(x3 – 18x2 + 81x)
Now, differentiating w.r.t. x, we get
2
dV dV
 4(3x 2  36x  81)  12(x 2  12x  27) and  12(2x  12)  24(x  6)
dx dx
dV
For maxima or minima 0
dx

E 185

 12(x2 – 12x + 27) = 0 or (x – 3) (x – 9) = 0
x = 3, 9 (but x = 9 is not possible) therefore x = 3
 d2V 
   24(3  6)  –72  0 (Maxima)
 dx x 3
(i) (A) The volume of the open box is
V = x(18 – 2x) (18 – 2x) = 4x(x2 – 18x + 81) [1]
2
dV dV
(ii) (A)  0 and 0 [1]
dx dx 2
(iii) (C) 3cm [1]
(iv) (B) Maximum volume of the open box
V = 4 × 3 (9 – 18 × 3 + 81)= 12 (9 – 54 + 81) = 432 cm3 [1]
(v) (C) Total area of removed square = 4x2 = 4 × 9 = 36 cm2
x4
12. Find the intervals in which the function f(x)   x3  5x 2  24x  12 is [CBSE 2018]
4
(a) strictly increasing (b) strictly decreasing
x4
Sol. Given ƒ(x) = – x3 – 5x2 + 24x + 12
4
ƒ'(x) = x3 – 3x2 – 10x + 24
ƒ'(x) = (x – 2)(x – 4)(x + 3) [1]
therefore ƒ'(x) = 0, gives x = – 3, 2, 4 points divides the real line into four disjoint interval,
namely (–, –3), (–3, 2), (2, 4) and (4, ). [1]
– + – +

– –3 0 2 4 +

f '(x) > 0, when x  (–3, 2)  (4,  )


and f '(x) < 0, when x  (–  , –3)  (2, 4)
Hence f(x) is strictly increasing in (–3, 2) (4, ) & strictly decreasing in (–, –3) (2, 4) [2]
13. An open tank with a square base and vertical sides is to be constructed from a metal sheet so as
to hold a given quantity of water. Show that the cost of material will be least when depth of the
tank is half of its width. If the cost is to be borne by nearby settled lower income families, for
whom water will be provided ? [CBSE 2018]
Sol. Let v and A be the volume and area of tank, x be the side of square base and y be the height of tank.
then v = x2y .....(1)
x
To show (minimum)A = x2 + 4xy at y =
2

186 E

v 4v
 A = x2 + 4xy  A = x2 + 4x  2  A = x2 + [1]
x  x
dA 4v
= 2x – 2
dx x
dA
for maxima or minima =0
dx
y
4v 4v 3 x
 2x – 2 = 0  2x = 2  2v = x [2]
x x x

x
From eq. (1) v = x2y 2x2y = x3  y =
2
d2 A 8v  d2 A  3
Now 2
= 2 + 3
  2  > 0 when x = 2v [1]
dx x  dx 
Hence cost of material will be least at y = x/2

6 MARKS QUESTIONS

14. Amongst all open (from the top) right circular cylindrical boxes of volume 125 cm3, find the dimensions
of the box which has the least surface area. [CBSE 2020]

Sol.
h

Let r be the radius, h be the height and V be the volume of the cylindrical box respectively [1]
V 125  cm3 (given)
125
r2h = 125  h= …..(1) [1]
r2
Let S be the surface area of cylindrical box

S = 2rh + r2 = r (2h + r) S = r  2  r  [from equation(1)]


250
 [1]
 r 

250  dS 250  d S 500  2


S=  r 2    2  2 r and 2  3  2 [1]
r dr r dr r
dS
for maxima or minima 0 [1]
dr

250   d2S 
 2 r  2
r3 = 125 r = 5   2   0
r  dr r 5
So surface area S of cylinder is least at r = 5
Hence; dimensions of box are r = 5 cm and h = 5 cm [1]

E 187

15. A tank with rectangular base and rectangular sides, open at the top is to be constructed so that its
depth is 2 m and volume is 8 m3. If building of tank costs Rs.70 per sq metres for the base and
Rs 45 per square metre for sides. What is the cost of least expensive tank? [CBSE 2019]
Sol. Given: A tank with rectangular base and rectangular sides, open at the top.
and Depth of tank = 2 m
Let x m be the length and y m be the breadth of the base of tank.
Volume of tank (= lbh) = x.y.2 = 8 m3 (given)
8 4
 y  …...(1)
2x x
Now cost of building the base of the tank at the given rate of
Rs 70 per square metre is Rs 70 xy …...(2)
Again cost of building the four sides (walls) of the tank at the rate of Rs 45 per square metre.
= 45(x.2 + x.2 + y.2 + y.2) = 45(4x + 4y) [2]
= Rs (180x + 180y) …...(3)
Let z denote the total cost of building the tank.
Adding (2) and (3), z = 70xy + 180x + 180y
4 4 4
Putting y  from (1), z = 70x. + 180x + 180.
x x x
720
or z = 280 + 180x + …..(4)
x
dz 720 d 2z 1440
  0  180  2 and  3 [1½]
dx x dx2 x
dz
Putting  0 to find turning points, we have
dx
720 720
180  2
 0  180  2  180x2 = 720
x x
720
 x2   4  x = 2 ( x being length can't negative)
180

d 2z 1440 1440
At x = 2,  3   180(ve) [1½]
dx 2 x 8
 z is minimum at x = 2
Putting x = 2 in (4), minimum cost
720
z = 280 + 180(2) + = 280 + 360 + 360 = 280 + 720 = Rs. 1000. [1]
2

188 E

You might also like